You are on page 1of 337

Instructors Manual

Financial Accounting & Reporting


Tenth edition Barry Elliott and Jamie Elliott
For further instructor material please visit:

www.pearsoned.co.uk/elliott
ISBN 0 273 70365 X

Pearson Education Limited 2006

Lecturers adopting the main text are permitted to download the manual as required.

1 Pearson Education Limited 2006

Pearson Education Limited Edinburgh Gate Harlow Essex CM20 2JE England and Associated Companies around the world Visit us on the World Wide Web at: www.pearsoned.co.uk First published 2006 Pearson Education 2006 The rights of Barry Elliott and Jamie Elliott to be identified as authors of this Work have been asserted by them in accordance with the Copyright, Designs and Patents Act 1988. ISBN: 0 273 70365 X All rights reserved. Permission is hereby given for the material in this publication to be reproduced for OHP transparencies and student handouts, without express permission of the Publishers, for educational purposes only. In all other cases, no part of this publication may be reproduced, stored in a retrieval system, or transmitted in any form or by any means, electronic, mechanical, photocopying, recording, or otherwise without either the prior written permission of the Publishers or a licence permitting restricted copying in the United Kingdom issued by the Copyright Licensing Agency Ltd, 90 Tottenham Court Road, London W1T 4LP.

2 Pearson Education Limited 2006

Barry Elliott and Jamie Elliott: Financial Accounting and Reporting (tenth edition) Instructors Manual

Contents
Chapter 1 Chapter 2 Chapter 3 Chapter 4 Chapter 5 Chapter 6 Chapter 7 Chapter 8 Chapter 9 Chapter 10 Chapter 11 Chapter 12 Chapter 13 Chapter 14 Chapter 15 Chapter 16 Chapter 17 Chapter 18 Chapter 19 Chapter 20 Chapter 21 Chapter 22 Chapter 23 Chapter 24 Chapter 25 Chapter 26 Chapter 27 Chapter 28 Chapter 29 Chapter 30 Chapter 31 Chapter 32 4 13 25 38 58 64 67 73 89 109 114 130 136 143 149 153 162 172 181 189 196 211 215 233 242 253 265 275 301 318 320 329

3 Pearson Education Limited 2006

Barry Elliott and Jamie Elliott: Financial Accounting and Reporting (tenth edition) Instructors Manual

CHAPTER 1

Chapter 1: Question 1 Jane Parker


(i) Cash budget (000)
Jan Initial capital Customers Total receipts Machinery Motor vehicles Premises Drawings Suppliers Rates Wages General expenses Insurance Total payments Net cash flow Balance b/f Balance c/f . 132.45 17.55 17.55 2.25 150.00 . 150.00 30.00 24.00 75.00 1.20 1.20 30.00 1.20 2.25 0.75 . 35.40 (35.40) 17.55 (17.85) 2.25 0.75 . 52.20 (52.20) (17.85) (70.05) 2.25 0.75 . 64.20 18.30 (70.05) (51.75) 2.25 0.75 . 64.20 (4.20) (51.75) (55.95) 2.25 0.75 2.10 66.30 8.70 (55.95) (47.25) (47.25) 1.20 48.00 1.20 60.00 1.20 60.00 1.20 60.00 . . . . Feb Mar Apr 82.50 . 82.50 60.00 60.00 75.00 75.00 May June Total 232.50 135.00 367.50 30.00 24.00 75.00 7.20 258.00 1.20 13.50 3.75 2.10 414.75

(ii)

Cash flow summary statement (000)


Realised operating cash flows for the period ended 30 June 20X1 135.00 258.00 1.20 13.50 3.75 2.10 278.55 (143.55)

Receipts from customers Payments to: Suppliers Rates Wages General expenses Insurance

4 Pearson Education Limited 2006

Barry Elliott and Jamie Elliott: Financial Accounting and Reporting (tenth edition) Instructors Manual

For information only Statement of financial position as at 30 June 20X1 000 Capital introduced withdrawn Net operating cash flows :Realised :Unrealised 232.50 (7.20) (143.55) (7.80) 73.95 Premises (NRV) Vehicles (NRV) Machinery (NRV) Net cash balance 75.00 19.20 27.00 (47.25) 73.95

(iii)

Further information re Jane Parker


(a) Nature of business linked to Parkers business background, technical ability, special skills, know how, existing/terminated business involvement, contacts, associates, related parties. (b) Type of business unit to be used and rationale for its selection. (c) Sources of long and short-term capital. (d) Products life cycle, cash flow projections over product life cycle. (e) Initial investment in fixed assets and their terminal value at the end of the life cycle. (f) Parkers attitude to risk and how this affects the choice of discount rate and payback period.

5 Pearson Education Limited 2006

Barry Elliott and Jamie Elliott: Financial Accounting and Reporting (tenth edition) Instructors Manual

Chapter 1: Question 2 Mr Norman


(a)
Sales Gross profit Purchases Payments

Purchases budget (000)


Jan 15.00 3.00 12.00 Feb 20.00 4.00 16.00 12.00 Mar 35.00 7.00 28.00 16.00 Apr 40.00 8.00 32.00 28.00 May 40.00 8.00 32.00 32.00 June 45.00 9.00 36.00 32.00

Notes: This is a start-up situation. Purchases = projected sales less a gross margin on sales at 20%. Goods are bought in month of sale; assume stocks remain constant.

(b)

Cash flow statement (000)


Jan Feb 10.00 7.50 17.50 Mar 17.50 10.00 27.50 Apr 20.00 17.50 37.50 May 20.00 20.00 40.00 June 22.50 20.00 42.50 Total 50.00 97.50 75.00 222.50 80.00 2.20 12.00 0.30 0.60 3.50 86.30 0.60 15.10 2.40 (28.80) (26.40) 2.20 16.00 0.40 0.60 19.20 8.30 (26.40) (18.10) 2.20 28.00 0.70 0.60 31.50 6.00 (18.10) (12.10) 2.20 32.00 0.80 0.60 35.60 4.40 (12.10) (7.70) 2.20 32.00 0.80 0.60 35.60 6.90 (7.70) (0.80) (0.80) 13.20 120.00 3.00 3.60 3.50 223.30 50.00 7.50 57.50

Initial capital Cash Sales Credit Sales

Premises Rent and rates Suppliers Commission Wages Insurance

80.00 2.20

Net cash flow Balance b/f Balance c/f

(28.80) (28.80)

6 Pearson Education Limited 2006

Barry Elliott and Jamie Elliott: Financial Accounting and Reporting (tenth edition) Instructors Manual

(c)

Statements of operating cash flows and financial position


Realised operating cash flows for the period ended 30 June 20X8
000

Receipts from customers Payments to: Suppliers Rates Wages Commission Insurance 120.00 13.20 3.60 3.00 3.50

172.50

143.30 29.20

Notes: The cash flow statement with summary attached is effectively a six-month cash budget showing the cash received, cash paid each month and the resulting month-end balances. It is necessary to separate sales and purchase transactions into cash and on credit and to identify clearly the month of receipt and payment. Commission is paid in the month after the sale is made, and all other cash flows are clearly indicated and allocated to specific months. Note that the format of the cash flow statement brings out key figures for management decision and control, e.g. Month-end balances assists in the control of liquidity cash deficiencies identifies how much must be financed early warning allows management to approach appropriate sources cash surpluses identifies amounts to be invested on best terms. Statement of financial position as at 30 June 20X8
000 Capital introduction Net operating cash flows :Realised :Unrealised (4.00) 75.20 Premises (NRV) Net cash balance 76.00 (0.80) 75.20 50.00 29.20

7 Pearson Education Limited 2006

Barry Elliott and Jamie Elliott: Financial Accounting and Reporting (tenth edition) Instructors Manual

Notes: This statement shows net assets of 75,200 make up: premises 76,000 less the negative cash balance 800. The negative cash balance indicates need for overdraft arrangements. The statement is based on cash flow concept it ignores accrual-based figures (36,900 less 25,250) accruals are not regarded as real assets and liabilities critics of the cash flow concept would maintain that its utility has therefore been seriously diminished.

(d)

Letter to bank requesting an overdraft facility should include


The maximum overdraft facility of 28,800 required at the end of January will be eliminated by July

Overdraft will fall progressively as per the cash budget. It might be practical to request a limit of 30,000 for the full six-month period reducing to 15,000 thereafter to allow for contingencies. The facility only to be called on as required. confirm that it is based on the most likely scenario agree a repayment schedule.

Refer to the Cash Budget to support the request

Specify that collateral security is available in form of premises if it should be required. If not an existing customer give outline details of business background explain future plans.

8 Pearson Education Limited 2006

Barry Elliott and Jamie Elliott: Financial Accounting and Reporting (tenth edition) Instructors Manual

Chapter 1: Question 3 Fred and Sally


(i) (a) Decision to close in winter
April Opening balance Sales (Wk1) Boards (custom-built) 14,100 Salary in Lanzarote: Fred and Sally: income Staff: income Purchases Materials for boards (custom-built)(Wk2) Wages Rent Misc. costs 1,000 500 500 12,100 Wk1 (120,000/6) = + 20,000 6,000 (paid by credit card) 14,000 (cash received in month) 5,760 (last months credit card sales less 4%) 19,760
9 Pearson Education Limited 2006

May 12,100 19,760 31,860

June 29,860 19,760 10,000 59,620

July 35,120 19,760 10,000 64,880

Aug 40,380 19,760 10,000 70,140

Sept 45,640 19,760 10,000 75,400

Oct 5,760

Nov

Dec 33,660

Jan 34,660

Feb 35,660

March 36,660

100 14,000

50,900 45,160 10,000 10,000 66,660 55,160

33,660

34,660

35,660

36,660

500 500 16,000 6,500 1,000 500 500 29,860 1,000 500 500 35,120 16,000 6,500 1,000 500 500 40,380 16,000 6,500 1,000 500 500 45,640 16,000 6,500 1,000 500 500 50,900 16,000 6,500

500 500 16,000 6,500

500 500

500 500

500 500

500 500

45,160 33,660

34,660

35,660

36,660

37,660

Wk 2 Under this alternative Materials Sails (not Dryline)

Per board 500 150

Total

650 10 = 6,500

Barry Elliott and Jamie Elliott: Financial Accounting and Reporting (tenth edition) Instructors Manual

(i) (b) Decision to remain open in winter and keep Dryline agency
April Opening balance Sales Boards May June July Aug 30,891 5,760 10,000 53,824 14,933 6,000 1,000 500 500 30,891 Sept 37,718 5,760 10,000 60,651 14,933 6,000 1,000 500 500 37,718 Oct 5,760 1,400 1,000 10,000 1,000 15,676 32,343 Purchases (Wk3/4) Boards Wages Rent Misc costs Closing balance Wk 3 20,000 60% Non Dryline 12,000 80% 9,600 + 8,000 2/3 Dryline 5,333 14,933 1,493 14,933 600 1,000 500 500 11,583 6,000 1,000 500 500 9,410 40,170 14,933 6,000 1,000 500 500 17,237 46,997 14,933 6,000 1,000 500 500 24,064 Wk4 2,000 60% 1,200 80% 960 + 800 2/3 533 1,493
10 Pearson Education Limited 2006

Nov

Dec 38,761 1,976

Jan 37,994 1,976 1,000

Feb 37,227 1,976 1,000 40,970 1,493 600 1,000 500 150 37,227

March

100 11,5839,41017,237 24,064 14,000 19,76019,76019,760 19,760 576 1,000 10,000

32,295 40,528

1976 1,000 40,203 1,493 600 1,000 500 150 36,460

10,000 10,000 54,878 44,271 14,933 6,000 1,000 500 150 1,493 600 1,000 500 150 42,504 1,493 600 1,000 500 150 38,761

41,737 1,493 600 1,000 500 150 37,994

32,295 40,528

Barry Elliott and Jamie Elliott: Financial Accounting and Reporting (tenth edition) Instructors Manual

Notes supporting (i) (a) Assuming closure during the winter months means that: there are no sales from October to March salaries of 1,000 per month are received from October to March a start-up situation arises each April with consequential effect on cash from sales of both custom-built and non-custom-built boards.

Cash from sales is calculated as follows: Non-custom-built (i) 70% received immediately (ii) 30% via credit card finance less 4% (6,000 240) = = 14,000 5,760 19,760

However, only 14,000 is received in first month and 5,760 in October. Custom-built Received per month two months in arrears Cash payments are calculated as follows: Non-custom-built 80% of the sales value is paid two months after purchase Custom-built Materials cost 650 each; total 6,500 per month; paid 2 months after purchase Other fixed costs Wages, rent, miscellaneous costs at uniform monthly rate of 2,000 for 6 months Projected cash balances show an upward trend to September and then they fall away. Notes supporting (i) (b) Cash receipts are calculated as follows: Non-custom-built Receipts are in two cycles: April to September and October to March. April cash received is 14,576 made up of 70% of 20,000 being the April sales plus 96% of Marchs credit card sales (i.e. 96% of 600). From May to September (inclusive) cash receipts will be the same as calculated on the closure assumption i.e. 19,760 per month. October receipts are 7,160 i.e. 5,760 from September plus 1,400 from October sales with 1,976 again for five subsequent months. Custom-built Cash receipts from June to November will be 10,000 per month as under the closure assumption. = 10,000

11 Pearson Education Limited 2006

Barry Elliott and Jamie Elliott: Financial Accounting and Reporting (tenth edition) Instructors Manual

In the winter months there will be a further 6,000 except that customers took an unexplained average of three months in winter, receipts are nil in December. Cash payments are calculated as follows: Non-custom-built From May to October purchases are 14,933 and 10% of this for the remaining six months Custom-built As for closure assumption Other fixed costs Wages, rent, miscellaneous costs at uniform monthly rate of 2,000 for 6 months

(ii)

Additional information required in order to advise

At first glance closing in the winter generates more cash but: 1. What about loss of future earnings from Dryline? 2. Is there any potential damage to customer goodwill? 3. Will this allow competition to creep into market? Notes relating to part (a): This is a start-up situation. Purchases = projected sales less a gross margin on sales at 20%. Goods are bought in month of sale; assume stocks remain constant. Objectives What are Fred/Sallys objectives? Is it a short-term pleasant lifestyle with high income, or long-term market maximisation? state of market whether demand for Dryline sails is expanding or contracting possibility of other agencies possibility of opening in other areas possibility of increasing winter sales higher profile for custom-made boards e.g. outline in Lanzarote whether or not the custom-made boards are independent of the rest of the business i.e. a different market.

We need to know more about the business e.g:

Demand

12 Pearson Education Limited 2006

Barry Elliott and Jamie Elliott: Financial Accounting and Reporting (tenth edition) Instructors Manual

CHAPTER 2

Chapter 2: Question 1 Jane Parker


(a) Cash budget (000)
Jan Initial capital Customers Total receipts Machinery Motor vehicles Premises Drawings Suppliers Rates Wages General expenses Insurance 132.75 Net cash flow Balance b/f Balance c/f 17.25 17.25 34.50 (34.50) 17.25 (17.25) 52.50 (52.50) (17.25) (69.75) 64.50 (4.50) (69.75) (74.25) 64.50 10.50 (74.25) (63.75) 2.25 2.25 0.75 2.25 0.75 2.25 0.75 2.25 0.75 2.25 0.75 26.40 90.90 59.10 (63.75) (4.65) (4.65) 13.50 3.75 26.40 439.65 150.00 30.00 24.00 75.00 1.50 1.50 30.00 1.50 48.00 1.50 60.00 1.50 60.00 1.50 60.00 150.00 60.00 60.00 75.00 75.00 Feb Mar Apr May June 75.00 75.00 150.00 Total 225.00 210.00 435.00 30.00 24.00 75.00 9.00 258.00

All balances are overdrawn except for January 20X1


Feb o/d 17.25 Mar 69.75 Apr 74.25 May 63.75 June 4.65

Note: No entries will be made for the 20X0/X1 local taxes that are paid in Feb 20X2 this situation arose because Jane Parker had assumed that the business would only pay the taxes from the start of the tax year e.g. 1.4.20X1. However, there will be an entry in the profit and loss and balance sheet.

13 Pearson Education Limited 2006

Barry Elliott and Jamie Elliott: Financial Accounting and Reporting (tenth edition) Instructors Manual

(b) Jane Parker profit and loss account for six months ended 30.6.20X1
000 Sales [60.00 + (5 75.00)] 378.00 (30.00) 348.00 87.00 13.50 4.50 4.00 13.20 2.40 1.50 39.10 47.90 Purchases Closing inventory Cost of sales Gross profit Wages General expenses Local taxes [1.1.X1 30.6.X1] Insurance Depreciation Vehicles Machinery Net profit 000 435.00

Budgeted balance sheet as at 30 June 20X1


Capital Net profit Less: Drawings (9.00) 263.90 225.00 47.90

Non-current assets
Premises Vehicles Less: Depreciation Machinery Less: Depreciation 24.00 2.40 30.00 1.50 28.50 21.60 75.00

Current assets
Inventory trade receivables [3 75.00] Insurance 30.00 225.00 13.20 268.20

14 Pearson Education Limited 2006

Barry Elliott and Jamie Elliott: Financial Accounting and Reporting (tenth edition) Instructors Manual

Current liabilities
Trade payables Local taxes [1.1.X1 30.6.X1] Bank overdraft General expenses Net current assets 120.00 4.00 4.65 0.75 (129.40) 138.80 263.90

(c) Possible action to deal with exceeding agreed overdraft limit


Approach bank to re-negotiate the overdraft or arrange a loan facility for an agreed term. The amount and period that additional facilities are required depends on preparing a projected cash flow statement for a longer period taking into account future plans e.g. owners drawings requirement, any additional capital expenditure. In particular, consider alternatives such as: leasing vehicles and/or machinery mortgaging the property getting debts in quicker introducing more capital obtaining or providing loan capital.

15 Pearson Education Limited 2006

Barry Elliott and Jamie Elliott: Financial Accounting and Reporting (tenth edition) Instructors Manual

Chapter 2: Question 2 Mr Norman


(a) Purchases budget ($000)
Jan Sales units Closing inventory + Closing inventory Purchases units 0.55 2.20 1.65 Feb 2.20 0.55 0.96 2.61 Purchases $000 Jan Feb Mar Apr May Jun (2,200 40) (2,610 40) (3,990 40) (4,400 40) (4,540 40) (5,090 40) 88.00 104.40 159.60 176.00 181.60 203.60 913.20 Mar 3.85 0.96 1.10 3.99 Apr 4.40 1.10 1.10 4.40 Sales $000 82.50 110.00 192.50 220.00 220.00 247.50 1,072.50 (1,650 50) (2,200 50) (3,850 50) (4,400 50) (4,400 50) (4,950 50) May 4.40 1.10 1.24 4.54 Jun 4.95 1.24 1.38 5.09

(b)

Cash flow forecast (000)


Jan Feb 55.00 41.25 191.25 96.25 1.65 88.00 8.00 17.00 0.35 105.35 85.90 85.90 114.65 (18.40) 85.90 67.50 131.60 19.65 67.50 87.15 188.45 17.80 87.15 104.95 205.40 14.60 104.95 119.55 211.00 22.75 119.55 142.30 8.00 17.00 80.00 2.20 104.40 8.00 17.00 3.85 159.60 8.00 17.00 4.40 176.00 8.00 17.00 4.40 181.60 8.00 17.00 Mar 96.25 55.00 151.25 Apr 110.00 96.25 206.25 May 110.00 110.00 220.00 June 123.75 110.00 233.75 Total 150.00 536.25 412.50 1098.75 80.00 16.50 709.60 48.00 102.00 0.35 956.45 150.00 41.25

Initial capital Cash Sales Credit sales Premises Commission Suppliers Administration Wages Insurance Total payments Net cash flow Balance b/f Balance c/f

16 Pearson Education Limited 2006

Barry Elliott and Jamie Elliott: Financial Accounting and Reporting (tenth edition) Instructors Manual

(c)

Budgeted profit and loss account for six months ended 30 June 20X8
$000 $000 1072.50 913.20 (55.20) 858.00 214.50 102.00 48.00 21.45 0.18 8.00 179.63 34.87

Sales Purchases Closing inventory [1,380 units 40] Cost of sales Gross profit Wages Administration Commission [2% of 1072.50] Insurance Amortisation of lease Net profit

Budgeted balance sheet as at 30 June 20X8


$000 Capital Net profit Non-current assets Leasehold premises Less amortisation Current assets Inventory Trade receivables Prepayments insurance Cash Current liabilities Trade payables Commission Net current assets 203.60 4.95 208.55 112.87 184.87 55.20 123.75 .17 142.30 321.42 80.00 (8.00) 72.00 $000 150.00 34.87 184.87

17 Pearson Education Limited 2006

Barry Elliott and Jamie Elliott: Financial Accounting and Reporting (tenth edition) Instructors Manual

(d)

Investment of surplus funds


Acid test ratio At the end of the first six months trading Normans balance sheet shows that the acid test ratio is 1.28:1 (266.22/208.55) this is higher than the basic 1:1 ratio but it should be compared with that of similar businesses in the same industry to establish a norm. It would appear however that the business has surplus funds to invest. Amount to invest A projected cash flow statement is required taking into account future plans re owners drawing requirements, future capital commitments and working capital criteria e.g. debtor collection and creditor payment terms. Period to invest The projected cash flow will give an indication of the period of the investment e.g. it could range from overnight on the money market to term investments.

The important aspect is that the owner should be aware of the projected cash flows so that return on surplus funds can be maximised.

18 Pearson Education Limited 2006

Barry Elliott and Jamie Elliott: Financial Accounting and Reporting (tenth edition) Instructors Manual

Chapter 2: Question 3 The Piano Warehouse Company Limited


(a) Conventional profit calculation
(i) 4 sold Sales Materials Labour Overheads Cost of production Less: inventory Cost of sales Profit 2,000 2,800 800 5,600 5,600 2,400 8,000 900 800 100 1,800 1,800 2,200 2,200 800 1,800 400 (ii) 2 part made (iii) 2 repaired/sold 3,000

Comments on profit calculation: (i) (ii) 4 pianos sold: profit is simply sales less cost. 2 partly completed pianos: No profit has been recognised Delay recognition until sale to comply with matching concept Work-in-progress carried forward. Match with revenue when sales take place (iii) presumably in the following year. Rebuilt pianos: profit has been taken in full Assumes no default on hire purchase payments. An alternative procedure that could be considered would be to provide for unrealised profit on instalments outstanding create a provision of 533 [2/3 of 800] this reduces profit from 800 to 267 [1/3 of 800].

(b)

It has been assumed that the work-in-progress is held at cost; and no profit has been recognised. The payment in advance of 900 will be shown as a creditor in the balance sheet.

19 Pearson Education Limited 2006

Barry Elliott and Jamie Elliott: Financial Accounting and Reporting (tenth edition) Instructors Manual

Given the facts that there is an assured market for work-in-progress; and the pianos are 50% complete

a case could be made out for attributing a % of the total profit of 900 (4,500 3,600 estimated cost to produce). Except in the simplest cases, the matching process can be highly subjective. The conventional approach is to recognise revenue at the point of sale or, alternatively on completion of production or receipt of payment.

The core determinant is the identification of the completion of the earnings cycle as signalled by a critical event e.g. production, delivery, receipt of payment. Applying these alternatives to The Piano Warehouse Company and assuming the critical event is: (i) On sale/delivery Recognise profit from the sale of 4 pianos (ii) During production Recognise profit from the 50% complete pianos (iii) On receipt of payment Restrict the profit from the 2 rebuilt pianos to the proportion of cash received. A more detailed consideration in IAS 18 relating to the sale of goods suggests the critical event for revenue recognition hinges on 2 conditions: There has been a transfer to the buyer of the significant risks and rewards of ownership, and no significant uncertainty exists concerning the sale price, costs, likelihood of rejection and return. Accruals for recognising profit Matching for associating costs incurred and to be incurred in arriving at a profit figure Prudence in attributing/not attributing profit on work-in-progress.

(c)

The significant cost conventions are:

(d)

The profit for the year could be increased by the recognition of a proportion of the profit on the two pianos in the course of production with an estimate of the amount earned to date. The maximum figure would normally be 450 [50% of the estimated total profit] but this would be dependent upon being reasonably certain that the future costs will not exceed the estimate. Conversely, it could be argued that the 800 profit on the rebuilt pianos should be reduced to 267.

20 Pearson Education Limited 2006

Barry Elliott and Jamie Elliott: Financial Accounting and Reporting (tenth edition) Instructors Manual

Chapter 2: Question 4 FRRP and Wiggins Group


FRRP membership
The Chairman of the Panel is Richard Sykes QC and the Deputy Chairman Matthew Patient CBE. There are currently 15 other Panel members drawn from a broad spectrum of commerce and the professions. Individual cases are normally dealt with by specially constituted groups of 5 or more members.

FRRP remit
The remit of the Financial Reporting Review Panel is to examine the annual accounts of public and large private companies to see whether they comply with the requirements of the Companies Act 1985. Within this framework a main focus is material departures from accounting standards where such a departure results in the accounts in question not giving a true and fair view as required by the Act.

Action if accounts are defective


Where a companys accounts are defective the Panel will, wherever possible, endeavour to secure their revision by voluntary means, but if this approach fails it is empowered to make an application to the court under section 245B of the Companies Act 1985 for an order compelling their revision. To date no court applications have been made, though in some instances the necessary steps have been at an advanced stage.

FRRP is reactive not proactive


The Panel does not itself monitor or actively initiate scrutinies of company accounts for possible defects, but acts on matters drawn to its attention, either directly or indirectly. The Panels responsibilities do not extend to the directors report, summary financial statements or interim statements.

Wiggins: Revenue recognition


Company policy
It is the companys accounting policy to recognise revenue in respect of commercial property sales on exchange of contract. In the 1999 accounts however, two sales had been recognised on the basis of non-binding heads of agreement and a third in respect of a contract dated after the end of the 1999 accounting period. In the 2000 accounts, the directors accepted that these treatments were not appropriate and adjusted the 1999 comparatives by the 21.5m turnover incorrectly recognised.

21 Pearson Education Limited 2006

Barry Elliott and Jamie Elliott: Financial Accounting and Reporting (tenth edition) Instructors Manual

FRRP questions raised re contracts


In those accounts, the directors did not address other concerns previously expressed by the Panel in connection with these contracts. One of the contracts was conditional upon the company obtaining planning permission on terms satisfactory to the purchaser without which he had certain rights not to proceed. A second contract had the appearance of a financing transaction rather than an outright sale which, under FRS 5, should not be recognised until the risks and rewards of ownership pass at a future date. In the particular circumstances, the Panel was of the view that neither contract could be recognised in the 2000 accounts. The directors have now accepted the Panels view and, in the revised accounts, have adjusted for these and other sales in earlier years, applying the same principles. Revenue recognition

Company policy in 1999 accounts


The 1999 accounts contained an accounting policy for turnover in the following terms: Commercial property sales are recognised at the date of exchange of contract, providing the Group is reasonably assured of the receipt of the sale proceeds.

FRRP accepted policy in itself was not objectionable


The Panel accepted that this wording was similar to that used by many other companies and was not on the face of it objectionable. In reviewing the companys 1999 accounts the Panel noted that the turnover and profits recognised under this policy were not reflected in similar inflows of cash; indeed, operating cash flow was negative and the amount receivable within debtors of 46m represented more than the previous two years turnover of 44m. As a result, the Panel enquired into the detailed application of the policy.

FRRP enquired into detailed application of the policy where contracts not exchanged
This enquiry established that revenue in respect of the sale of Manston Park and the Northern Grass area of Manston Airport had been recognised at 31 March 1999 on the strength of nonbinding Heads of Agreement which were not turned into contracts until 29 July 1999. The enquiry also established that the sale contract for Fairfield had not been signed until 1 April 1999. In the Panels view the manner in which these transactions had been accounted for was not in accordance with the stated accounting policy and was unacceptable. The Panel also expressed its concern that the chairmans statement for 1999 compounded the error in the accounts by stating that during the year the most important achievements were the exchange of contracts with MEPC for the sale and development of Manston Park and London Manston Airport when the contract with MEPC for Manston had, in fact, not been exchanged during that year.

22 Pearson Education Limited 2006

Barry Elliott and Jamie Elliott: Financial Accounting and Reporting (tenth edition) Instructors Manual

Company response
The company accepted that the 1999 accounts were in error in respect of these transactions and decided to recognise turnover on exchange of contracts in accordance with its stated accounting policy which it did in the 2000 accounts by amending the 1999 comparatives.

FRRP enquired into detailed application of the policy where contracts were exchanged
However, the Panel had also questioned whether it would be correct to recognise revenue in respect of two of these transactions even after the contracts had been exchanged in the following year. The contract for the sale of Fairfield, concluded on 1 April 1999, was conditional upon the companys subsequent fulfilment of a material condition: namely, that planning permission had been obtained on terms satisfactory to the purchaser and without which the purchaser had certain rights not to proceed. Furthermore, the contract for the sale of the Northern Grass area of Manston appeared to have the characteristics of a financing deal rather than an outright sale and, under FRS 5, should not be recognised until the risks and rewards of ownership pass at a future date. The companys decision to recognise turnover and profit in respect of the conditional Fairfield contract and the Northern Grass transaction in its 2000 accounts was made without discussion with the Panel and in spite of the Panels reservations. The directors justified their recognition of revenue from the sale of Fairfield on the basis that they believed the planning permission was virtually certain to be obtained in due course. The Panel noted that the company had first recognised revenue in respect of Fairfield in its 1997 and 1998 accounts and had had to reverse those amounts in its original 1999 accounts when the contracts had lapsed. The contracts signed on 1 April 1999 replaced these original contracts. On the principle of reflecting sales of property that are subject to planning permission, the Panel had two main concerns, which are interrelated. The first is that, if the company has still to perform a significant amount of work in order to satisfy the condition, it has not yet completed the earning process sufficiently to entitle it to recognise the revenue at the balance sheet date. The second is that the outcome of a conditional contract is necessarily uncertain, and unless that uncertainty has been reduced to an acceptable level by the time that the accounts have been finalised, in general, the prudent course would be not to recognise the conditional contract until the condition is satisfied. The Panel accepts that there may be specific instances where it might be appropriate to recognise a sale even though it remains conditional; for example, where all the work required to satisfy the conditions has been performed and the relevant costs have been incurred before the year-end and the relevant conditions satisfied before the accounts are issued, it may be reasonable to recognise the sale. Those basic steps had not been effected in the case of Fairfield before the 2000 accounts were issued. The company has now accepted the Panel view. The amount of turnover recognised in the 2000 accounts in error and now to be corrected amounts to 37m.

Revisions required to the accounts turnover restated


As indicated in its interim statement, the company has also acknowledged that applying these principles, 2.34m turnover included in the 1996 accounts and subsequently reversed in 1997

23 Pearson Education Limited 2006

Barry Elliott and Jamie Elliott: Financial Accounting and Reporting (tenth edition) Instructors Manual

and 1998 should not have been recognised in the 1996 accounts. The accounts for 1996 to 1998 are being revised to correct these entries. A similar adjustment is being made by the company to its 1995 accounts.

Revisions required to the accounts compliance with FRS 5


As to the sale of Northern Grass the Panel considered that the substance of the transaction reflected a development being financed by the purchaser with the company paying an interest cost on monies received from the purchaser and retaining certain risks and rewards for the time being. Accordingly, in compliance with FRS 5, the transaction should have been treated as a financing arrangement until the interest payments ceased, either on completion of the development or on 30 January 2002, whichever is the earlier. The company has now accepted the Panel view. The amount of turnover recognised in the 2000 accounts in error and now to be corrected amounts to 5m. Debtors The Companies Act 1985 requires the amounts of debtors that are receivable after more than one year to be separately disclosed. In the 1999 accounts 10m of the anticipated proceeds from the sale of Manston Park was wrongly included in debtors due within 12 months. The company agreed to correct this error in the 2000 accounts but in the event recognition of the sale was reversed in its entirety.

24 Pearson Education Limited 2006

Barry Elliott and Jamie Elliott: Financial Accounting and Reporting (tenth edition) Instructors Manual

CHAPTER 3

Chapter 3: Question 1 Jim Bowater


(a) (b) Refer to Question 2 (a) below for description of underlying theory Jim Bowater ideal economic income model:

Investment of 36,000, cost of capital 20% (i) Jims economic income () for each of the three years is: 31 Dec 20X5 6,828 31 Dec 20X6 6,695 31 Dec 20X7 6,833 (ii) Jims economic capital will be preserved at the 1 January 20X5 level of 34,144 provided: he reinvests excess actual income of 672 on 31 December 20X5 and 34,107 at 31 December 20X7, generating a return of 20% an excess of actual income of 695 at 31 December 20X6 created, in effect, a cumulative injection of capital of (672 695) 23 to maintain his income of 20% p.a. will necessitate an investment of 34,167 from the proceeds of the proposed sale.

Workings (A) Ideal economic income (i.e. conditions of certainty)


Period 20X5 t0 t1 20X6 t1 t2 20X7 t2 t3 C 7,500 6,000 41,000 54,500 (a) t0 t1 7,500 + 1.2 (b) t1 t2 6,000 1.2
2

Kt 33,472 (b) 34,167 (c)

Kt1 34,144 (a) 33,472 (b) 34,167 (c)

Ye 6,828 6,695 6,833 20,356

C Ye 672 (695) 34,167 34,144

41,000 1.2
3

34,144

6,000 + 1.2

41,000 1.2
2

33,472

(c) t2 t3

41,000 = 1.2

34,167

25 Pearson Education Limited 2006

Barry Elliott and Jamie Elliott: Financial Accounting and Reporting (tenth edition) Instructors Manual

(B) Reinvestment under certainty to maintain 20% p.a. income


Economic income from: original investment t0 t1 t1 t2 t2 t3 6,828 6,695 (20% 672) 6,833 (20% 23) reinvestment total 133 (approx.) (5) (approx.) economic income 6,828 6,828 6,828

26 Pearson Education Limited 2006

Barry Elliott and Jamie Elliott: Financial Accounting and Reporting (tenth edition) Instructors Manual

Chapter 3: Question 2 Hickss Concept of Income; Spock


(a) Hicks economic model of income and capital

Hickss economic model of income and capital is based on his concept of welloffness. Welloffness is: the maximum income enjoyed by the individual without depleting that individuals capital stock. It is based on the precept of consumption which embraces the opportunity for consumption as well as actual consumption. As an extension of Fishers original model: it takes savings into account.

It is an ex ante model in that it usually measures expected income in advance of the time period concerned. Measurement of capital is necessitated in order to compute income. Income is the difference between opening and closing valuations of capital stock. The capital stock is computed by utilising the concept of present values. This concept adopts the idea of compound interest in order to compensate for the time element between cash flows.

Limitations In the field of accountancy there are serious practical limitations in measuring the accountants version of income and capital e.g. Subjectivity: the present value factor, often referred to as the discount cash flow element, is subjective. It requires the use of an interest rate and, as such, depending upon personal inclinations, it can utilise opportunity cost of capital, or the return on existing capital employed within a business entity, or contemporary short-term interest rates such as that charged on bank overdrafts, or the average rate pertaining in the current economic climate, or a speculative rate as assessed on the basis of perceived risk involved. Unrealised and realised flows: the model uses a mix of unrealised and realised cash flows. As a measure it is thus not of practical value in determining taxation liability and dividend policy. Financial strategy: attainment of flows as per a financial strategy is an integral part of the calculations. Targets are rarely achieved with precision. Variations from target destroy the models accuracy. Predictions are invariably unachievable with absolute accuracy. Windfalls: windfall flows cannot be foreseen and consequently cannot be incorporated within the model.

Balance sheet values: balance sheet valuations of net assets or capital employed concern aggregations of individually valued assets and itemised liabilities. It is not easy to apply the concept of present values across a range of individual assets and liabilities for balance sheet discount purposes.

27 Pearson Education Limited 2006

Barry Elliott and Jamie Elliott: Financial Accounting and Reporting (tenth edition) Instructors Manual

(b)
Year

Calculate Spocks ideal economic income using Hickss theorem


Economic capital value of the business at KO
Cash Flow DCF Factor 1/(1+r)
n

PV

K1 K2 K3

400 500 600 400 1,900

0.909 0.826 0.751 0.751

364 413 451 300 1,528

Economic value at X0 i.e. the beginning of the year is 1,528 [Note: initial capital was 1,000 therefore subjective goodwill is 528]

Economic capital value of the business at K1


Year Cash Flow K1 K2 K3 400 500 600 400 1,900 Economic value at K1 Rate of income return 1.000 0.909 0.826 0.826 DCF Factor 1/(1+r)
n

PV 400 455 496 330 1,681

= 1,681. So Y for Y1 = 1,681 1,528 = 153 = 153 / 1,528 = 10%

Economic capital value of the business at K2


Year Cash Flow K1 K2 K3 400 500 600 400 1,900 Economic value at K2 = 1,849. So Y for Y2 Rate of income return = 168/1,681 = 10% 1.100 1.100 0.909 0.909 DCF Factor 1/(1+r)
n

PV 440 500 546 363 1,849

= 1,849 1,681 = 168

28 Pearson Education Limited 2006

Barry Elliott and Jamie Elliott: Financial Accounting and Reporting (tenth edition) Instructors Manual

Economic capital value of the business at K3


Year Cash Flow K1 K2 K3 400 500 600 400 1,900 Economic value at K3 = 2,034. So Y for Y3 Rate of income return = 185/1,849 = 10% Note rate of income return is a constant 10%. 1.121 1.100 1.000 1.000 DCF Factor 1/(1+r)
n

PV 484 550 600 400 2,034

= 2,034 1,849 = 185

29 Pearson Education Limited 2006

Barry Elliott and Jamie Elliott: Financial Accounting and Reporting (tenth edition) Instructors Manual

Chapter 3: Question 3 Jason


(a) (i) Accounting income for Jason (using historical cost concept and applying conventional accounting by compiling an Income Statement account).
Income statement for the year ended 31 December,20X1
Opening inventory Gross profit c/d Depreciation Net profit 10,000 41,700 51,700 5,000 36,700 41,700 41,700 Gross profit b/d Sales less purchases Closing inventory 36,200 15,500 51,700 41,700

(Note: The Sales less purchases figure of 36,200 is derived from the debtors/creditors account below.)

So accounting profit = 36,700 (assumes traditional concept of going concern). Workings are shown below in T account form Cash Book
1 Jan 20X1 Balance 135,000 135,000 20X1 Debtors/creditors 40,000 40,000 1 Jan 20X2 Bal b/d 15,000 31 Dec 20X1 Drawings Balance 1.1.20X1 Purchase of business Legal costs 130,000 5,000 135,000 25,000 15,000 40,000

Purchase of business
1 Jan 20X1 Cash 130,000 1 Jan 20X1 Premises Stock Debtors ______ 130,000 Goodwill 100,000 10,000 4,000 16,000 130,000

Shop premises
1.1.20X1 Purchase of business 100,000 31 Dec 20X1 Balance 105,000

30 Pearson Education Limited 2006

Barry Elliott and Jamie Elliott: Financial Accounting and Reporting (tenth edition) Instructors Manual

Cash Legal costs capitalised 5,000 105,000 105,000

Depreciation
31 Dec 20X1 Balance c/d 5,000 5,000 1 Jan 20X2 Balance b/d 31 Dec 20X1 P&L a/c 5,000 5,000 5,000

Inventory
1 Jan 20X1 Purchase of business 31 Dec 20X1 P&L a/c 10,000 15,500 31 Dec 20X1 P&L a/c 10,000

Goodwill
1 Jan 20X1 Purchase of business 16,000

Capital
31Dec 20X1 Drawings Balance c/d 25,000 146,700 171,700 1 Jan 20 2 Balance b/d 1 Jan 20X1 Cash 135,000 36,700 171,700 146,700 31 Dec 20X1 P&L a/c

Trade receivables/trade payables account [prepared in order to derive net sales less purchases]
1 Jan 20X1 Purchase of business: receivables 20X1 Sales 31Dec 20X1 1 Jan 20X2 purchases 36,200 5,000 45,200 Receivables b/d 5,200 1 Jan 20 2 Payables b/d 31 Dec 20X1 Receivables c/d 5,200 45,200 5,000 Payables c/d = balancing figure 4,000 31 Dec 20X1 Cash balance 40,000

Drawings
31 Dec 20X1 Cash 25,000 31 Dec 20X1 Capital a/c 25,000

31 Pearson Education Limited 2006

Barry Elliott and Jamie Elliott: Financial Accounting and Reporting (tenth edition) Instructors Manual

Balance sheets
(NA0) At 1 Jan 20X1 Premises Add capitalisation of legal costs Less depreciation 105,000 Goodwill Inventory Trade receivables Cash Less drawings Trade payables 135,000 16,000 10,000 4,000 40,000 25,000 15,000 (5,000) 146,700 5,000 105,000 5,000 105,000 5,000 100,000 16,000 15,500 5,200 100,000 (NA1) At 31 Dec 20X1 100,000

So profit

= NA1 NA0 + Drawings = 146,700 135,000 + 25,000 = 36,700 as confirmed by the profit and loss account above.

Comparing the opening and closing balance sheets and allowing for drawings will enable profit to be derived but it is usual for accounting profit to be shown via a profit and loss account. It has been assumed that the traditional historical cost concept applies. It was intended that the legal costs be capitalised giving a fair value at 1 January 20X1 of 105,000 Thus depreciation is 5,000 (105,000 100,000) Alternatively the 5,000 could have been treated as an expensed cost (i.e. written off in the profit and loss account) The net profit would remain as 36,700, the depreciation having been replaced by the legal costs.

It was assumed that opening balance sheet values represent fair values (i.e. cost) of the individual assets concerned.

(ii)

Realisable income
Y0-1 = Net RV1 Net RVo + drawings = 136,200 135,000 + 25,000 = 26,200

32 Pearson Education Limited 2006

Barry Elliott and Jamie Elliott: Financial Accounting and Reporting (tenth edition) Instructors Manual

Workings: Net realisable values


At 1 Jan 20X1 Premises Goodwill Inventory Trade receivables Cash Less drawings Trade payables Net realisable values 40,000 25,000 15,000 (5,000) 136,200 135,000 85,000 16,000 20,000 5,200 At 31 Dec 20X1 105,000 16,000 10,000 4,000

Assumptions: (a) The realisable values are not based on an enforced sale. (b) Goodwill would possess a realisable value equivalent to its original cost in an enforced sale. (c) The entity is capable of being sold as a business entity in order to realise goodwill. Note: Some commentators might dispute the validity of goodwill because the concept of realisable income contravenes the going concern concept. In this situation the realisable income would be 10,200.

(iii)

Economic income ex ante


Ye = C1 + (K1t Kt 1) = 25,000 + (142,361 139,467) = 27,894 income for 20X1

Assumptions: (a) The difference of 4,467 between the actual cost of opening capital of 135,000 and its present value of 139,467 is to be treated as subjective goodwill. (b) The anticipated drawings represent expected cash flows. (c) The discount factor does not vary over the timespan. (d) The cash flows predicted will materialise. (e) Only the original capital of a present value of 139,467 needs to be maintained. (f) All the price levels are constant.

33 Pearson Education Limited 2006

Barry Elliott and Jamie Elliott: Financial Accounting and Reporting (tenth edition) Instructors Manual

Workings: Kt 1 = = K1t = capital at 1 January 20X1 ex ante CF (1+r)n = 25,000 (1.2) + 25,000 (1.2)2 + 25,000 + 150,000 (1.2)3

139,467

= capital at 31 December 20X1 ex ante = CF 25,000 = (1.2) (1+r)n 142,361 + 25,000 + 150,000 (1.2)2

An extension of the tabulated workings might be helpful:


Year Y0Y1 X1 Y1Y2 X2 Y2Y3 X3 C
1

1 t

Kt 1 139,467 175,000 175,000

Ye 27,894 35,000 35,000 97,894

W 32,639

Ye+W 27,894 67,639 35,000 130,533

C (Ye+W) (2,894) (32,639) 175,000 139,467

25,000 35,000 210,000 270,000

142,361 175,000

Workings:
Y1Y2 20X2 Y2Y3 20X3 (2) 175,000 175,000 175,000 (1) (2)

(1)

35,000 + (1.2)

35,000 + 175,000 = 175,000 (1.2)2

(2)

35,000 + 175,000 = 175,000 (1.2)

(iv)

Economic income ex post

Ye = C + (Kn Kn 1) = 35,000 + (175,000 175,000) = 35,000

34 Pearson Education Limited 2006

Barry Elliott and Jamie Elliott: Financial Accounting and Reporting (tenth edition) Instructors Manual

Assumptions: (a) The difference of 40,000 between the actual cost of opening capital of 135,000 and its present value of 175,000 is to be treated as subjective goodwill. (b) The discount factor is not subject to change. (c) Price levels are constant. (d) Cash flows for years following 20X1 will be as predicted. (e) All flows occur at the year-end. Workings: W1 = CF = (1+r)n = CF W2 = (1+r)n = 175,000 = 175,000 35,000 35,000 + 175,000 + (1.2) (1.2)2 35,000 + (1.2) 35,000 + (1.2)2 35,000 + 175,000 (1.2)3

A tabular extension of the workings might be helpful:


Year Y0Y1 Y1Y2 Y2Y3 C
1

Kn

Kn 1

Ye 35,000 35,000 105,000

W Ye+W 35,000 35,000 35,000

C (Ye+W)

35,000 175,000 W2 175,000 W1 35,000 35,000 175,000 W3 175,000 35,000 175,000 280,000 _____ -175,000

______ ______

W3 =

35,000 + 175,000 1.2

= 175,000

(b) (i)

Evaluation of the income figures Accounting income (36,700)

As an indicator of performance Based on actual transactions in this respect it is objective. However, it also utilises subjective data (e.g. depreciation) which incorporates an element of estimation into the results for the year.

35 Pearson Education Limited 2006

Barry Elliott and Jamie Elliott: Financial Accounting and Reporting (tenth edition) Instructors Manual

If such subjective data are substantial as a proportion of total costs, then the resultant profit or loss would be of reduced reliability.

Being based on historical cost it can be misleading as an indicator of real profit in times of changing price levels. It ignores unrealised capital gains/losses in pursuit of the going concern concept. It could be said therefore that on the one hand the figure is incorrect; but on the other it is realistic because there is no intention to realise the net assets.

The balance sheet is not a valuation statement. Consequently, profit expressed as a return on capital employed may be incorrect as an indicator of performance.

As an aid in decision making It is historic and history may not be a guide to the future. Circumstances of trade, costs and setting prices may be subject to factors not encountered by the results to date. However, historical trends over years may be of considerable assistance.

It does not enable precise comparisons to be made with the return yield of other businesses as historical costs can mean differing values across trade and industry as inflation develops. In general, accounting income has a considerable degree of authenticity because of its objective nature it is traditional and it is understood it can be of assistance as an indicator of performance and as an aid in decision making if it is used as a base figure capable of amendment in the light of: subjective content a changing price economy and anticipated future commercial trends regarding costs and sales.

(ii)

Realisable income (26,200)

As an indicator of performance It avoids the subjective assessment of depreciation and in this sense its measured income can be said to be realistic, but it embraces unrealised capital gains and losses which can be said to be irrelevant when the intention to sell does not exist. If the going concern concept is paramount then as an indicator realisable-value-based profit is unrealistic. Realisable values are subjective.

As an aid to decision making It can be said to equate asset values with opportunity cost which is relevant when considering the going concern versus the cash realisation potential of disposal.

36 Pearson Education Limited 2006

Barry Elliott and Jamie Elliott: Financial Accounting and Reporting (tenth edition) Instructors Manual

(iii)

Economic income ex ante (27,894)

As indicator of performance Very subjective figure in terms of future cash flows in respect of amount, timing and discount factor. These effects can make it impractical to implement as a system. However, it can accommodate inflation by taking account of changing price levels when forecasting the future cash flows As an indicator it is predictive and thus windfall gains can occur in this system when anticipated cash flows are changed by new circumstances. It is a guide to prudent conduct as it represents maximum consumption for a defined period without eroding capital.

As an aid to decision making By attempting to value a business at different time points it takes account of a strict capital maintenance concept via a time value of money thus the possibility of profit distributions being excessive and consequently eroding the capital is restricted this is not so with historical cost.

The adoption of a discounting factor enables cash flows to be adjusted to take account of risk. Whilst these two qualities are perhaps too subjective in terms of valuing the entire business entity they can be of considerable assistance when considering investment in the individual asset where choice amongst alternatives or the option of buying or renting exists.

(iv)

Economic income ex post (35,000)

As indicator of performance Accuracy is dependent upon the validity of the forecasting cash flows as with the ex ante system. However, unlike that model adjustments can be made to past as well as future capital values. Thus as an indicator of performance it has the potential to better the ex ante concept.

By adjusting past as well as future cash flows due to windfall elements it tends to have characteristics akin to traditional accounting. The figure of 35,000 is close to the traditional accounting figure of 36,200.

As an aid to decision making Expectations can change over time thereby affecting income and capital calculations. Windfall gains and losses can influence the calculations and thus inhibit confidence in the reliability of the measure. Will the profit be 35,000 next year? However absolute accuracy is not pretended by the model; as with the ex ante measure its intention is to give guidance only.

37 Pearson Education Limited 2006

Barry Elliott and Jamie Elliott: Financial Accounting and Reporting (tenth edition) Instructors Manual

CHAPTER 4

Chapter 4: Question 1 Shower Ltd


20X3 HC (iv) Sales Inventory (8,000 units) (4,000 units) 4,000 9,000 13,000 C Inventory (2,000 units) Sundry expenses Depn 6,000/5 3,000 10,000 10,000 5,000 1,200 3,800 Monetary gains Loan (8,000 240) 8,000 (100) 11,200 240/120 240/100 240/120 (i) 20,000 240/120 240/100 9,600 27600 6,000 21,600 18,400 10,000 2,880 5,520 Realised holding gains FA 2,4001,200 Inventory 12,00010,000 2,000 1,200 HC 150/150 HC 200/100 HC 150/150 CPP/PLA P P (ii) 40,000 HC 150/100 HC 150/150 6,000 9,000 15,000 3,000 12,000 8,000 5,000 2,400 600 Price variation adjustments FA 6,0002,000 (4,000) Inventory 3,0005,100 2,100 10,000 10,000 5,000 0,000 5,000 RC (iii) 20,000 CoCoA (iv) 20,000

Purchase (6,000 units)

240/120 18,000

38 Pearson Education Limited 2006

Barry Elliott and Jamie Elliott: Financial Accounting and Reporting (tenth edition) Instructors Manual

Monetary Losses (6,000 240) 6,000 (120) (6,000)

Unrealised holding Gains FA 9,6004,800 Inventory 5,1003,000 2,100 10,700 4,800

Capital Maintenance

2,000 240 100 2,000 (2,800) 300

PLA Net Income

10,720

39 Pearson Education Limited 2006

Barry Elliott and Jamie Elliott: Financial Accounting and Reporting (tenth edition) Instructors Manual

Balance sheets as at 31 December 20X3


HC Share capital Retained earnings 2,000 3,800 240/100 PLA CPP/PLA 4,800 10,720 Realised holding Unrealised holding RCA 2,000 600 3,200 6,900 Capital maintenance reserve 2,000 240 100 2,000 Loan 8,000 13,800 Non-current asset Depn Inventory Cash 6,000 1,200 4,800 3,000 6,000 13,800 240/100 14,400 240/100 240/120 2,880 11,520 6,000 6,000 23,520 200/100 200/100 255/150 12,000 2,400 9,600 5,100 6,000 20,700 NRV NRV 2,000 5,100 6,000 13,100 8,000 23,520 8,000 20,700 8,000 13,100 2,800 CoCoA 2,000 300

40 Pearson Education Limited 2006

Barry Elliott and Jamie Elliott: Financial Accounting and Reporting (tenth edition) Instructors Manual

Chapter 4: Question 2 Toy plc


(a)
1. Cost of sales adjustment This is the amount by which the historic cost of goods sold and charged in the historical cost income statement falls short of the replacement cost of those goods as at the date of sale. The CCA model requires all costs, assets, revenues and liabilities to be reported at their current entry value. The COSA is therefore an additional charge to the income statement account intended to bring the historical cost of sales to their replacement cost equivalent. The COSA is regarded as a realised holding gain in the sense that it is a gain relating to realised assets (i.e. sold inventory) and one made during the asset holding period.

2. Additional depreciation Is an amount charged to the income statement to make charge for year equate to that related to the replacement cost.

3. MWCA This is the gain or loss from holding monetary working capital. If prices are rising MWCA will be a gain if net monetary liabilities are held and a charge if net monetary assets are held.

4. Gearing adjustment Is an amount that is based on the proportion of the above charges that accrues to ordinary shareholders because there are lenders who bear a proportion of the charges.

5. Accumulated current cost depreciation This is to provide sufficient retention of funds to allow for the replacement of the fixed asset.

6. Current cost reserve This is a revaluation reserve where all holding gains (realised as well as unrealised) are credited in order to avoid the distribution of such gains and therefore reserve enough funds to replace assets at their current replacement costs as they are consumed.

41 Pearson Education Limited 2006

Barry Elliott and Jamie Elliott: Financial Accounting and Reporting (tenth edition) Instructors Manual

The composition of the CCA reserve account is likely to include realised items as follows: debits: backlog depreciation gearing adjustment credits: COSA monetary working capital adjustment. closing inventory revaluation increase non-current asset revaluation increase.

and the following unrealised items:

(b)
It quantifies cost of sales and depreciation after allowing for changing price levels so that the trading results are free of inflationary elements and provide a clearer picture of management performance. Resources are maintained by eliminating the risk of disturbing profits out of real capital. Time series and inter firm comparisons are more indicative of management performance.

42 Pearson Education Limited 2006

Barry Elliott and Jamie Elliott: Financial Accounting and Reporting (tenth edition) Instructors Manual

Chapter 4: Question 3 Parkway plc


(a) Monetary working capital

Making and stating assumptions: COSA provides for the maintenance of inventory levels in times of inflation. There is a view that MWC is also an integral part of daily operating activities and should be treated similarly via a provision out of revenue, from any detrimental impact caused by rising price levels. However, a consensus does not exist. in that some commentators maintain that MWC is not a part of the operating capital and so should be ignored when considering the operating capital maintenance concept, apparently in the belief that investment in such items as debtors is not an essential ingredient of day-to-day operations.

Even where critics accept MWC is a part of operating activities, varying views exist as to which assets and liabilities should be included in the MWC calculation. Conflicting views are: MWC should embrace monetary assets only; or all monetary assets less all monetary liabilities should be taken into account; or only short-term monetary liabilities should be accepted into the calculation; that longterm monetary liabilities should be part of the gearing adjustment; or even short-term monetary liabilities should be ignored; or only monetary assets and liabilities that have been generated by operating activities should be involved and thus these should be segregated from other monetary assets and monetary liabilities.

Usual inclusions: In spite of ongoing contentious debate, there is general acceptance to include the following items as part of MWC: trade receivables, including prepayments, trade bills receivable and VAT recoverable on trade purchases trade payables, including accruals, trade bills payable and VAT payable on turnover; and any stock not subject to COSA.

Usual exclusions: receivables and trade payables arising from fixed assets sold, bought or under construction or those arising out of any other non-trading activities; any cash or bank balances; and

43 Pearson Education Limited 2006

Barry Elliott and Jamie Elliott: Financial Accounting and Reporting (tenth edition) Instructors Manual

certain investments such as long-term and short-term investments. The former will be treated as fixed assets and the latter as cash and bank balances. If cash is essential to support day-to-day ordinary operations (e.g. a retail supermarket), then such cash is part of the MWC. Similarly if part of a bank balance or overdraft is subject to temporary but material changes as a reaction to fluctuations in levels of stock, trade debtors, trade purchases or sales then it should be treated as MWC. Any surplus will become part of the gearing adjustment.

Some critics formulate a case for including all or a portion of liquid resources as part of MWC:

Taking account of the above scenarios the following MWCA calculation involves the assumptions stated below and corresponding reasons for making them i.e. that: MWC is part of day-to-day ordinary operating activities. Trade receivables are substantial at 60,000 this is almost 50% of the capital invested in fixed assets (126,000) they amount to 63% of inventories (after eliminating an average profit content in debtors of 16% of sales i.e. 118,000/738,000 100 based on the year-end debtors figure of 60,000 i.e. 84% of 60,000/80,000 100) = 63%. If COSA is considered necessary in respect of inventories of 80,000 then so too is MWC in respect of trade receivables, inclusive of profit, of 60,000.

Total inventories of 80,000 are all subject to COSA. Trade payables, being also substantial at 90,000, are deemed essential to the entitys daily operating activities. Trade payables amount to an average 37 days credit i.e. [((80,000 + 70,000)/2) / 738,000] 365 during 20X8. Short-term investments are not essential to MWC, i.e. they do not constitute a provision of finance for, say, imminent investment in trade receivables as part of a marketing strategy to stimulate sales by increasing terms of credit to customers. Cash and bank balances, and any part thereof, are essential to day-to-day ordinary activities. The rate of credit given and taken remains unchanged over the period. Inventories have been charged out on the basis of FIFO and the inventory price level index is appropriate to the MWCA. Inventory movements have been evenly spread throughout the year.

Calculation of MWCA
30.6.20X8 Trade receivables Trade payables 60,000 (90,000) (30,000) 1.X.20X7 40,000 (60,000) (20,000) (10,000) Change

44 Pearson Education Limited 2006

Barry Elliott and Jamie Elliott: Financial Accounting and Reporting (tenth edition) Instructors Manual

Adjustment to average price levels:


30,000 160 180 = 26,667 Reduction in MWC 20,000 160 140 22,857 = Volume change (3,810) 6,190

(b)

Critical evaluation of the influence of MWCA


The concept of a MWCA acknowledges the existence of the interaction of physical assets and monetary assets by allowing for the protection of MWC against erosion by inflation in the same way that COSA protects capital in stocks consumed. The provision for additional MWC supplements the provision for extra depreciation and COSA in maintaining the capital substance of the entity. The calculation is not over-prudent as it takes cognisance of the protection granted by credit suppliers in their indirect funding of credit customers. The inclusion of monetary assets and trade payables within the inflation protection framework reduces the risk of an excess dividend being paid. This could threaten the going concern by overlooking the impact of inflation on the monetary working funds. The concept recognises the lag between realising a sale and realising the resultant cash. Changes in credit periods between that granted to trade receivables and that given by suppliers can be affected by inflation in that impact may otherwise remain hidden if the MWCA were not applied. However, a point of criticism is that if trade creditors become unstable in terms of credit given and credit received, the MWCA calculation increases in complexity and may not be so readily understood by the users of the accounts. A further criticism lies in the determination of any cash floats and bank balance movements deemed to be part of MWC by some business entities. These may be very subjective and, consequently, inaccurate or prone to abuse by the compilers.

45 Pearson Education Limited 2006

Barry Elliott and Jamie Elliott: Financial Accounting and Reporting (tenth edition) Instructors Manual

Chapter 4: Question 4 Raiders plc


(a)
(i)

All in 000s
Cost of goods sold

Note: Closing inventory purchased on average on 31 December. Average index is index at 30.9.X4 i.e. 150 Alternatively calculate the average of indices at 1.4.X4 and 31.3.X5 (138 + 162 ) ( 2 ) = 150
HC Revaluation ratio Inventory 1.4.X4 Purchases Inventory 31.3.X5 COGS 9,600 39,200 48,800 11,300 37,500 150/156 150/133 150/150 = Current cost 10,827 39,200 50,027 10,865 39,162

(ii)

Inventory figure in balance sheet


Balance sheet value 11,300 162/156 11,735

(iii)

Equipment depreciation charge


HC Revaluation ratio Purchased 1.4.X2 16,000 1.4.X3 20,000 1.4.X4 21,600 180/145 180/162 180/180 = Current cost 1.4.X4 31.3.X5

19,862 22,222 21,600 63,684 200/180 70,760

CC depreciation =15% [63,684 + 70,760] [ 2 ]

10,083

Alternatively calculate as: 15% 70,760

= 10,614

46 Pearson Education Limited 2006

Barry Elliott and Jamie Elliott: Financial Accounting and Reporting (tenth edition) Instructors Manual

(iv)

Balance sheet value of equipment


Purchase date 1.4.X2 1.4.X3 1.4.X4 Current cost 200 @ 1.4.X4 19,862 22,222 21,600 180 22,069 24,691 24,000 70,760 Net balance sheet value = 70,760 20,938 = 49,822 Gross CC Accumulated depreciation (45%) (30%) (15%) 9,931 7,407 3,600 20,938

(b)

Evaluation of incremental informational content


Discuss users and their decisions and how current cost number should improve predictions and control. Should also refer to recent empirical evidence and may discuss ongoing controversy within (and outside) ASB.

(c)

Consider power of providers; cost; economic companies, etc.

47 Pearson Education Limited 2006

Barry Elliott and Jamie Elliott: Financial Accounting and Reporting (tenth edition) Instructors Manual

Chapter 4: Question 5 Smith plc


(i) CPP requires the restatement of the income statement and balance sheet in terms of purchasing power of money at the end of the accounting period in units of CPP. It is rather like translating the historic figures into another currency. CPP accounts are derived from the historic accounts by applying the general price index, and are issued as supplementary statements aimed at the shareholders. The intention is to ensure that shareholders capital is maintained in terms of general purchasing power and distributions would be restrained during a period of general inflation. CPP accounts are objective/factual because they are based on HC figures updated to year end values; as such can be audited.

They show gains and losses on monetary items not incorporated into basic CCA model.

(ii)

(a) Restate the income statement in CPP


CPP income statement for the year ended 31.12.20X8
HC000 Index 236/228 236/216 236/228 236/232 1,720 280 20 100 120 160 236/120 236/228 39 104 143 123 350 1,739 2,089 (285) 1,804 266 CPP000 2,070

Sales Cost of sales Inventory Purchases Closing inventory Gross profit Depreciation Admin. expenses Net profit 320 1,680 2,000 280

2,000

48 Pearson Education Limited 2006

Barry Elliott and Jamie Elliott: Financial Accounting and Reporting (tenth edition) Instructors Manual

(b)

Restate the closing balance sheet in CPP


Balance sheet as at 31.12.20X8
HC000 Index 236/120 236/232 CPP000 2,321 284 160 120 564 (140) 424 2,745 2,745

Non-current assets Land and buildings Net current assets Inventory Trade receivables Cash/bank Less Trade payables Net total assets Equity 280 160 120 560 (140) 420 1,600 1,600 1,180

(c)

Restate the opening balance sheet in CPP (as at 31.12.20X8 rate)


Balance sheet as at 31.12.20X7
HC000 Index 236/120 236/216 236/220 236/220 236/220 CPP000 2,360 350 86 43 479 (215) 264 2,624 2,624

Non-current assets Land and buildings (net) Net current assets Inventory Trade receivables Cash/bank Less Trade payables Net total assets Equity (balancing figure) 320 80 40 440 (200) 240 1,440 1,440 1,200

(d)

Calculation of monetary loss as at 31/12/20X8


Equity [balance] at 31.12.20X7 in CPP000 Equity [balance] at 31.12.20X8 in CPP000 Increase Profit per income statement in CPP000 Monetary loss
49 Pearson Education Limited 2006

2,624 2,745 121 123 2

Barry Elliott and Jamie Elliott: Financial Accounting and Reporting (tenth edition) Instructors Manual

(e)

Reconciliation of monetary loss as at 31.12.20X8


HC000 Net monetary liabilities at 31.12.20X7 Increase Net monetary assets at 31.12.20X8 Monetary loss [140 142] (80) 220 140 2 Index 236/220 236/228 CPP000 (86) 228 142

Net monetary liabilities are made up as follows: 31.12.20X7 000 Trade receivables Bank Trade payables 80 40 (200) 31.12.20X8 000 160 120 (140)

(80)

140

50 Pearson Education Limited 2006

Barry Elliott and Jamie Elliott: Financial Accounting and Reporting (tenth edition) Instructors Manual

Chapter 4: Question 6 Aspirations Ltd


Income statement for year ended 31 December 20X1, prepared on RCA basis Sales Purchases Less: Inventory at 31.12.X1 Add: Cost of sales adjustment Adjusted cost of sales Adjusted gross profit Expenses Depreciation Operating gain (W2) 95,750 33,000 128,750 236,083 (W1) 520,125 24,250 495,875 7,717 503,592 364,833 868,425

Balance sheet as at 31st December 20X1 RCA Basis


Non-current assets Freehold property Depreciation Office equipment Depreciation Current assets Inventories at replacement cost Trade receivables Cash Less: Current Liabilities Payable within 1 year Net current assets Less: Non-current liabilities Payable after 1 year 500,00 751,800 2,158,800 116,250 1,251,800 24,250 253,500 1,090,300 1,368,050 (W3) (W3) 975,000 9,750 965,250 465,000 23,250 441,750

51 Pearson Education Limited 2006

Barry Elliott and Jamie Elliott: Financial Accounting and Reporting (tenth edition) Instructors Manual

Issued share capital 1,500,000 ordinary shares at 1 each Holding gains Retained earnings (W4) 1,500,000 422,717 236,083 2,158,800

Workings for RCA Model


W1 Cost of sales adjustment (COSA)
HCA Initial stock 1.1.X1 Purchases Closing inventory 31.12.X1 34,375 485,750 520,125 24,250 495,875 130/ 150 Indexed 130/115 130/ 130 RCA 38,859 485,750 524,609 21,017 503,592 Difference 4,484 4,484 3,233 7,717

The calculation has utilised the device of averaging. The user of an average index assumes that inventory was consumed on average at a price applying midway through the financial period. The increase in the cost of sales due to upward-moving price levels is 7,717. Purchases have been acquired evenly throughout the year, apart from the initial inventory, therefore the historical cost also represents average current cost and thus will not require any amendment. The advantages of averaging are those of speed and convenience. W2 Depreciation Depreciation is being based on the year-end replacement cost.
HCA Depn Property Equipment 6,500 18,750 25,250 127/110 145/125 Indexed RCA Depn 7,505 21,750 29,255

As far as the balance sheet is concerned, however, the cumulative depreciation for one year based on year-end values would still have to be 33,000. The difference of 3,745 (33,000 29,255) would constitute backlog depreciation for the current year. This is an important aspect of the calculation if average price level movements are used to determine depreciation. At first sight many students find the concept of backlog depreciation for the current year as distinct from previous years more difficult to understand.

52 Pearson Education Limited 2006

Barry Elliott and Jamie Elliott: Financial Accounting and Reporting (tenth edition) Instructors Manual

W3 Revaluation of non-monetary items at balance sheet date


HCA Freehold property Office equipment Inventory 650,000 375,000 1,025,000 24,250 145/145 Indexed 165/110 155/125 RCA 975,000 465,000 1,440,000 24,250 Difference 325,000 90,000 415,000

The index of 145/145 used to convert the inventory to RCA is not strictly correct. Inventories of 24,250 were part of purchases for the year and as such were not bought on the last day of the year. However we have assumed that the inventory was bought in the closing days of the year and is tending towards the specific price level measured at 145. If the inventory had been bought much earlier and the amount was material then it would be necessary to ascertain the index at the date of purchase. W4 Holding gains:
On stocks consumed (W1) On stocks carried at the year-end On fixed assets (W3) 7,717 nil 415,000 422,717

Income statement for year ended 31.12 20X1, GPP basis


Sales Initial inventory Purchases Less: Inventory 31 December Cost of sales GPP gross Expenses Depreciation (W7) (W8) 105,016 31,796 136,812 GPP net profit before loss on monetary items Less: Loss on monetary items GPP net profit after loss on monetary items (W10) 263,859 142,003 121,856 (W6) (W6) (W6) 43,287 532,758 (24,250) (W6) 551,795 400,671 (W5) 952,466

53 Pearson Education Limited 2006

Barry Elliott and Jamie Elliott: Financial Accounting and Reporting (tenth edition) Instructors Manual

Balance sheet as at 31st December 20X1, GPP Basis


Non-current assets: Freehold property Office equipment (W9) (W9) Cost 818,518 472,222 1,290,740 Current assets: Inventories at GPP valuation (W6) Trade receivables Cash 24,250 253,500 1,090,300 1,368,050 Less: Current Liabilities Payable within 1 year Net current assets Less: Non-current liablities Payable after 1 year 500,000 751,800 2,010,744 Issued share capital 1,500,000 ordinary shares fully paid Retained earnings 1,888,888 121,856 2,010,744 116,250 1,251,800 Depn 8,185 23,611 31,796 810,333 448,611 1,258,944

Workings (W): General or current purchasing power model With the GPP model historic pounds must be converted into general purchasing power pounds as at the end of the financial year. Where sales are generated and costs incurred evenly throughout the year, we may convert the historic pounds by using an average general price index. However, where substantial outlays of cash are involved on a particular day, as in the case of fixed assets and initial acquisition of inventories it will be more precise to utilise the index applying at that date, if available.
HCA W5 Sales 868,425 Adjustment 170 155 W6 Initial inventory acquired 34,375 170 135 Purchases 485,750 520,125
54 Pearson Education Limited 2006

GPP/CPP 952,466

43,287

170 155

532,758 576,045

Barry Elliott and Jamie Elliott: Financial Accounting and Reporting (tenth edition) Instructors Manual

Closing inventory

(24,250)

170 170 = no change (24,250)

Cost of sales

495,875

551,795

Inventory assumed acquired on or close to December 31 W7 Expenses 95,750 170 155 W8 Depreciation 25,250 170 135 W9 Fixed assets HCA cost Freehold Equipment 650,000 375,000 1,025,000 HCA Depn 6,500 18,750 25,250 643,500 356,250 999,750 170/135 170/135 NBV Index CPP cost CPP 818,518 472,222 1,290,740 CPP Depn CPP 8,185 23,611 31,796 CPP 810,333 448,611 1,258,944 31,796 105,016

W10 Gain or loss on monetary items Change in trade receivables during year: Change in cash occurring during year: In hand at 31 December 20X1 Received 1 January 20X1 Less payments non-current assets inventory In hand at 1 January 20X1 Change (increase) during year Change in payables occurring during year Trade payables (increase) Other payables Loans (increase) Change in monetary assets occurring during year (116,250) (500,000) (616,250) 286,925 1,500,000 (1,025,000) (34,375) 440,625 649,675 1,090,300 253,500

55 Pearson Education Limited 2006

Barry Elliott and Jamie Elliott: Financial Accounting and Reporting (tenth edition) Instructors Manual

So: Loss on holding net monetary assets during years inflation 286,925 (170 155)/155 Add loss on holding cash during the year i.e. balance at 1 January was held for full year and excluded from the above calculation: 440,625 (170 135)/135 114,236 142,003 27,767 CPP:

Income statement for year ended 31.12.20X1 NRV basis


Sales Purchases Less: Inventory at 31.12.X1 Cost of sales Gross profit Expenses Depreciation Operating gain Holding gain (W12) (W11) 95,750 35,000 130,750 241,800 18,188 259,988 520,125 24,250 495,875 372,550 868,425

Balance sheet as at 31 December 20X1 NRV basis


Non-current assets Freehold property Equipment Current assets Inventories at NRV Trade receivables Cash (W12) 42,438 253,500 1,090,300 1,386,238 Less: Current Liabilities Payable within 1 year Net current assets 116,250 1,269,988 (W11) (W11) 640,000 350,000

56 Pearson Education Limited 2006

Barry Elliott and Jamie Elliott: Financial Accounting and Reporting (tenth edition) Instructors Manual

Less: Non-current liabilities Payable after 1 year 500,000 769,988 1,759,988 Issued share capital 1,500,000 ordinary shares at 1 each Retained earnings 1,500,000 259,988 1,759,988 W11 Reduction in value of non-current assets at 31 December 20X1 Freehold HCA Less: NRV at 31.12.X1 650,000 640,000 10,000 The reduction in value is treated as depreciation W12 Holding gain in inventory at 31 December 20X1 NRV = cost + profit content of 75% 24,250 + 75% of 24,250 Less: Cost 42,438 24,250 18,188 Equipment 375,000 350,000 25,000 Total 1,025,000 990,000 35,000

57 Pearson Education Limited 2006

Barry Elliott and Jamie Elliott: Financial Accounting and Reporting (tenth edition) Instructors Manual

CHAPTER 5

Chapter 5: Question 1 Membership of FRC, ASB and FRRP FRC members


The membership of the Council is designed to include wide and balanced representation at the most senior levels of preparers, auditors and users of accounts and others interested in them. The members are:

Chairman
Sir Bryan Nicholson Sir Bryan has been Chairman of the Financial Reporting Council since 2001; Pro-Chancellor and Chair of the Council, Open University since 1996; Chairman of Education Development International plc since 2004 and non-executive director of Equitas Holdings Ltd since 1996. Born in 1932, Sir Bryan joined Unilever as a management trainee following National Service as a Second Lieutenant in the Royal Army Service Corps (RASC) and graduated with Second Class Honours from Oriel College, Oxford, where he read Politics, Philosophy and Economics (PPE). Having progressed within sales and marketing management at Unilever he moved first to the Jeyes Group as Sales Manager and then to the Remington division of Sperry Rand in 1964 as Sales Director. In 1966 he became General Manager in Australia and returned to Europe in 1969 as Managing Director for the UK and France. He joined Rank Xerox in 1972 as Director, Operations, Rank Xerox (UK) Limited becoming Chairman in 1979. He also supervised the European Subsidiaries of Rank Xerox. In 1984 the Government invited him to become Chairman of the Manpower Services Commission (MSC) and he was knighted in 1987 for his work at the MSC. In October 1987 he became Chairman and Chief Executive of the Post Office for five years until the end of December 1992. He was Chairman of BUPA from 1992 to 2001, Chairman of Varity (Europe) Limited from 1992 to 1996 and Chairman of The Cookson Group Plc from 1998 to 2003. Sir Bryan was also Chairman of the Council for National Academic Awards (CNAA) from 1988 to 1991 and of the National Council for Vocational Qualifications (NCVQ) from 1990 until 1993. He was Chancellor of Sheffield Hallam University from 1992 to 2001. Sir Bryan was President of the Confederation of British Industry (CBI) from 1994 to 1996. He was a member of the National Economic Development Council (NEDC) from 1985 to 1992. He is a past President of the Oriel

58 Pearson Education Limited 2006

Barry Elliott and Jamie Elliott: Financial Accounting and Reporting (tenth edition) Instructors Manual

Society and in 1989 was elected an Honorary Fellow of Oriel in recognition of his services to the College.

Deputy Chair
Barbara Thomas Barbara Thomas received her BA from the University of Pennsylvania. She then went on to NYU Law School where she received a JD degree with honors. Thereafter she practised corporate and securities law in New York, becoming a partner of Kaye, Scholer, Fierman, Hays & Handler in 1978. In 1980 she was appointed youngest ever Commissioner of US Securities and Exchange Commission and an informal spokesman for its accounting division. In 1983 she moved to Hong Kong as the first woman main board director of a London merchant bank, Samuel Montagu & Co. Ltd. In 1993 she came to the UK as an executive director of News International plc. She subsequently led a buy-in of Scotia Haven Food Group and then of Whitworths Food Group. Currently she is Chairman of Private Equity Investor plc and Deputy Chairman of Friends Provident plc, as well as a nonexecutive director of Capital Radio plc and Quintain Estates and Development plc, among others. She is also a Trustee of the Royal Academy of Arts and of The Wallace Collection, and a member of the Governing Body of the School of Oriental and African Studies. In addition, she is Chairman of the Professional Standards Advisory Board of the Institute of Directors.

Directors
Sir John Egan David Illingworth President of the CBI Chairman of the Consultative Committee of Accountancy Bodies and President of the Institute of Chartered Accountants in England and Wales Investor Community Representative

Vacancy

Members (ex-officio)
Sir John Bourn KCB Richard Fleck Mike Fogden Mary Keegan Bill Knight Chairman, Professional Oversight Board for Accountancy Chairman, Auditing Practices Board Chairman, Accountancy Investigation and Discipline Board Chairman, Accounting Standards Board Chairman, Financial Reporting Review Panel

59 Pearson Education Limited 2006

Barry Elliott and Jamie Elliott: Financial Accounting and Reporting (tenth edition) Instructors Manual

Members
Charles Allen-Jones Mike Barnes Scott Bell CBE Sir Victor Blank Sir John Bond Martin Broughton Sir David Clementi Don Cruickshank Michael Foot CBE Stephen Haddrill Sir Derek Higgs Douglas Kerr Rory Murphy Paul Myners CBE Richard Pearson Colin Perry Ian Plaistowe Sir Nigel Rudd Vincent Sheridan Sir Robert Smith Rosemary Thorne Graham Ward Formerly Senior Partner, Linklaters & Alliance Head of Technical Development, Audit Commission Formerly Group Managing Director, The Standard Life Assurance Company Chairman, GUS plc and Chairman of Trinity Mirror plc Group Chairman, HSBC Holdings plc Chairman, British American Tobacco plc Chairman, Prudential plc Formerly Chairman of the London Stock Exchange Managing Director, Deposit Takers & Markets Directorate, Financial Services Authority Director General, Fair Markets Group (Government nominee) Senior Adviser in the UK, UBS Warburg Group Finance Director, CPL Industries Ltd Joint General Secretary, UNIFI Chairman, Guardian Media Group plc Senior Partner, PKF Chairman, LTE Scientific Ltd Formerly Chairman, Auditing Practices Board Chairman, Boots Group plc and Pilkington plc Chief Executive, VHI Healthcare Chairman, The Weir Group plc Group Finance Director, Bradford & Bingley plc Senior Partner, Global Energy and Utilities, PricewaterhouseCoopers

Observers
Sir John Bourn KCB Peter Brierley Sir Andrew Likierman Comptroller & Auditor General, National Audit Office Head of Domestic Finance Division, Bank of England (Bank of England nominee) Managing Director, Financial Management, Reporting and Audit, HM Treasury, and Head of the Government Accountancy Service

Secretary
Ann Wilks CBE
60 Pearson Education Limited 2006

Barry Elliott and Jamie Elliott: Financial Accounting and Reporting (tenth edition) Instructors Manual

ASB members
Membership of the ASB is limited to a maximum of ten, of whom two (the chairman and the technical director) are full-time members and the remainder are part-time members. Appointments to the ASB are made by an Appointments Committee which comprises the FRC chairman and deputy chairman together with three members of council.

Chairman
Mary Keegan On 1 January 2001 Mary Keegan, then head of the Global Corporate Reporting Group of PricewaterhouseCoopers (PwC), succeeded Sir David Tweedie as full-time Chairman of the ASB. She joined Price Waterhouse (now PwC) in 1974, becoming, in 1985, the first woman admitted to partnership as an auditor in the UK firm. In 1991 she took charge of PWs UK technical function and in 1993 joined the group running the firms European audit practice. She formalised the firms support for the International Accounting Standards Committee (IASC). She served on the UITF 199399 and was a founder-member of IASCs Standing Interpretations Committee. She served on the Technical Committee of the Hundred Group of Finance Directors 19962000. From 1990 she actively contributed to the work of the ICAEW, including membership of its Council. From 1997 to 2000 she represented the UK accountancy bodies on the Council of the Fdration des Experts Comptables Europens (FEE) and was a vice president of FEE.

Technical Director
Andrew Lennard

Members
Michael Ashley Douglas Flint Huw Jones Roger Marshall Isobel Sharp John Smith Jonathan Symonds Partner, KPMG Group Finance Director, HSBC Holdings plc Director of Corporate Finance, M&G Investment Management Limited Partner, PricewaterhouseCoopers Partner, Deloitte & Touche Director of Finance, Property & Business Affairs, British Broadcasting Corporation Chief Financial Officer, AstraZeneca plc

61 Pearson Education Limited 2006

Barry Elliott and Jamie Elliott: Financial Accounting and Reporting (tenth edition) Instructors Manual

Observers
Allan Cook CBE Bernadette Kelly Sir Andrew Likierman Member of the European Financial Reporting Advisory Groups Technical Expert Group Director, Company Law and Investigations, Department of Trade and Industry Managing Director, Financial Management, Reporting and Audit, HM Treasury, and Head of the Government Accountancy Service Liaison member of the International Accounting Standards Board

Professor Geoffrey Whittington CBE

Secretary
Charles Bridge

FRRP members
Chairman
Bill Knight Bill Knight is a solicitor and a former Chairman of the Law Societys Company Law Committee. He was senior partner at Simmons & Simmons until 2001. He is currently Deputy Chairman of Council at Lloyd's of London and Chairman of the Enforcement Committee of the General Insurance Standards Council.

Deputy Chairman
Ian Brindle Ian Brindle BA Econ FCA retired from PricewaterhouseCoopers on 30 June 2001 having been appointed the Senior Partner of Price Waterhouse in 1991, and the Chairman of PricewaterhouseCoopers on the merger in 1998. Before joining the Accounting Standards Board in 1993 he served as a founder member of the Board's Urgent Issues Task Force. He was previously a member of the Auditing Practices Committee, becoming its Chairman in 1990. He was a member of the Council of the Institute of Chartered Accountants in England and Wales from 1994 to 1998.

Members
Rupert Beaumont Sir John Bourn KCB Stephen Box Formerly Partner, Slaughter and May Comptroller & Auditor General, National Audit Office Formerly Finance Director, The National Grid Group plc
62 Pearson Education Limited 2006

Barry Elliott and Jamie Elliott: Financial Accounting and Reporting (tenth edition) Instructors Manual

Michael Brindle QC Richard Delbridge Martin Eadon John Grieves Gordon Hamilton Andrew Higginson Robert Hildyard QC Nigel Macdonald David Mallett Ron Paterson Andrew Popham George Rose Rosemary Thome Tony Wedgwood

Barrister Formerly Group Chief Financial Officer, NatWest Group Partner, Deloitte Formerly Senior Partner, Freshfields Partner, Deloitte Finance Director, Tesco plc Barrister Formerly Partner, Ernst & Young Formerly Group Head of Finance, Standard Chartered Bank Formerly Partner, Ernst & Young Partner, PricewaterhouseCoopers Finance Director, BAE Systems plc Group Finance Director Bradford & Bingley plc Formerly Partner, KPMG

Secretary
Ann Wilks CBE

Director, Panel Operations


Carol Page

63 Pearson Education Limited 2006

Barry Elliott and Jamie Elliott: Financial Accounting and Reporting (tenth edition) Instructors Manual

CHAPTER 6

Chapter 6: Question 1 Financial Statements from Different Countries


Most companies will begin their accounting policies note with an explanation of the general accounting convention (e.g. US GAAP or IASs). The first point that should be considered is whether the overall convention is what might be expected given the origin and/or listing of the company. Secondly, although the companies are based in different countries, they may be using the same overall accounting convention. Students might comment that this is helpful as it allows greater comparability. The next stage of the analysis should involve a comparison of the accounting policies for dealing with specific accounting items. Things to look out for might include: Level of detail does one country give more than the other? Is this related to the level of regulation or is it down to the individual company? Which items are treated in the same way? Which items are treated differently? What is the effect of the differences? Is there a common thread in the differences? For example, might one set of regulations seem to be protecting a particular user group (e.g. creditors) or might there be some other underlying assumption (e.g. earnings should not be volatile)?

Chapter 6: Question 2 Web Exercise using EDGAR


The approach to this question should probably be similar to that for Question 1. However, this question allows students to measure the relative importance of the differences in policies. Students will identify the main areas of difference from the numerical reconciliations, but will need to refer to the narrative disclosures to understand the reasons. This exercise would work well with groups, asking different members of the group to look at particular countries. Tutors may wish to steer students towards particular countries to allow representation of different parts of the world. It is worth remembering that disclosures that satisfy the SECs requirements need not necessarily be easy for an outsider to follow. If students are studying the book in the order it is presented, then care should be taken that they do not get bogged down with the detail of different methods of accounting at this stage they will have an opportunity to address this later.

64 Pearson Education Limited 2006

Barry Elliott and Jamie Elliott: Financial Accounting and Reporting (tenth edition) Instructors Manual

Chapter 6: Question 3 Taxation and Financial Reporting


The chapter refers to the relationship between tax and financial reporting fairly briefly. A full answer to this question would probably require some further reading. Suggestions might include: Haller, A. (1992) The relationship of financial and tax accounting in Germany: a major reason for accounting disharmony in Europe, International Journal of Accounting, Vol. 27, pp. 31023. Hoogendoorn, M.N. (1996) Accounting and taxation in Europe a comparative overview, European Accounting Review, Vol. 5 (Supplement), pp. 78394. Lamb, M., Nobes, C.W. and Roberts, A.D. (1998) International variations in the connections between tax and financial reporting, Accounting and Business Research, Vol. 28 (3), pp. 17389. Nobes, C.W. (2003) A Conceptual Framework for the Taxable Income of Businesses and How to Apply It under IFRS, Certified Accountants Educational Trust (this can be downloaded from the ACCA website: www.accaglobal.com). Aisbitt, S. (2002) Tax and accounting rules: some recent developments, European Business Review, Vol. 14 (2), pp. 927. The question hinges on whether the tax and commercial accounts are (or will be) based on the same figures (congruence). Some advantages and disadvantages of congruence are set out below.
Congruence between tax and commercial accounts Interest group (a) Preparers Advantages Only one set of information required (b) Users Clear relationship between figures in accounts and tax charge no need for deferred tax (c) Tax authorities Clear information that has been fully audited Disadvantages Tax considerations might

drive commercial decisions Accounts do not necessarily reflect economic reality they are prepared so as to minimise tax liabilities May become involved in accounting issues rather than concentrating on macroeconomic policy issues

If there is (or will be) a high degree of congruence, then tax authorities need to know the extent to which income under IASs will differ from income under current regulations in order to estimate their expected share of that income. Further action may be required (e.g. in terms of grants or changes in tax rates) to achieve policy objectives. In the UK, the Inland Revenue is aiming for transition to IASs to be tax neutral.

65 Pearson Education Limited 2006

Barry Elliott and Jamie Elliott: Financial Accounting and Reporting (tenth edition) Instructors Manual

In the longer term international bodies (e.g. the EU) may impose the tax base, e.g. consolidated IAS financial statements. However, the financial reporting lobby (e.g. Nobes, 2003) would oppose that on the basis that 'tax pollution' of financial statements is undesirable due to the differing needs of the user groups (tax authorities versus investors).

66 Pearson Education Limited 2006

Barry Elliott and Jamie Elliott: Financial Accounting and Reporting (tenth edition) Instructors Manual

CHAPTER 7

Chapter 7: Question 1 MCRV Ltd


(a) Operations Statement for year ended 31.12.20X8
Turnover Opening stock at market price Purchases at cost Closing stock at market price Charge for goods sold Contribution Salaries Interest paid Wealth created by operations 30 9 39 97 50 80 130 66 64 136 200

(b)

Statement of changes in wealth


97 40 20 157

Increase in wealth due to operations Increase in value of fixed assets Decrease in value of long-term loans Realisable increase in net assets

Changes in wealth: The net assets have increased from 195 to 352. Fixed asset values reviewed at net realisable value 270 overall at the end of the year. Long-term loan value reviewed at net realisable value 70 at the end of the year due to rise in yields on long-dated stocks. Movements are set out in W1 (cash flow) and W2 (worksheet).

(c) The value of long-term loans is affected by changes in the prevailing interest rates. In the example the yields have increased. If they had decreased, the value of the loans would have been increased.

67 Pearson Education Limited 2006

Barry Elliott and Jamie Elliott: Financial Accounting and Reporting (tenth edition) Instructors Manual

W1: Cash flow statement Cash held at start of year Cash flow from operations Long-term loan finance raised Fixed assets acquired Cash held at end of year

10 *66 30 106 30 76

* MCRV does not prescribe the amount of detail which might be shown here. The conclusions of the feasibility study were that it would be appropriate to report inflows and outflows in the level of detail shown in the following cash book summary.

Notes on transactions during year


Cash book: Cash received during year credit customers long-term loans 190 30 220 Cash paid during year credit suppliers fixed assets salaries loan interest 85 30 30 9 154 Net increase in cash included in cash flow statement Journal: DR Debtors CR Sales DR Stock CR Creditors Credit purchases for year DR Cost of goods sold CR Stock Goods issued for sales DR Stock 6 6 CR Cost of goods sold Uplift in closing stock value from cost to net realisable value 70 70 80 80 66 200 200

68 Pearson Education Limited 2006

Barry Elliott and Jamie Elliott: Financial Accounting and Reporting (tenth edition) Instructors Manual

W2: Worksheet for information


Assets liabilities and at Cash transactions Journal Changes in wealth Operations Assets liabilities and at

start of year Dr Fixed assets Stock Debtors Cash Creditors Long-term loan Net assets Sales Cost of sales 30 Salaries Interest paid Operations 97 290 290 374 374 356 356 157 157 97 200 200 9 9 30 70 200 50 30 220 10 85 35 30 60 195 200 6 64 20 157 200 80 Cr Dr 30 190 154 86 200 70 Cr Dr Cr Inc 40 Dec Dr Cr

end of year Dr 270 66 40 76 30 70 352 Cr

452

452

(d) The ratio will differ to the extent that NRV differs from historical costs.

69 Pearson Education Limited 2006

Barry Elliott and Jamie Elliott: Financial Accounting and Reporting (tenth edition) Instructors Manual

Chapter 7: Question 2 Conceptual Framework


(a) Statements of Principles or Financial Reporting which set out the concepts that underlie the preparation and presentation of financial statements for external users have been widely developed. Their primary purpose is to provide a coherent frame of reference for standard setters to use in the development and review of accounting standards. In particular the framework provides a basis for choosing between alternative accounting treatments. (b) In practice the conceptual framework has provided standard setters with a framework for developing standards rather than providing a frame of reference for practitioners in resolving questions in the absence of a specific promulgated standard. Auditors are under pressure to accept practices which are commercially convenient to the client in the absence of a standard, e.g. selecting favourable revenue recognition criteria, adopting merger accounting where possible, massaging income in times of recession.

70 Pearson Education Limited 2006

Barry Elliott and Jamie Elliott: Financial Accounting and Reporting (tenth edition) Instructors Manual

Chapter 7: Question 3 Fairness


(a) There is an overriding requirement that financial statements should give a true and fair view of the financial position, performance and financial adaptability of an enterprise. In the UK, the ASB considers it to be a dynamic concept whose content will evolve in response to matters such as advances in accounting and changes in business practice. The Board considers that the evolution of the interpretation of the concept will be influenced over time by the accounting standards and other statements that the Board issues. It is an important concept in the UK because it allows companies to override statutory requirements. In such a case the company is required to include a note to the accounts giving particulars of any such departure, the reasons for it and its effect. The use of the override has been considered at various times by the Financial Reporting Review Panel, e.g. FRRP Press Notice 42 Sutton Harbour The Financial Reporting Review Panel considered the 1995 accounts of Sutton Harbour Holdings plc. The Panel accepted the directors justification for their departure from the provisions of Statement of Standard Accounting Practice (SSAP) 4 in the particular circumstances of the company. AIM Group The Financial Reporting Review Panel considered the Report and Accounts of AIM Group plc for the year ended 30 April 1998 and discussed them with the companys directors. The primary matter at issue was the departure from compliance with Financial Reporting Standard (FRS) 7 Fair Values in Acquisition Accounting and the use of the true and fair override following the companys acquisition of certain of the assets and business of Hunting plc.

(b) This raises the question of the feasibility of general purpose accounts to satisfy the information needs of non-equity shareholders. Discussion should embrace the interests of each of the user groups and consider the effectiveness of current measurement systems (HC/CPP/RCA/NRVA) and disclosure requirements e.g. socio-economic information, three bottom lines, environmental reporting.

71 Pearson Education Limited 2006

Barry Elliott and Jamie Elliott: Financial Accounting and Reporting (tenth edition) Instructors Manual

Chapter 7: Question 4 Control of Standard Setting


There is no unique answer to this question it may be approached in a number of ways, e.g. PRO arguments Technical requirements. These are now so complex as transactions have become more complex, e.g. financial instruments, that accountancy international professional firms are the only professional group with competence in many of the areas that will require standards in the future. Globalisation. National standard setters do not have the breadth that exists within the international firms. Accountability requirements. Standards will be set that are feasible based on current expertise and costs. Liability. Given that the main liability lies with the professional firms it is important that they are actively involved in formulation of standards. CON arguments Enforcement. The major requirement is for effective enforcement of existing standards which can only be achieved by making the international firms accountable. Lack of independence. The firms are too closely allied with the client and are therefore inclined to accept measurement and disclosure practices which do not comply with existing international standards. Investor confidence. This depends on transparency the existence of standards; their effective enforcement and adequate monitoring of audit performance. In practice, this requires the active participation of all parties: the companies preparing accounts, the accounting standard setters, the profession and user groups.

72 Pearson Education Limited 2006

Barry Elliott and Jamie Elliott: Financial Accounting and Reporting (tenth edition) Instructors Manual

CHAPTER 8

Chapter 8: Question 1 Old NV


(a)
Sales Less: returns Inventory at 1.1. 20X1 Purchases Carriage on purchases Less: returns Inventory at 31.12.20X1 Depreciation of plant Gross Profit Administration: Wages Administration expenses [28612] Directors remuneration Selling: Salesmens salaries Distribution: Distribution expenses Depreciation of vehicles Carriage Financial: Goodwill impairment Audit fee Debenture interest Rent receivable Tax Profit for year 177 38 25 (100) 2,929 2,795 562 2,233 290 187 125 800 738 274 375 6,263 13 (313) 5,963 6,788 1,125 5,663 313 5,976 5,724

Income Statement (internal) for the year ended 31 December 20X1 (000)
12,050 350 825 11,700

73 Pearson Education Limited 2006

Barry Elliott and Jamie Elliott: Financial Accounting and Reporting (tenth edition) Instructors Manual

(b)

Income statement for publication


Income Statement of Old NV for the year ended 31 December 20X1
000 000 11,700 5,976 5,724 1,402 1,602 (100) 2,904 2,820 25 2,795 562 2,233

Sales Cost of sales Gross profit Distribution costs W1 Administrative expenses W2 Other operating income Trading profit Interest payable Profit on ordinary activities before tax Income tax Profit on ordinary activities after tax W1 Salesmens salaries Distribution expenses Depreciation of vehicles Carriage W2 Wages Administrative expenses Directors remuneration Goodwill impairment Audit fee There will be a disclosure note as follows: Profit on ordinary activities after tax is after charging Goodwill impairment Audit fee Depreciation Directors remuneration 177 38 500 375 738 274 375 177 38 800 290 187 125

1,402

1,602

74 Pearson Education Limited 2006

Barry Elliott and Jamie Elliott: Financial Accounting and Reporting (tenth edition) Instructors Manual

Balance Sheet of Old NV as at 31 December 20X1


000 Non-current assets Intangible assets [1062 177] Property, plant and equipment Current assets Inventories Receivables Cash at bank and in hand Prepayments 1,125 3,875 1,750 12 6,762 Current liabilities Payables Provision for income tax Accrued charges Dividends proposed Net current assets Total assets less current liabilities Non-current liabilities Debentures Capital and reserves Ordinary shares of 1 each Preference shares of 1 each Share premium Retained earnings Note 2 3,125 625 250 2,746 6,746 250 6,746 738 562 63 362 1,725 5,037 6,996 Note 1 885 1,074 000

75 Pearson Education Limited 2006

Barry Elliott and Jamie Elliott: Financial Accounting and Reporting (tenth edition) Instructors Manual

Disclosure notes to show make-up of balance sheet items


Note 1: Property, plant and equipment
Motor Plant 000 Cost At 1.1.20X1 Additions Disposals At 31.12.20X1 Accumulated depreciation At 1.1.20X1 Charge for year At 31.12.20X1 Net book value At 31.12.20X1 At 31.12.20X0 Working: Accrued expenses Audit fee Debenture interest Note 2: Movements on reserves 000 Retained earnings at 1.1.20X1 Amount transferred from income statement Dividends proposed Balance at 31.12.20X1 875 2,233 (362) 2,746 511 824 000 38 25 563 750 1,074 1,574 738 313 1,051 375 187 562 1,113 500 1,613 1,562 1,125 2,687 1,200 362 1,125 2,325 vehicles 000 Total 000

Property, plant and equipment

76 Pearson Education Limited 2006

Barry Elliott and Jamie Elliott: Financial Accounting and Reporting (tenth edition) Instructors Manual

Chapter 8: Question 2 HK Ltd


(a) Income Statement for year ended 30 June 20X1
$000 Turnover Cost of sales Per trial balance + Hire 2,400 + depreciation 799 Insurance 150 + inventory loss 250 Gross profit Administration expenses Per trial balance 9,000 + Directors 562 + Bad Debt 157 + Auditor remuneration 112 Distribution costs Profit on disposal of fixed assets Profit before tax and interest Interest payable [454 + 151 tax on interest] Other operating income Profit before tax Income tax at 35% Tax on profit on disposal Profit for the year
Note: Depreciation consists of Buildings 94 + Plant 619 + Fixtures 86

$000 381,600

318,979 3,299 322,278 59,322

9,831 35,100 44,931 14,391 536 14,927 605 14,322 17 14,339 4,887 461 5,348 8,991

Balance sheet as at 30 June 20X1

(b)

Intangible non-current assets


480 2,880 4,680 648 3,096 1,857 864 259 605 1,239 4,032

Goodwill Tangible non-currrent assets Freehold land Freehold buildings Aggregate depreciation Plant and machinery Aggregate depreciation Fixtures and fittings Aggregate depreciation Current assets

77 Pearson Education Limited 2006

Barry Elliott and Jamie Elliott: Financial Accounting and Reporting (tenth edition) Instructors Manual

Inventory [11,794 500 obsolescence] Receivables [7,263 + 250 inventory sale + 150 insurance] Bank Current liabilities Payables Dividends Tax [4,887 + 461] Net current assets Non-current liabilities 9% loan Deferred Income Government grant (see Note) Capital Ordinary shares 50c each 9% preference shares of $1 each Reserve for increased cost of plant Retained earnings [6,364 + 780 Revaluation now realised 310 transfer to Increased Cost of Replacement Reserve + 8,991 profit for the year 1,074 dividends]

11,294 7,663 11,561 30,518 2,591 486 5,348 8,425 22,093 31,329 7,200 24,129 68 24,061 3,600 5,400 310

14,751 24,061

Note: The grant could be deducted from the cost of the plant under IAS 20.

(c)

The usefulness of the non-current asset schedule

(1) The column headings allow the user to see the type of non-current assets owned by the business. This can give helpful initial indications, for example: Realisability intangible assets might be more difficult to sell than property. Appreciation land is more likely to appreciate than office equipment. Depreciation licences are subject to amortisation and possible fall in value due to competion Security land and buildings are more likely to be accepted as security for loans and overdrafts than intangible assets.

78 Pearson Education Limited 2006

Barry Elliott and Jamie Elliott: Financial Accounting and Reporting (tenth edition) Instructors Manual

(2) The carrying values may be at cost or revaluation. If at cost it may be that the balance sheet gives too low an indication of current market values this is often an important consideration if existing shareholders are assessing a takeover offer. (3) The accumulated depreciation figure when related to the cost gives an indication of the age of the assets and possible need for capital outlays to replace with cash flow implications. (4) Disposals may be an indication that there is replacement occurring which could indicate growth or maintenance of existing capacity. If no replacement then consider implications for future capacity or other reason e.g. change of direction, disposal of non profit making parts of the business.

79 Pearson Education Limited 2006

Barry Elliott and Jamie Elliott: Financial Accounting and Reporting (tenth edition) Instructors Manual

Chapter 8: Question 3 Basalt plc


(a) Income statement for the year ended 31.12.20X0 (000)
935 Note 1 Note 2 Note 3 460 475 218 118 139 Other operating income (i.e. rent receivable) Profit on ordinary activities before tax Tax on profit of ordinary activities Profit on ordinary activities after tax Retained profits brought forward Proposed ordinary dividend Retained profits carried forward 000 Note 1: Opening inventory Purchases Carriage inwards Returns out Closing inventory 66 500 9 (25) (90) 460 Note 2: Warehouse wages Salesmens salaries Distribution expenses Hire of vehicles Depreciation 101 64 6 19 28 [7/11 of 20% of 220,000] 218 Note 3: Admin. wages Admin. expenses Directors remuneration Auditors remuneration Depreciation (4/11) 60 10 30 2 16 118 7 146 58 88 55 143 75 68

Turnover (962 27 returns) Cost of sales Gross profit Distribution costs Administrative expenses

80 Pearson Education Limited 2006

Barry Elliott and Jamie Elliott: Financial Accounting and Reporting (tenth edition) Instructors Manual

Balance Sheet as at 31 December 20X0 (000)


Non-current assets Tangible assets [cost 220 Depn b/f 49 Depn for year 44] Current assets Inventory Trade receivables Cash at bank Liabilities Amounts falling due within 1 year: Trade payables Other payables [Audit 2 + Corporation tax 58 + Dividends 75] Net current assets Total assets less current liabilities Capital and reserves Called-up share capital Share premium a/c General reserve Retained earnings 300 20 16 68 404 135 201 277 404 66 90 326 62 478 127

(b) (i) Directors report must deal with certain matters by law, e.g.
Proposed dividends Likely future developments in the companys business Principal activities of the company Political and charitable contributions Must be consistent with other statements reviewed by auditors May be highly personalised review of the business, its developments and the environment in which it operates Not subject to audit

(ii) Chairmans report

(iii) Auditors report expresses an opinion as to whether the financial statements give a true and fair view.

81 Pearson Education Limited 2006

Barry Elliott and Jamie Elliott: Financial Accounting and Reporting (tenth edition) Instructors Manual

Chapter 8: Question 4 Raffles Ltd


(a) Income Statement for the year ended 31 December 20X6
$ Revenue Cost of sales [W1] Gross profit Administration expenses [W2] Distribution expenses Operating profit Income from fixed asset investments Interest payable and similar charges [18,000 Deb + 3000] Exceptional costs Profit before tax on ordinary activities Tax on ordinary activities Profit after tax on ordinary activities [8] [7] [2] [5] [6] (21,000) (150,000) 260,750 115,750 145,000 1,628,000 1,098,400 529,600 (71,050) (32,800) 425,750 6,000

W1: Cost of sales


$ Per question Less: Realisable on obsolescent inventory Add: depreciation Less: capital item incorrectly posted 1,100,000 (5,600) 14,000 (10,000) 1,098,400

W2: Administration expenses


Per question Less: restructuring expense Less: provision written back Audit: fee Goodwill impairment Depreciation 206,300 (150,000) (1,500) 7,000 2,500 6,750 71,050

82 Pearson Education Limited 2006

Barry Elliott and Jamie Elliott: Financial Accounting and Reporting (tenth edition) Instructors Manual

Balance sheet as at 31 December 20X6


Non-current assets Intangible Property, plant and equipment Investments Current assets Inventory [156,360 + Inventory obsolescence realisable 5,600] Receivables [179,830 + 1500 Provision written back] Current liabilities Net current assets Total assets less current liabilities Non-current liabilities [5] [4] 161,950 181,330 343,280 (210,530) 132,750 644,000 (180,000) 464,000 Share capital Revaluation reserve Retained earnings [b/f 98,000 + 91,000] 250,000 25,000 189,000 464,000 [1] [2] [3] 40,000 341,250 130,000 511,250

(b)
1

Notes to the income statement


Accounting policies General Depreciation Exceptional items Operating profit is stated after charging audit fee 7,000 Staff costs Purchasing Distribution Administration 6 3 1 10 45,000 22,500 7,500 75,000

2 3

Directors emoluments Chairman nil Highest paid director 19,800 Other directors earn salaries in the range of 15,000 20,000 Income from non-current asset investments 6,000

83 Pearson Education Limited 2006

Barry Elliott and Jamie Elliott: Financial Accounting and Reporting (tenth edition) Instructors Manual

Interest payable and similar charges Bank interest Debenture interest paid and payable Exceptional costs Restructuring Less: Tax relief Taxation on ordinary activities Tax on profits [165,000 45,000 from Note 7] Overprovision Dividends Interim at 3.6p Proposed at 7.2p

3,000 18,000 21,000 150,000 (45,000) 105,000 120,000 (4,250) 115,750 18,000 36,000 54,000

Notes to balance sheet Non-current assets


1 Intangible assets Goodwill Cost at 1 January 20X6 Impairment At 1 January 20X6 Charge for year included in Administration expenses At 31 December 20X6 Net book value at 31 December 20X6 Net book value at 1 January 20X6 2 Property, plant and equipment Land/ blgs Cost At 1.1.20X6 Additions Disposals Revaluation At 31.12.20X6 25,000 250,000 110,000 54,000 225,000 20,000 90,000 (24,000) 78,000 323,000 90,000 (24,000) 25,000 414,000 Plant/ machinery Fixtures Total 7,500 2,500 10,000 40,000 42,500 50,000

84 Pearson Education Limited 2006

Barry Elliott and Jamie Elliott: Financial Accounting and Reporting (tenth edition) Instructors Manual

Depreciation At 1.1.20X6 Charge Disposals At 31.12.20X6 NBV 31.12.20X6 NBV 1.1. 20X6 3 Investments Listed on recognised stock exchange Notes relating to Liabilities, Post balance sheet events and Capital 4 Current liabilities Bank overdraft Trade payables Taxation Dividends Accrual interest 12,700 32,830 120,000 36,000 9,000 210,530 5 Non-current liabilities 10% Debentures redeemable 2003 (i) (ii) (iii) Provisions for liabilities Lawsuit details. Post balance sheet events Details re investment in Diat Por. Share capital Details of authorized capital Reserves At 1.1.20X6 Movement during year Less dividends (Note 9) At 31.12.20X6 Revaluation 25,000 Income statement 98,000 145,000 (54,000) 189,000 180,000 130,000 12,000 238,000 220,000 27,000 83,000 4,000 9,000 3,000 16,000 11,000 37,500 6,750 (10,500) 33,750 20,250 40,500 62,500 20,750 (10,500) 72,750 341,250 260,500

85 Pearson Education Limited 2006

Barry Elliott and Jamie Elliott: Financial Accounting and Reporting (tenth edition) Instructors Manual

Comments Provision is an amount retained from profit provides for a loss or liability likely to be incurred but amount is uncertain is shown in balance sheet after creditors or deducted from assets e.g. bad debt provision

Reserve is a realised or unrealised gain which has not either legally or at the companys discretion been distributed as dividends, i.e. is retained in the business, e.g. retained profits, share premium, revaluation reserve is shown in the balance sheet after share capital

Liability is an obligation in the future requiring the transfer of assets, e.g. cash payment or provision of services to other entities entails a probable future sacrifice may also include the amount owed to the owners of the business is reported under current or long-term liabilities

Contingent liability is a condition that exists at the balance sheet date where the outcome will be determined by a future uncertain event appears as a note to the balance sheet

86 Pearson Education Limited 2006

Barry Elliott and Jamie Elliott: Financial Accounting and Reporting (tenth edition) Instructors Manual

Chapter 8: Question 5 Phoenix plc


(a) Income Statement for year ended 30 June 20X7
000 Revenue Cost of sales [4,165 +280 dep] Gross profit Distribution cost Administration expense [1,126 + 31 dep + 415] Operating loss Exceptional item: Gain on disposal of associate Dividend received Loss before taxation Taxation Loss after taxation 75 80 (66) (96) (162) 6,465 (4,445) 2,020 (669) (1,572) (221)

(b)

Balance sheet as at 30 June 20X7


4,159 365

Property, plant and equipment Investment Current assets Inventory Trade receivables Cash at bank Current liabilities Net current assets Share capital and reserves Share capital Share premium Revaluation reserve Retained earnings 1,468 947 175 (868)

1,722 6,246 4,500 500 1,170 76 6,246

87 Pearson Education Limited 2006

Barry Elliott and Jamie Elliott: Financial Accounting and Reporting (tenth edition) Instructors Manual

(c)

Statement of Movement of Property, plant and equipment


L&B P&M 1,800 160 2,250 1,960 620 F&F 620 Total 4,820 (150) 160 4,830 2,400 (150)

Balance b/f Disposal Revaluation reserve Balance c/f Accumulated depreciation Balance b/f Revaluation reserve P&L charge Balance c/f WDV at 30.6.20X7 Current assets Trade receivables Creditors Trade payables Taxation Dividend proposed

540 (540) 280 280 2,250 1,680

360 31 391 22

900 (540) 311 671 4,159

947

566 122 180 868

Balances in revaluation reserve and retained earnings are made up as follows: Revaluation reserve Balance b/f Plant and machinery revaluation Transfer on disposal Transfer additional depreciation Loss for year Dividends Balance c/f 1,170 600 700 (30) (100) 30 100 (162) (380) 76 Retained earnings 488

(d)

Statement of Recognised Gains and Losses


(162) 700 538

Loss after taxation Revaluation gain Total recognised gain

88 Pearson Education Limited 2006

Barry Elliott and Jamie Elliott: Financial Accounting and Reporting (tenth edition) Instructors Manual

CHAPTER 9

Chapter 9: Question 1 Springtime Ltd


(a) Income Statement for the year ended 31 March 20X4
Continuing operations Turnover Cost of sales Gross profit Distribution costs Administrative costs Closure costs Operating profit Income from fixed asset investment Taxation [3200 200 + 150] Profit for the year
Note: Tax: Income Tax 3,200 Overprovision 200 + Transfer to deferred tax account 150

Discontinued operations 5,000 4,000 1,000 (425) (15) 560 (350) 210

Total 35,000 23,000 12,000 (3,490) (1,240) 7,270 (350) 6,920 1,200 8,120 3,150 4,970

30,000 19,000 11,000 (3,065) (1,225) 6,710 6,710

Workings
Continuing Distribution costs Delivery costs Depn vans Depn stores equip. Storeroom costs Delivery staff Directors Storeroom staff 900 40 50 1,000 700 75 300 3,065 25 100 425 300 1,200 40 50 1,000 700 100 400 3,490 Discontinued Total

89 Pearson Education Limited 2006

Barry Elliott and Jamie Elliott: Financial Accounting and Reporting (tenth edition) Instructors Manual

Admin. costs Audit Depreciation cars Office expenses Directors Office staff 30 10 800 300 85 1,225 15 15 30 10 800 300 100 1,240

Note: as allowed under IFRS 5, disclosures are given on the face of the income statement

(b) IFRS 5 has required companies to disclose in detail activities that are discontinued. This
disclosure is both numerical and narrative and provides a full explanation of the activities to be discontinued, when the discontinuance should occur and the financial effect of the discontinuance. This information is useful to users in enabling them to interpret the future performance of the enterprise and assessing the performance of management over the period. When considering the future performance of an enterprise only the continuing operations should be considered as it is only these that will continue into future periods. The management performance can be assessed to some extent by having knowledge of discontinuing activities because the users will be able to judge whether the management decision to discontinue is a good one. Users can also get benefits from the disclosure in understanding the future strategic direction of the business. By discontinuing activities the management may be refocusing the business towards more core areas and this would be seen through the disclosures.

(c) Reasons why accounting policy notes are important to users.


The underlying reason for a change might be more important than the change itself, e.g. a reduction in the life over which assets are depreciated could indicate a threat from technological changes within the sector; or a structural change, e.g. repositioning in the market. Inter-company comparison requires ideally that two companies should have similar accounting policies, or that their accounting policies should be amended to make them uniform, e.g. restating all non-current assets at current replacement costs. This is important when comparing ROCE, and a policy note about the use of modified historical accounts, together with reconciliations to historical cost profit required by some national standards (eg FRS 3 in the UK) might improve the interpretation of differences.

90 Pearson Education Limited 2006

Barry Elliott and Jamie Elliott: Financial Accounting and Reporting (tenth edition) Instructors Manual

Chapter 9: Question 2 Olive A/S


(a) Income Statement for year ended 30 September 20X4
3,460 W1 W2 W3 (1,557.1) 1,902.9 (362) (917.9) 623 6 45 (30) 644 Taxation (Sch 1) Profit for the year (197) 447

Revenue Cost of sales Gross profit Distribution cost Administration expenses Operating profit Exceptional items: Gain disposal of fixed assets Dividend received Interest and similar charges

(b)

Balance Sheet as at 30 September 20X4

Non-current assets Intangible assets Tangible assets (Sch 2) Investments Current assets Inventory [364 + 40] Receivables (Sch 3) Cash and bank Current liabilities (Sch 4) Net current assets Non-current liabilities 12% debentures Net capital employed Share capital: ordinary shares of 1 each Share premium account Retained earnings (Sch 5) Revaluation reserve 500 2,058 600 30 1,055 373 2,058 404 599 38 (636) 405 2,558 425 1,480 248

91 Pearson Education Limited 2006

Barry Elliott and Jamie Elliott: Financial Accounting and Reporting (tenth edition) Instructors Manual

Schedule 1: Taxation charge Income tax Underprovision 20X3 (140 128) 185 12 197 Schedule 2: Statement of Movement of Non-current Assets Land and Buildings Balance b/f Revaluation Acquisitions Disposal Balance c/f Balance b/f Revaluation Income charge Disposal Balance c/f WDV 30.9.X4 WDV 30.9.X3 Schedule 3: Receivables Trade receivables Prepaid rent 584 15 599 Schedule4: Current liabilities Trade payables Debenture interest (3 months) 20X3 Income tax 20X4 Income tax 296 15 140 185 636 600 300 900 80 (80) 15 15 885 520 Plant and Machinery 520 320 (240) 600 160 54 (66) 148 452 360 Fixtures Prepayand fittings ments 80 40 120 26 11 37 83 54 60 60 60 Total 1,200 300 420 (240) 1,680 266 (80) 80 (66) 200 1,480 934

92 Pearson Education Limited 2006

Barry Elliott and Jamie Elliott: Financial Accounting and Reporting (tenth edition) Instructors Manual

Schedule 5: Statement of Movement of Reserves Share premium A/c Balance b/f Formation expenses w/off Profit for the year Dividend paid Revaluation gain Transfer extra depreciation Balance c/f 150 (120) 30 Revaluation reserve 380 (7) 373 Income statement 661 447 (60) 7 1,055

Notes 1 Expenses charged in the year includes the following: Depreciation written off Directors emoluments Directors pension Audit fees and expenses 80,000 180,000 18,000 198,000 65,000

2 3 4

Company employs 646 persons, of whom 428 work at the factory and the rest at the head office. Land and Buildings were revalued during the year by Messrs XYZ, Chartered Valuers, at open market value on existing use basis and the surplus recorded in a Revaluation Reserve. Administration expenses includes an exceptional item of 60,000 being the underprovision for a claim that arose in a previous year.

Workings
W1 Cost of Sales Inventory on 1.10.20X3 Purchases Carriage inwards Depreciation Building Depreciation Machinery [18 + 28 + 8] Salaries [55% of 820] Pension cost [10% of 451] Heat and light [80% of 80] Inventory 30.9.20X4 211 925 162 9 54 451 45.1 64 (364) 1557.1

93 Pearson Education Limited 2006

Barry Elliott and Jamie Elliott: Financial Accounting and Reporting (tenth edition) Instructors Manual

W2 Distribution cost Advertising Sales commission Bad debts 112 92 158 362 W3 Administration expenses Depreciation Buildings Depreciation Fixtures and equipment [8 + 3] Underprovision for litigation Salaries Directors emoluments Pension costs [10% of 549] Heat and Light Audit fees and expenses Stationery Other administrative expenses 369 180 549 54.9 16 65 28 128 917.9 6 11 60

94 Pearson Education Limited 2006

Barry Elliott and Jamie Elliott: Financial Accounting and Reporting (tenth edition) Instructors Manual

Chapter 9: Question 3 Cryptic plc


(a)
Company is wise to depreciate buildings because IAS 16 requires the depreciation of all assets with finite life. This can be treated as a change in accounting policy; however, the guidance is not clear. If it is a change of policy, the material amount of backlog depreciation up to 30.6.X3 should be treated as a prior period adjustment, and explained in a note to published accounts.

(b)
The company is not merely permitted, it is encouraged to undertake a periodical review of its estimate of UEL of fixed assets. Necessary for avoiding situation where assets already fully written off continue in use and assist in earning income. The change in the estimate of UEL is not a change in accounting policy and hence the impact of the change would not qualify to be treated as prior period adjustment. The written-down value of the machinery on 30.6.20X4 of 288,000 should be written off over the remainder of its revised UEL (of two years) on the same policy (straight-line method). the current years charge will be 144,000 and this will be included in the figure of Cost of Sale. Since the current years charge of 144,000 exceeds the normal charge of 48,000 by a substantial amount, the difference may be reported in a note to accounts as an exceptional item.

(c)

Statement of Movement of Property, Plant and Equipment


Land and buildings 000 Plant and machinery 000 480 480 Furniture, tools etc. 000 380 (80) 300 Total 000 1,460 150 (80) 1,530

Balance b/f Acquisitions Disposals Balance c/f

600 150 750

95 Pearson Education Limited 2006

Barry Elliott and Jamie Elliott: Financial Accounting and Reporting (tenth edition) Instructors Manual

Acc. depreciation Balance b/f Prior period adj Income statement charge Disposal Balance c/f WDV on 30.6.20X4 WDV on 30.6.20X3 73 677 600 192 64 9 95 144 336 144 288 287 18 (9) 104 196 285 64 171 (9) 513 1,017 1,173

(d)

Income Statement for the year ended 30 June 20X5


000 000 2,285.0

Revenue

Cost of sales (Note 1) Gross profit


Distribution cost (Note 1) Administration expenses (Note 1) Operating Profit: Dividend income Finance cost Unusual items: Results of discontinued operations Fundamental reorganisation Disposal of fixed assets Taxation (Note 2) Profit for the year (192) (145) (7)

(1,466.2) 818.8
( 60.6) (281.2) 477.0 24.0 (24.0)

(344.0) 133.0 (33.0) 100.0

Balance Sheet as at 30 June 20X5


Non-current assets Property, plant and equipment (Ans c) Non-current asset investments 1,017 240

96 Pearson Education Limited 2006

Barry Elliott and Jamie Elliott: Financial Accounting and Reporting (tenth edition) Instructors Manual

Current assets Inventory (Sch 3) Receivables Cash and bank balance Current liabilities (Note 3) Non-current liabilities Deferred tax (Note 2) Preference shares @ 1 each Share capital Ordinary shares @ 50p each Share premium account Retained earnings Workings Cost of sales Inventory Raw materials Work-in-progress Finished goods Purchases Depreciation Machinery Buildings Furniture Salaries Rent Electricity Advertising Factory power Stationery Other administration expenses [468310] Audit fee Inventory Raw materials Work-in-progress (172) (54) 158 18 48 12 288 72 3.6 18 18 7.2 65 144 3.6 0.9 4.5 18.0 54 30 25.2 112 76 264 1,200 Distribution Administration 1,000 150 330 1,480 (174) (200) 1,480 590 475 29 (497) 597 1,854

97 Pearson Education Limited 2006

Barry Elliott and Jamie Elliott: Financial Accounting and Reporting (tenth edition) Instructors Manual

Finished Goods Less: discontinued operations

(364) 1,621.2 (155.0) 1466.2 109.1 (48.5) 60.6 319.7 (38.5) 281.2

Notes (1) Discontinued operations

The results of discontinued operations are made up from:


000 Revenue Cost of sales Distribution expenses Administrative expenses Costs of cancelling contracts 215 (155) (48.5) (38.5) (165) (192)

(2)

Taxation
65,000 (21,000) (11,000) 33,000

Income tax Overprovision for 20X4 taxation Deferred tax

Deferred tax Balance b/f Taxation for the year 185,000 (11,000) 174,000

(3)

Current liabilities
18 30 360 65 (charged as interest) 24 557

Audit fee Sales tax [(2,875 15/115) (1,380 15 /115) 165] Trade payables Tax Dividend

98 Pearson Education Limited 2006

Barry Elliott and Jamie Elliott: Financial Accounting and Reporting (tenth edition) Instructors Manual

Items to be disclosed

Expenses charged in the year Expenses charged in the year includes the following:
Depreciation Auditors remuneration 171,000 18,000

Unusual item: Depreciation charged on machinery includes an exceptional item of 96,000 arising from the revision of estimated useful life of machinery. Prior period adjustment The company has decided to depreciate buildings. The effect of this change in policy of 64,000 has been charged against Retained Earnings b/f.

99 Pearson Education Limited 2006

Barry Elliott and Jamie Elliott: Financial Accounting and Reporting (tenth edition) Instructors Manual

Chapter 9: Question 4 Reporting Financial Performance


(a) Statement of Changes in Equity

The Statement of Changes in Equity is of benefit to both investors and lenders to a company in a number of ways. The statement highlights the way that shareholders funds have changed over a period, and also the gains and losses recognised in the financial statements in the period that have not been charged or credited to the income statement. Investors will use the information in the statement to understand how the financial position of the company has changed. This will help the investors to understand whether the performance of the company has been good or poor. Investors will also see gains and losses that are not recognised in the income statement. For example the company could have been holding property that is increasing in value in the period, and this will be seen in the statement if the company has revalued in the period. Lenders will use the information in the statement to help assess the financial position of the company with a view to lending to it. The recognition of revaluation gains in the statement will help a lender to decide whether a company has a sufficient asset base to give security to loans. It can be argued that all the information that is in the Statement of Changes in Equity is already available in other disclosures in the financial statements. The statement, however, more clearly presents the information and therefore it is of benefit to users. Segmental disclosures Segmental disclosures provide information about the performance and position of an enterprise by reference to its business activities and geographical locations. The disclosures are quite extensive for primary segments and less extensive for secondary segments. The primary segmentation can either be by business activities or geographical location; it is the one that has the biggest impact on business risk. Investors want to make decisions on whether to buy, sell or hold shares in a company. Investors will need to understand how risky the investment is therefore as the return required will vary depending on the level of risk. Risk can either be financial risk or business risk, and segmental disclosures give information about the business risk that a company faces. If a company operates in a number of different business lines, for example, it will be exposed to less business risk than an enterprise that operates in only one. Investors can also use segmental disclosures to help to assess the quality of management. The investors will be able to see the performance of management in all of the entitys trading activities and therefore they will be able to see if the performance is poor in any area. Lenders will use the information in a similar way but to make decisions on whether to lend to the enterprise. The level of business risk to a lender may have an influence on the rate of interest that is set by the lender and the other terms of the loan.

100 Pearson Education Limited 2006

Barry Elliott and Jamie Elliott: Financial Accounting and Reporting (tenth edition) Instructors Manual

Discontinued activities IFRS 5 requires companies to present information about their continuing and discontinued activities separately. This information will often be presented on the face of the income statement although disclosure in the notes is acceptable. Users (investors, lenders and other users) like to see information about discontinued activities so that they can try to assess the future performance of the enterprise. When assessing the future performance of an enterprise the discontinuing activities should be ignored as these are not present in the future. If, for example, it can be seen that the discontinuing activities are the poorly performing activities of an enterprise, the enterprise might be a good investment even though overall its performance is not very good. Also the separate information about discontinued activities will help assess the performance of management. Users will be able to assess if the management is discontinuing unprofitable or profitable business activities. This will help the users decide whether the company is a good or poor investment decision. Disclosed information about future discontinuances that the company intends to undertake will also be useful to users in assessing the longer-term future potential. Companies must disclose information about discontinued activities as soon as the operations are classified as held for sale, and this could be before the actual discontinuance occurs.

(b) Bedok Ltd


20X9 financial statements Note to the accounts On 1 June 20X9 the board of directors announced a plan to dispose of the clothing manufacturing segment as this segment is not in line with the core activities of the company. The company is actively seeking a buyer and hopes to have completed the sale by the end of 20Y0. At 31 December 20X9 the carrying amount of the clothing segment assets was 20 million and the liabilities were 4 million. The clothing division had revenue of 65 million, incurred expenses of 50 million, had an operating profit of 14 million and a tax charge of 5 million on the profits for the year ended 31 December 20X9. A provision of 1 million has been made in respect of redundancy costs expected when the division is sold.

101 Pearson Education Limited 2006

Barry Elliott and Jamie Elliott: Financial Accounting and Reporting (tenth edition) Instructors Manual

20Y0 financial statements Income statement for 20Y0


Discontinued 20Y0 000 activities 20X9 000

Turnover Expenses Redundancy costs Provision for redundancy costs Operating profit Profit on disposal of division Taxation

40,000 (32,000) (1,000) 1,000 8,000 2,000 (3,000)

65,000 (50,000) (1,000) 14,000 (5,000)

Please note that 20X9 has been restated to present the results as discontinuing activities.

Note to the accounts On 10 May 20Y0 the board signed an agreement to sell the clothing division for 20 million. This plan had been announced on 1 June 20X9. The company decided to dispose of the division because its activities were inconsistent with the core activities of Bedok Ltd. Bedok Ltd recognised a provision of 1 million in 20X9 for the costs of redundancy of employees, and this provision was released in 20Y0. Actual redundancy costs of 1 million were paid. The process of selling the company was completed on 1 July 20Y0 and the assets of the division at this date were 23 million and liabilities were 5 million. A pre-tax profit of 2 million was made on the disposal.

102 Pearson Education Limited 2006

Barry Elliott and Jamie Elliott: Financial Accounting and Reporting (tenth edition) Instructors Manual

Chapter 9: Question 5 Parnell Ltd


(a) Accounting treatment of items 15

Item 1 is an exceptional unusual item General rule is to include under format heading to which it relates In this case distribution costs No adjustment necessary to the income statement but disclose bad debt by way of note. Item 2, profit/loss on the sale or termination of an operation should be Shown separately on face of the income statement after operating profit and before interest Analysed under appropriate heading as continuing or discontinued.

Item 3 enables distinction to be made between continued and discontinued operations Improves the comparability of current year with previous and next year.

Item 4 would normally be considered a change of accounting policy and requires A depreciation charge of 6m for 2003 and A prior year adjustment of 12m in respect of 2001/2002 to be charged against retained profits brought forward.

Item 5 is an exceptional unusual item which should be charged As an administrative expense in respect of continuing operations No adjustment is required to the income statement but the restructuring costs must be disclosed by way of note.

(b)

Redraft of the income statement for 2003


Continuing m Discontinued m 100 30 70 00 70 10 44 80 Total m 563 310 253 45 94 114 10 124 45 79
103 Pearson Education Limited 2006

Sales Cost of sales Distribution costs Administration expense (W1) Operating profit Profit on disposal of asset Profit on ordinary activities before tax Taxation Retained profit

463 280 183 45 94 44

Barry Elliott and Jamie Elliott: Financial Accounting and Reporting (tenth edition) Instructors Manual

Reserves Retained earnings m At beginning of year Prior year adjustment Transfer from income statement At year-end 101 (12) 89 79 168

W1 78 per question + 10 being profit on sale of distribution division to be separately disclosed + 6 depreciation on offices.

104 Pearson Education Limited 2006

Barry Elliott and Jamie Elliott: Financial Accounting and Reporting (tenth edition) Instructors Manual

Chapter 9: Question 6 Related Party Scenarios


(a) IAS 24 para. 9 states that a related party is a party that can exercise significant influence over another party. Significant influence would normally be assumed if a party owns at least 20% of voting rights. Arthur is therefore presumed to be a related party. However, Arthur would not appear to be able to influence the financial and operating policies because of the disagreement. Further enquiry would be required to establish that this has actually been the effect of the disagreement. (b) Brenda appears to fall within the definition of key management personnel. Brenda is therefore presumed to be a related party. Supporting evidence: Key management is defined as a person in a senior position having authority or responsibility for directing or controlling the major activities and resources of the reporting entity. Brendas ability to take 30% of the turnover with her would be prima facie evidence of such authority. (c) Donald and Emma through their relationship with Carrie would be related parties of Z Ltd. Both would be presumed to be related parties to Z Ltd. However, Donald is an employee and there would be no requirement to disclose emoluments. Supporting evidence: Emma is not an employee and amounts paid to Emma by Z Ltd would be disclosed. The disclosure required by para.17 of IAS 24 includes: (i) description of relationship (ii) description of the transaction (iii) the amounts involved (iv) any additional information required to understand the transaction (v) amounts due at balance sheet date. (d) This requires consideration of the difference between common control and common influence.

105 Pearson Education Limited 2006

Barry Elliott and Jamie Elliott: Financial Accounting and Reporting (tenth edition) Instructors Manual

Control brings with it the ability to cause the controlled party to subordinate its separate interests, whereas the outcome of the exercise of influence is less certain.

Paragraph 11(a) of IAS 24 states that two entities are not related simply because they have a director in common. Further enquiry is required to consider whether one or both transacting parties, subject to control and influence from the same source or common influence, have subordinated their own separate interests in entering into that transaction.

106 Pearson Education Limited 2006

Barry Elliott and Jamie Elliott: Financial Accounting and Reporting (tenth edition) Instructors Manual

Chapter 9: Question 7 Maxpool plc


Sale of factory outlet: As Bay plc is an investor owning more than 20% of Ching Ltd it is a related party of Ching Ltd. Details of the transaction will have to be included in both sets of financial statements for the financial year ending 31 December 20X0. Disclosure is required of any elements of the transactions necessary for an understanding of the financial statements. In this case it means that, as the factory site was sold to a major investor, the financial statements should note that the price was determined by an independent surveyor. What is position re Maxpool? Although both Maxpool plc and Bay plc have an investment in Ching Ltd this does not by itself make these companies related parties. There would appear to be no related party relationship between them and therefore there will be no disclosure in Maxpool plcs financial statements. If Maxpool and Bay fall within IAS 24 para. 9(a) and there is control or influence over Maxpool Group plc they could be related parties. Evidence of influence would, for example, be presumed if Maxpool plc persuaded Bay to sell the factory at below market value. What is effect of changes in shareholdings in 20X1? Maxpool plc is a related party of Ching Ltd. Maxpool plc is presumed to be a related party of Bay plc as Maxpool plc has a holding of more than 20% as per IAS 24 para. 9(b). Bay plc is not necessarily a related party of Ching Ltd because there is no presumption that 10% of shareholders have the requisite level of influence. although Maxpool plc controls Ching and has influence over Bay plc, the relationship between Bay and Ching would not automatically justify their being treated as related parties of each other. However, one would have to check whether one party has subordinated its interests to the other. How is the sale of vehicles treated? As regards the disclosure of the purchase of the vehicles by Bay plc, as it appears that Bay and Ching are not related parties there is no disclosure required in the financial statements of either company. Although Bay plc is not a related party of Ching Ltd it is an associate of Maxpool plc and therefore a related party of Maxpool plc.
107 Pearson Education Limited 2006

Barry Elliott and Jamie Elliott: Financial Accounting and Reporting (tenth edition) Instructors Manual

Maxpool plc will have to disclose details of the transaction between a group member and Bay plc in the group financial statements.

108 Pearson Education Limited 2006

Barry Elliott and Jamie Elliott: Financial Accounting and Reporting (tenth edition) Instructors Manual

CHAPTER 10

Chapter 10: Question 1 Distribution Rules


(a) A distribution is every description of distribution of a companys assets to its members
whether in cash or otherwise with the exception of: an issue of bonus shares redemption of a companys own shares out of capital a reduction in share capital by reducing the outstanding liability on unpaid shares or by paying off paid-up share capital. a distribution of assets to shareholders on winding up.

(b) The rules for distribution are made to ensure that losses are properly made good before
any dividends are paid ensuring that the capital of the company is not eroded. Before 1980, case law had not made this position clear, thus statute was enacted to clarify the position.

(c)

Profits available for distribution are:


accumulated realised profits less accumulated realised losses in so far as they have not already been distributed or capitalised capital and revenue profits are taken together provided they are realised unrealised profits are not available for distribution.

The additional rule for a public limited company is that the accumulated realised and unrealised profits must exceed the unrealised and realised losses of a company before a distribution can be made. Net unrealised profits are not distributable.

(d) Other constraint on the companys ability to distribute may be liquidity i.e. availability of
cash to pay a dividend.

109 Pearson Education Limited 2006

Barry Elliott and Jamie Elliott: Financial Accounting and Reporting (tenth edition) Instructors Manual

(e)

(i) A and B as private companies


A 000 B 000 10 15

Profit and loss account is the only distribution profit (i) (ii) (iii) Add back depreciation charged on revalued amount allowed by companies specifically for distribution Add adjustment to take account of gain now realised on sale of property Take off contingent loss as it is probable. Should be accrued for and hence in accordance with general accounting principles be treated as a realised loss Distributable profits for a private limited company

90 5

95

(13) 12

(ii) A and B as public companies


A 000 The fixed asset investment is an unrealised loss take into account but offset against unrealised profits revaluation reserve therefore no adjustment to distributable profits 95 12 (10) 40 (10) 35 B 000

110 Pearson Education Limited 2006

Barry Elliott and Jamie Elliott: Financial Accounting and Reporting (tenth edition) Instructors Manual

Chapter 10: Question 2 V.R. Confident Ltd


(a) In a private company: accumulated realised profits less accumulated realised losses by the
balance sheet date. For a public company: as above but net unrealised losses also need to be deducted.

(b)

Maximum distributable profits


000 Profit after tax as per accounts Less: (c) Stock (e) Bad debts Tax saving at 33% Add: (b) Depreciation (a) Asset revaluation reserve Less: Profit and loss a/c brought forward Add: General reserve 90.0 60.0 150.0 49.5 (100.5) 213.5 60.0 90.0 363.5 (288.0) 75.5 160.0 235.5 18.0 217.5 Maximum available for distribution Less: Preference share dividend Maximum distribution for ordinary shareholders 000 314.0

Assumptions: Research and development unrealised Profit and loss a/c brought forward all realised General reserve all realised

No difference here if the company was a public company as no accumulated net unrealised losses.

111 Pearson Education Limited 2006

Barry Elliott and Jamie Elliott: Financial Accounting and Reporting (tenth edition) Instructors Manual

Chapter 10: Question 3 Alpha, Beta and Gamma


(a)(i) Private companies
For a private company the maximum distribution that can be made is accumulated realised profits of the company (to the extent that they have not previously been distributed or capitalised) less its accumulated realised losses (to the extent that they have not been written off in a reduction or reorganisation of capital). Such profits and losses may be revenue or capital in origin.

(ii)

Public companies
For a public company the same rules apply subject to the additional restriction that IF accumulated unrealised losses exceed accumulated unrealised profits THEN the net unrealised loss must be deducted from the amount otherwise available for distribution.

Again profits and losses may be revenue or capital in origin.

(b)(i) Alpha plc


Maximum amount available for distribution at 31 March 20X7:
Realised capital profit Realised revenue profit brought forward Realised revenue profit for year Depreciation relating to revaluation surplus treated as realised (2,200/50) Maximum distribution 44 2,844 1,600 800 400 2,800

(ii)

Beta plc
Maximum amount available for distribution at 31 March 20X7:
Realised revenue profit brought forward Realised revenue loss for year Net unrealised capital loss (revaluation deficit) Maximum distribution 1,000 (400) 600 (400) 200

112 Pearson Education Limited 2006

Barry Elliott and Jamie Elliott: Financial Accounting and Reporting (tenth edition) Instructors Manual

(iii)

Gamma Ltd
Maximum amount available for distribution at 31 March 20X7:
Realised revenue loss brought forward Realised revenue profit for loss (3,200) 400 (2,800)

No distribution possible accumulated realised losses exceed accumulated realised profits.

113 Pearson Education Limited 2006

Barry Elliott and Jamie Elliott: Financial Accounting and Reporting (tenth edition) Instructors Manual

CHAPTER 11

Chapter 11: Question 1 Distribution identifying unrealised and realised profits


(a) (i) Share premium is the amount by which the value at which a company issues its shares
exceeds their nominal value.

(ii) Section 130(1) of the Companies Act 1985 requires that


where a company issues shares at a value that exceeds their nominal value a sum equal to the difference between the issue value and the nominal value must be transferred to a share premium account. Three circumstances in which this is necessary are as follows: 1. Where the company issues its shares for a cash consideration and the total amount it expects to receive in cash exceeds the par value stated on the share certificate. 2. Where shares were forfeited and re-issued thereafter, and the total consideration received on both transactions exceeded the nominal value. 3. Where the company issues shares for a consideration other than cash (e.g. in relation to an acquisition of business or merger) and the fair value of the shares issued exceeds their fair value.

(iii) Company law seeks to protect any balance in the share premium account from abuse,
because: it is part of the capital paid up by shareholders and it forms the creditors buffer that, according to company law, ought to be protected, in the interest of those whose interests are at peril on account of the privilege of limited liability.

Company law seeks to protect the balance in share premium from abuse by specifically stating the purpose for which alone such a balance may be applied as follows: pay up fully paid bonus shares write off preliminary expenses write off any expenses of any issue of shares or debentures write off commission paid or discount allowed on any issue of shares or debentures provide for any premium payable on redemption of debentures and provide for any premium payable on redemption of its own shares under the circumstances specified in answer to question (b)(i) below.

114 Pearson Education Limited 2006

Barry Elliott and Jamie Elliott: Financial Accounting and Reporting (tenth edition) Instructors Manual

(b) (i) Depreciation


Depreciation on a fixed asset is a realised expense BUT where the fixed asset has been revalued the depreciation on the revalued portion may be treated as a realised profit which is available for distribution. Development Development expenditure capitalised in the balance sheet and subsequently charged to the profit and loss account is a realised loss in the year the expense appears in the profit and loss account NOT when the cost is incurred.

(ii)

(iii)

Associates The share of profits from an associated company to an investing company which is not included in consolidated financial statements is not considered to be realised profits; only the dividends received are considered realised.

(iv) Profit on disposal The profit on disposal of a fixed asset which has been revalued will be the difference between the proceeds and the valuation of the asset.
However, the excess of the valuation over net book value will not have been treated as realised profit in the year of valuation and so in the year of disposal the whole of the profit (proceeds less net book value) will be treated as realised. Provisions The fact that the provision appears in the balance sheet, and not as a note to the balance sheet concerning a contingent liability, clearly indicates that the directors expect to pay damages. This decision is probably based on legal advice and, in accordance with SSAP 2 Disclosure of Accounting Policies, has been provided for in the accounts. In these circumstances this would be regarded as a realised loss. Revaluation surplus

(v)

(vi)

Any surplus arising from a revaluation of an asset would be regarded as unrealised. It would only be realised when the asset was sold. This is in accordance with SSAP 2 prudence concept.

(vii) A provision for bad debts A provision would be realised since it is generally accepted accounting practice to make such provisions in accordance with the prudence concept in SSAP 2.

115 Pearson Education Limited 2006

Barry Elliott and Jamie Elliott: Financial Accounting and Reporting (tenth edition) Instructors Manual

Chapter 11: Question 2 Smith Family Ltd


To: From: Date: Subject: Directors of Smith Family Ltd The Auditor August 20X7 Report on the Proposed Buyback of Ordinary 1 shares held by Mr Otto Smith Senior

1. Introduction 1.1 Otto Smith Senior wishes to retire from the business and dispose of his holding of 200,000 ordinary 1 shares. These have been valued by independent valuers at 300,000 and the price is not disputed. 1.2 The purpose of this report is to explain: whether the company may make the purchase the procedure which must be followed the rules relating to private companies in respect of the purchase of shares from capital.

1.3 The report will also include illustrative figures and journal entries for the two financing options (internal resources and a combination of internal resources and a new share issue) and will explain the changes to the balance sheet. (a) 2. Power to purchase own shares The Companies Act 1985 permits the purchase whether or not the shares were issued as redeemable. Power to purchase must also be permitted by Smith Family Ltds Articles of Association.

(b) 3. Procedure to be followed Off market purchase will, therefore, require specific contract approval by special resolution in general meeting. Shares bought in from Otto Senior must be cancelled immediately and not reissued. Permanent capital (share capital and undistributable reserves, e.g. share premium) generally required to be preserved by Companies Act 1985. A transfer must normally be made from distributable profits to undistributable reserves (capital redemption reserve) equivalent to the difference between the proceeds of the new issue (if any) and the nominal value of the shares bought back.

The premium on redemption (100,000 in respect of Otto Seniors shares) must normally be met from distributable reserves (profit and loss) although some limited relief may apply if the buyback is partly or wholly financed by a new share issue and Otto Seniors shares were originally issued at a premium, as indeed they were.

(c) 4. Redemption from capital private companies Smith Family Ltd, as a private company, may be allowed to reduce its permanent capital (share capital and undistributable reserves) where undistributable reserves are insufficient to finance the buyback. The amount of permissible reduction is found by the formula:

116 Pearson Education Limited 2006

Barry Elliott and Jamie Elliott: Financial Accounting and Reporting (tenth edition) Instructors Manual

Redemption cost less sum of the distributable reserves and the proceeds of any new issue. This is called the permissible capital payment (PCP). Purchase must be permitted by the company and a special resolution is needed. Statutory declaration of solvency is required by directors that, immediately after the buyback and within 1 year of the buyback, the company will be able to pay its debts. The directors may become personally liable for the debts. Report from company auditors must be attached to the declaration stating their agreement. Dissenting shareholders who did not vote can apply to the courts to have the purchase from capital set aside.

(d)

Option 1: Purchase from internal resources


Balance sheet after buyback:

Before 000 400 850 1,250 Financed by: 1,000 100 150 1,250 Workings: Redemption cost Less: Distributable reserves Permissible capital payment (PCP) Ordinary 1 shares Share premium Capital redemption reserve Profit and loss Net assets Cash Other sundry assets

After 000 100 (400 300) 850 950 800 (1,000 200) 100 50 (200 NV150 PCP) 950 300,000 (150,000) 150,000 200,000 (150,000) 50,000 Dr 200,000 200,000 Cr

Transfer to capital redemption reserve (CRR) Nominal value of shares redeemed Less: PCP Transfer to CRR Journal entries: Ordinary 1 shares Purchase of ordinary 1 shares Write out of shares to be bought back

117 Pearson Education Limited 2006

Barry Elliott and Jamie Elliott: Financial Accounting and Reporting (tenth edition) Instructors Manual

Profit and loss account Purchase of ordinary 1 shares Premium payable on buyback Purchase of ordinary 1 shares Cash account (or bank) Payment to buy back shares Profit and loss account Capital redemption reserve

100,000 100,000

300,000 300,000

50,000 50,000

Transfer from distributable to undistributable reserves in accordance with the requirements of the Companies Act 1985. (Nominal value of shares redeemed 200 less permissible capital payment 150.)

Explanation: Distributable reserves (100,000) are lower than the redemption costs (300,000), therefore, as a private company, Smith Family Ltd will be permitted to reduce its permanent capital. Permissible capital payment (the amount of permitted reduction) is the redemption cost (300,000) less the distributable reserves (150,000), that is 150,000 and the permanent capital after the buyback has fallen by this amount from 1,100,000 to 950,000. The capital redemption reserve is created because the nominal value of the shares redeemed (200,000) that is, the amount by which the permanent capital of the company would fall after the buyback, is higher than the permissible capital repayment of 150,000. Cash falls by 300,000, the cost of redemption. The profit and loss balance (distributable reserves) is wiped out by the premium due on redemption and the statutory transfer to capital redemption reserve. Ordinary shares fall to 800,000 after the buyback.

118 Pearson Education Limited 2006

Barry Elliott and Jamie Elliott: Financial Accounting and Reporting (tenth edition) Instructors Manual

Option 2: Partly from internal resources and partly by a new issue of ordinary l share
Balance sheet after buyback:
Before 000 400 850 1,250 Financed by: 1,000 100 150 1,250 Workings: PCP Redemption cost Less: Distributable reserves Proceeds of new issue PCP PCP Transfer to capital redemption reserve: Nominal value of shares redeemed Less: PCP Proceeds of new issue (to ordinary shares and share premium) Transfer to capital redemption reserve 100 (150) 50 50 200 150 100 (250) 50 Ordinary 1 shares Share premium Capital redemption reserve Profit and loss 850 (1,000 200 + 50) 110 (100 + 50 40) 50 40 (150P 60 (Prem) 50 CRR) 1,050 000 000 300 Net assets Cash Other sundry assets After 000 200 (400 300 + 100) 850 1,050

Premium on redemption: treatment


Redemption costs partly financed by new issue and Otto Smith Seniors shares originally issued at a premium: Premium on issue 200,000 @ 20p Premium on buyback (300 200) Proceeds of new issue Balance on share premium after new issue (100 + 50) 60,000 off against profit and losses. 150,000 40,000 100,000 100,000

Write 40,000 of premium due on buyback off against share premium and write balance of

119 Pearson Education Limited 2006

Barry Elliott and Jamie Elliott: Financial Accounting and Reporting (tenth edition) Instructors Manual

Journal entries:
Dr Ordinary 1 shares Purchase of ordinary 1 shares Write out of shares to be bought back Profit and loss account Share premium Purchase of ordinary 1 shares Premium payable on buyback Purchase of ordinary 1 shares Cash account (or bank) Payment to buy back shares The following two entries may be combined: Cash account Application and allotment account Proceeds of new issue Application and allotment account Ordinary 1 shares Share premium Allotment at a premium of 1 Profit and loss account Capital redemption reserve 50,000 50,000 100,000 50,000 50,000 100,000 100,000 300,000 300,000 60,000 40,000 100,000 200,000 200,000 Cr

Statutory transfer from profit and loss to capital redemption reserve: nominal value of shares bought back (200,000) exceeds the combined total of the permissible capital payment (50,000) and the proceeds of the new issue (100,000). Explanation: Distributable reserves and proceeds of new issue are lower than the redemption costs therefore, as a private company, Smith Family Ltd will be permitted to reduce its permanent capital on the buyback. Permissible capital payment is 50,000 and the permanent capital would have fallen by this amount except that part of the premium due on buyback may be financed through share premium, and therefore permanent capital has fallen by 90,000, the combined effect of the PCP (50,000) and the write-off against share premium (40,000).

120 Pearson Education Limited 2006

Barry Elliott and Jamie Elliott: Financial Accounting and Reporting (tenth edition) Instructors Manual

Capital redemption reserve is created because the nominal value of the shares redeemed (200,000) exceeds the combined sum of the PCP (50,000) and the proceeds of the new issue (100,000). Cash falls by 200,000, the net difference between the redemption cost and the proceeds of the new issue. Share premium account shows a net increase of 10,000, being the difference between the premium on the new issue of 50,000 and the permitted amount of 40,000 of the premium due on the buyback which has been charged against the share premium. Profit and loss falls to 40,000, after charging 60,000 premium due on the buyback and transferring 50,000 to capital redemption reserve. Ordinary shares fall to 850,000, showing the net effect of the buyback (200,000) and the new issue (50,000).

Conclusions and recommendations Option 2, partly financed by new share issue, is the more favourable because: Company is still left with distributable reserves (40,000 profit and loss). Share premium, which is a restricted reserve in terms of utilisation, may be used to absorb some of the cost (40,000) of the premium due on the buyback.

121 Pearson Education Limited 2006

Barry Elliott and Jamie Elliott: Financial Accounting and Reporting (tenth edition) Instructors Manual

Chapter 11: Question 3 Telin plc


(i)
1/10 4/10 12/10 31/10 1/11

Cash and bank


Balance Debentures Ord. shares Share premium P&L a/c Balance b/d 5,450,000 2,340,000 6,000,000 600,000 275,000 14,665,000 6,185,000 10% debentures 4/10 Bank Deb. discount 1/11 Balance b/d 2,340,000 60,000 2,400,000 31/10 Balance c/d 6,185,000 14,665,000 28/10 Redemption of preference shares 8,480,000

31/10

Balance c/d

2,400,000

Discount on debentures 4/10 Debentures 60,000 6/10 Share premium 60,000

Share premium 6/10 29/10 31/10 Deb. discount Premium on redemption Balance c/d 160,000 4,380,000 4,600,000 1/11 Profit and loss 6/10 29/10 29/10 29/10 31/10 Research exp. Dividends on pref. shares Premium on redemption Capital redemption reserve Balance c/d 1,400,000 1,755,000 4,875,000 4,875,000 240,000 80,000 31/10 Cash (profit) 275,000 1,400,000 1/10 Balance 4,600,000 Balance b/d 4,600,000 4,380,000 12/10 Cash 600,000 60,000 1/10 Balance 4,000,000

122 Pearson Education Limited 2006

Barry Elliott and Jamie Elliott: Financial Accounting and Reporting (tenth edition) Instructors Manual

Product development costs 1/10 Balance 1,400,000 6/10 P&L a/c 1,400,000

Ordinary share capital 1/10 12/10 30/10 31/10 Balance c/f 18,900,000 1/11 Balance c/d Balance Bank (Bonus issue) CRR 12,000,000 6,000,000 900,000 18,900,000 18,900,000

12% preference share capital 29/10 Redemption of shares 8,000,000 1/10 Balance 8,000,000

Redemption of preference shares 29/10 Cash 8,480,000 29/10 Pref. shares Premium on red. P&L a/c 8,480,000 Premium on redemption 29/10 Redemption a/c 400,000 29/10 Share premium P&L a/c Capital redemption reserve 30/10 31/10 Ordinary share capital bonus issue Balance c/d 900,000 500,000 1,400,000 1/11 Bal. b/d 1,400,000 500,000 29/10 P&L a/c 1,400,000 160,000 240,000 8,000,000 400,000 80,000 8,480,000

(ii)

Balance sheet as at 31 October 20X5


18,900,000 500,000 4,380,000 1,755,000 Sundry assets Cash at bank 32,170,000 6,185,000

Ordinary share capital Capital redemption reserve Share premium Retained profits

123 Pearson Education Limited 2006

Barry Elliott and Jamie Elliott: Financial Accounting and Reporting (tenth edition) Instructors Manual

10% debentures Creditors

2,400,000 10,420,000 38,355,000 38,355,000

Note: Advantageous course of action for shareholders is not to reduce distributable profits unless there is no other course of action. Therefore, whenever legally possible, reduction has been made from share premium account. Bonus issue was made from capital redemption reserve, as this is restricted to bonus issues only whereas share premium can be used for some other purposes also.

(iii)

Under the Companies Act 1981:

(a) Premium on redemption of shares can be written off against share premium maximum allowed being premium received on the issue of shares, which are now being redeemed i.e. 2% of 8,000,000 = 160,000 to share premium. Balance must be written off against profits. (b) Transfer to capital redemption reserve is the amount by which the aggregate receipts from specific new issue exceeds the nominal value of shares redeemed. Nominal value of shares redeemed Less: total receipts from new issue To capital redemption reserve (from distributable profits) 8,000,000 6,600,000 1,400,000

124 Pearson Education Limited 2006

Barry Elliott and Jamie Elliott: Financial Accounting and Reporting (tenth edition) Instructors Manual

Chapter 11: Question 4 Alpha Ltd


(a) Capital reduction and reorganisation account
000 7% notes Ordinary shares reissue Profit and loss account Shares in sub. Plant 50 15 177 55 57 354 Ordinary shares Ordinary shares Preference shares Freehold property 000 75 15 250 14 ___ 354

(b)

Balance sheet as on 1 July 20X8


000 000

Fixed assets Tangible assets Freehold property Plant Investment Shares in subsidiary company Loans Current assets Inventory Trade receivables Bank Payables: Amounts falling due within one year Trade payables Net current assets Payables: Amounts falling due after one year 7% notes Total assets less liabilities Ordinary share capital 200 25 25 282 63 225 107 345 132 106 45 40 85 162 55 22 77

125 Pearson Education Limited 2006

Barry Elliott and Jamie Elliott: Financial Accounting and Reporting (tenth edition) Instructors Manual

Ordinary share capital 000 Capital reduction Capital reduction Balance c/f 75 15 25 115 Balance Bank OSC 7% notes 25 150 175 Balance b/f 107 7% notes Balance c/f 200 200 Balance b/f Bank Capital reduction 150 50 200 200 Balance b/f Shares in sub. Balance c/f 58 10 107 175 Balance b/f Bank Reissue 000 75 25 15 115 25

126 Pearson Education Limited 2006

Barry Elliott and Jamie Elliott: Financial Accounting and Reporting (tenth edition) Instructors Manual

Chapter 11: Question 5 Doxin plc


This question is essentially concerned with the issue and redemption of shares by a plc where there is a trading loss impacting on the cash liquidity position. Part (a) requires students to illustrate the effect on key balance sheet components. Part (b) requires a discussion and evaluation of the effects of applying the Companies Act 1985 capital maintenance rules in circumstances where shares are redeemed partly out of distributable profits.
Opening (i) (iia) (iib) 000 (iii) 000 (iv) 000 5 (300) 80 20 200 400 400 (15) (80) (5) (500) (v) Closing BS 000 000 000 Ord. shares Pref. shares Capital red. Reserve Share premium Reserves Creditors Debentures 80 (380) 705 400 400 800 300 200 000 000 1,005 -

1,700 Bank
Other assets Debenture discount 1,700

1,505 220 (315) 360


40

200
1,500

(500)

(35)
1,500 40 1,505

(i) (a) Premium on redemption out of profits or lowest of: premium received on issue of shares to be redeemed (75,000) balance of share premium account including premium on new issue (20,000) total proceeds of the new issue (220,000).

(ii) (b) Capital redemption reserve: excess of nominal value of shares redeemed over (300,000 220,000 = 80,000) total receipt from new issue

127 Pearson Education Limited 2006

Barry Elliott and Jamie Elliott: Financial Accounting and Reporting (tenth edition) Instructors Manual

Comments on Doxin plc (a) (i) The issue of 200,000 ordinary shares at a premium of 10p each increases the share capital, the share premium and cash balance; note that the issue must be made within specified time limits if it is to be effective in applying the capital maintenance rules which require a transfer to capital redemption reserve. (ii) On redemption of the preference shares it is necessary to calculate the extent to which the premium on redemption can be charged to the share premium account, and the transfer, if any, to the capital redemption reserve from distributable profits in this case from the general reserve 200,000. The full premium on redemption can be charged to the share premium account which was brought into existence by the replacement issue. The limitation imposed by % premium originally received on the shares does not apply. The preference shares (300,000) disappear from the balance sheets and the share premium account becomes 5,000 with the bank balance reduced by 315,000. The transfer from general reserve to CRR is always in excess of nominal value redeemed over the proceeds of other issue (made specifically for redemption). (iii) The issue of 7% debentures 400,000 valued at 90 results in a long-term liability of 400,000 and a net increase in the bank balance of 360,000 with discount on debentures 40,000. The Companies Act 1985 is silent on treatment of this item apart from the option to write it off against the share premium account. Write-off over the life of the debenture might be the appropriate treatment. (iv) The use of the share premium balance 5,000 to cover a bonus issue of ordinary shares is reflected by a transfer to the ordinary share capital account as permitted by the Companies Act 1985. (v) The trade loss 500,000 incurred in the year is recorded as impacting on the bank balance where it creates an overdraft of 35,000. (b) The interest of creditors is protected by the creation of the CRR 80,000 which is nondistributable and can only be used to issue bonus shares.

However, because of the use of SPA to cover premium on redemption 15,000 the original capital of 1,100,000 is only maintained up to 1,085,000 capital meaning issued share capital plus undistributable reserves. The effect of this loophole in capital maintenance regulation could be remedied by an additional transfer from distributable profit to CRR in this case of 15,000.

128 Pearson Education Limited 2006

Barry Elliott and Jamie Elliott: Financial Accounting and Reporting (tenth edition) Instructors Manual

Chapter 11: Question 6


(a) The advantages of purchasing and cancelling own shares are:

It is a method of returning surplus cash that a company is unable to invest profitably within the company It is a method of overcoming a problem as when shares are acquired from a dissenting shareholder so as to remove the nuisance value. It is a method of providing cash as a help to a shareholder in liquidating their shareholding as where shares have been issued to employees as part of a profit sharing scheme and the employee wishes to convert to cash or they are acquired from the estate of a deceased shareholder It is a possible method of improving the share price if the directors consider the current share price are undervalued on cancellation each remaining share has a greater interest in the net assets. It is taken as a means of increasing the earnings per share.

(b)

The advantages of purchasing and holding shares in treasury

It provides a company with greater flexibility in managing its share capital It allows a company to optimise its gearing by buy back rather than by increasing or decreasing its debt It reduces the cost of raising new capital if the shares are reissued later through a Broker rather than through a more expensive placing or rights issue. It can stimulate an inactive market particularly if existing shareholders have been finding it difficult to sell their shares It can lead to an increase in the earnings per share Treasury shares can be used to satisfy the exercise of employee share options and may be acquired at the date the option is granted and held in treasury.

129 Pearson Education Limited 2006

Barry Elliott and Jamie Elliott: Financial Accounting and Reporting (tenth edition) Instructors Manual

CHAPTER 12

Chapter 12: Question 1 Post Balance Sheet Events


(a) IAS 10 requires that financial statements should be prepared on the basis of conditions existing at the balance sheet date, and therefore lays down the following treatment for events after the balance sheet date. A material event after the balance sheet date requires changes in the amounts to be included in financial statements where: it is an adjusting event (i.e. an event which provides additional evidence of condition existing at the balance sheet date); or it indicates that application of the going concern concept to the whole or a material part of the company is not appropriate. A material event after the balance sheet date should be disclosed where: it is a non-adjusting event i.e. an event which concerns conditions which did not exist at the balance sheet date of such materiality that its non-disclosure would affect the ability of the users of financial statements to reach a proper understanding of the financial position; or it is the reversal or maturity after the year end of a transaction entered into before the year end the substance of which was primarily to alter the appearance of the companys balance sheet. The purpose of the recommended treatment is: To ensure that the financial statements show the true position as it existed at the year end. All information available to management at the date that the financial statements are finalised should be taken into account. To the extent that such information relates to the financial year dealt with in the financial statements, they should be adjusted. To the extent that material matters relate to the subsequent period the user should be informed by way of note so that they are not misled as to the current position of the company. However, If events subsequent to the year end indicate a need to consider whether the enterprise (or a material part of it) is a going concern they are to be treated as adjusting events because the use of the going concern basis implies a belief that the company will continue in business for the foreseeable future.

130 Pearson Education Limited 2006

Barry Elliott and Jamie Elliott: Financial Accounting and Reporting (tenth edition) Instructors Manual

(b) (i) (ii) As the agreement to purchase this business relates to the period after the year end it is a nonadjusting event. Knowledge of it in no way affects the position of the company at 31 December 20X6. Therefore it should be disclosed by way of note. (iii) (iv) This is an adjusting event because it provides evidence of conditions existing at the balance sheet date. The potential loss of 9,000 should be written off in the income statement and deducted from debtors in the balance sheet. The rights issue relates entirely to the period after the year end, therefore it is a nonadjusting event. It would be wrong to adjust the companys share capital and cash at 31 December 20X6 because of a subsequent share issue. This should be disclosed by way of note. This fraud is an adjusting event, because it has been discovered that the financial statements will not be correct unless they are adjusted for it. The 8,000 should therefore be written off in the profit and loss account so that a true and fair view is shown.

131 Pearson Education Limited 2006

Barry Elliott and Jamie Elliott: Financial Accounting and Reporting (tenth edition) Instructors Manual

Chapter 12: Question 2 SEAS Ltd


(i) The closure of the Garratt factory is a discontinuance of a business segment, as it was clearly a material and separately identifiable component of the companys business operations. Under IFRS 5, disclose under discontinuing heading. (ii) The fraud is an adjusting event under IAS 10, and, as it has been discovered, the financial statements will not show a true and fair view unless they are adjusted for it. The $30,000 should therefore be written off in the income statement. (iii) This is an exceptional item, because it is material but arises from the ordinary activities of the business. It should be charged in arriving at the profit on ordinary activities, and should be disclosed separately. (iv) The agreement to purchase this business relates to the period after the year end and is therefore a non-adjusting event under IAS 10, as knowledge of it does not affect the companys position at 31 March 20X8. However, disclose the information by way of note. (v) This is also an exceptional item and should be treated as in (iii) above. It is not a prior year adjustment because it arises from a change in trading conditions and not from a change in accounting policy or a fundamental error. (vi) Under IAS 10 this is an adjusting event as it provides evidence of conditions existing at the balance sheet date. The potential loss of 30,000 should be written off in the income statement and deducted from the debtors in the balance sheet. (vii)The rights issue relates entirely to the period after the year end, therefore it is a nonadjusting event under IAS 10. This should be disclosed by way of note.

132 Pearson Education Limited 2006

Barry Elliott and Jamie Elliott: Financial Accounting and Reporting (tenth edition) Instructors Manual

Chapter 12: Question 3 World Wide Nuclear Fuels


(a)
(i)

Explanation
Need for guidance

Difficulties included: Definition the IASB define provisions as a liability of uncertain timing or amount. Treatment of future operating losses considered these should be accounted for in the future. Provisions differ from liabilities in that provisions are often subject to disclosure requirements whereas other creditors are not e.g. statutory requirement to disclose may however be insufficient detail. Adequate level of disclosure of movements is important as these do not go through Income Statement once provision is established. Unacceptable practice of big bath provisioning used to absorb expenses incurred in later years. Management has been able to control the recognition and timing of movements so that user does not have a clear picture of current years performance smoothing profits. There has been inconsistency between the accounting for provisions between different companies. Recognition

(ii)

IAS 37 applies Framework approach provisions are an element of the liabilities and not a separate element of the financial statements. Provisions should therefore be recognised only when: (i) an enterprise has a present legal or constructive obligation and benefits as a result of past events (ii) it is probably that an outflow of resources embodying economic benefits will be required to settle the obligation (iii) a reasonable estimate of the amount required to settle the obligation can be made. IAS 37 takes a balance sheet perspective by concentrating on liability recognition rather than the recognition of an expense. Criteria include: An obligation exists when the entity has no realistic alternative to making a transfer of economic benefits may be legally enforceable or constructive. Only recognised if existing at balance sheet date. Must have arisen from past events. Must exist independently from the companys future actions.

133 Pearson Education Limited 2006

Barry Elliott and Jamie Elliott: Financial Accounting and Reporting (tenth edition) Instructors Manual

If avoidable by future actions then no provision is recognised. No provision should be recognised for future operating losses. A constructive obligation for restructuring only exists when the recognition criteria laid out in IAS 37 are satisfied. If an enterprise has a contract which is onerous, the present obligation should be recognised and measured as a provision.

(b)

Transactions

Although IAS 37 states that no provision should be made for future operation losses, this does not apply if there is an onerous contract. This contract appears to be onerous and so the provision of $135m should remain in the financial statements. With regard to the provisions for environmental liabilities, the question is whether this is a constructive obligation. There is no current obligation but it could be argued that there is a constructive obligation to provide for the remedial work because the conduct of the company has created a valid expectation that the company will clean up the environment. We say could be argued because there is no clear answer and it may well be determined by the subjective assessment of the directors and auditors as to whether there is a constructive obligation. The example 2B in IAS 37 would support making a provision.

134 Pearson Education Limited 2006

Barry Elliott and Jamie Elliott: Financial Accounting and Reporting (tenth edition) Instructors Manual

Chapter 12: Question 4


The proposal to include the 62.5m and 40m in sales and cost of sales is incorrect. If it were a genuine disposal it should have been treated as the disposal of a non-current asset with the profit included in the income statement, separately disclosed if material. The substance of this transaction is, however, a secured loan in that the company is going to have the continued use of the equipment for the whole of its economic life. There should be a charge of 10% in the Income Statement for the three months from 1 October 20x5. In the balance sheet the asset will continue to be shown as a non-current asset and a years depreciation charged. There will also be a liability for the amount of the loan less any capital repayment that has been made.

135 Pearson Education Limited 2006

Barry Elliott and Jamie Elliott: Financial Accounting and Reporting (tenth edition) Instructors Manual

CHAPTER 13

Chapter 13: Question 1 DDB AG


Issue of deep discount bond
Charges to income statement and carrying value in BS shown in tabular form (i) Cash Carrying value At 1 Apr 1 1 2 3 4 5 (2,500125100) (10% of 2,500) (10% of 2,500) (10% of 2,500) (10% of 2,500) (2,500 +10% of 2500) Net cash flow flows 000 (2,275) (2,250 (2,250 (2,250 (2,250 (2,750 1,475 (12.5% 2,275) (12.5% 2,309.375) (12.5% 2,348.046) (12.5% 2,391.552) (12.5% 2,440.496) Adj.* 284.375 288.671 293.506 298.944 305.062 4.442 1,475.000 At 31 Mar At 31 Mar At 31 Mar At 31 Mar At 31 Mar (ii) to IS 000 (iii) in BS 000 2,275.000 2,309.375 2,348.046 2,391.552 2,440.496

Finance charge

*Adjustment necessitated by rounding of implicit rate to 12.5%

Workings Implicit rate has been determined by interpolation via formula t=n t=1

At (1+r)t

The initial cost of 2,275,000 is deducted to arrive at the net present value. Using 13% t=n t=1

250 + 1.131 1.132

250 + 1.133

250 + 1.134

250 + 1.135

2,750

2,275,000

= 221,239 + 195,787 + 173,263 + 153,330 + 1,492,590 2,275,000 = 38,791 then using 12% t=n

223,214 + 199,298 + 177,945 + 158,880 + 1,560,424 2,275,000 = 44,761

136 Pearson Education Limited 2006

Barry Elliott and Jamie Elliott: Financial Accounting and Reporting (tenth edition) Instructors Manual

t=1 Implicit rate = 12% +

[(

44,761 (44,761 + 38,791)

= 12.5%

) ]
1%

= 12.536 say

137 Pearson Education Limited 2006

Barry Elliott and Jamie Elliott: Financial Accounting and Reporting (tenth edition) Instructors Manual

Chapter 13: Question 2 RPS plc


Redemption of preference shares The treatment of the finance cost of preference shares follows the pattern of loan debt. IAS 32 requires that redeemable preference shares are presented and treated as debt instruments because they are in substance debt. Calculation of finance costs and outstanding principal sum
(i) Cash Balance At 1 Oct 1 (100 50) 1 (Div 5% 1,000) 2 (Div 5% 1,000) 3 (Div 5% 1,000) 4 (Div 5% 1,000) 5 (1000+div of 5% 1,000) 1050 Net cash flow 300 (6.2% 989.1) Adj.* 61.3 (0.4) 300.00 flows 000 (950) 50 50 50 50 (6.2% 950) (6.2% 958.9) (6.2% 968.4) (6.2% 978.4) At 30 Sept At 30 Sept At 30 Sept At 30 Sept At 30 Sept (ii) Finance charge 000 58.9 59.5 60.0 60.7 000 950 958.9 968.4 978.4 989.1 (iii)

*Adjustment caused by rounding in determining implicit rate of 6.2%, namely

t=n t=1 For interest, using 6% t=n t=1 = then 7% = t=n t=1 =

At (1+r)t

1 = 0

50 + 50 + 50 1.061 1.062 1.063 47.2 + 44.5

+ 50 + 1050 1.064 1.065

950

+ 41.98 +

39.60 + 784.6 950 = 7.88

50 + 50 + 1.071 1.072 46.7 + 43.7 +

50 + 1.073

50 + 1050 950 1.074 1.075 + 748.6 950 = 32.1

40.8 + 38.1

138 Pearson Education Limited 2006

Barry Elliott and Jamie Elliott: Financial Accounting and Reporting (tenth edition) Instructors Manual

Interpolation gives rate of 6% + 39.98

7.88

1%

= 6.2%

Treatment of total finance costs through the life-span of the capital instrument IAS 32 stipulates that the finance costs of redeemable preference shares are to be shown in the income statement but separately after interest. Income Statement for year ended 30 September (extracts)
Years 1 000 Interest Finance cost on redeemable 2 000 3 000 4 000 5 000

preference shares

58.9

59.5

60.0

60.7

60.9

The balance sheet extracts reveal the impact of the IAS regarding liabilities as follows: Balance Sheet as at 30 September (extracts)
Years 1 000 Long-term liabilities: Redeemable preference shares 958.9 968.4 978.4 989.1 2 000 3 000 4 000 5 000

139 Pearson Education Limited 2006

Barry Elliott and Jamie Elliott: Financial Accounting and Reporting (tenth edition) Instructors Manual

Chapter 13: Question 3 Little Raven


(a)

The considerations involved in deciding how to account for the issue:


the issue is made at a substantial discount the coupon rate is significantly below market rates adopting substance over form, the discount is effectively rolled-up interest and should be accounted for over the period of the borrowing the balance sheet should report the obligation to redeem at par and the income statement should report the true cost of the borrowing. If the borrowing was accumulated for:

(i) As per the question:


DR Cash CR Debt and each year, DR Income statement CR Cash 300 300 4,000 4,000

neither the obligation to repay nor the true cost of the borrowing would be fairly reported. (ii) Taking advantage of the legal point (available in some countries) that permits discount on issue to be debited to share premium account, the debt could be reported as follows:
DR Cash DR Share premium a/c CR Debt and, each year DR Income statement CR Cash 300 300 4,000 1,000 5,000

in which case the amount of debt would be fairly reported but not the true cost of the debt. (iii) Alternatively,
DR Cash DR Unamortised discount CR Debt And, each year, DR Income statement CR Cash DR Income statement CR Unamortised discount X 300 300 4,000 1,000 5,000

with amortisation of discount on an appropriate basis over period of debenture.

140 Pearson Education Limited 2006

Barry Elliott and Jamie Elliott: Financial Accounting and Reporting (tenth edition) Instructors Manual

At each year-end the debt would be reported as 5,000 less unamortised discount. Such accounting achieves the objective of reporting the actual amount repayable and the true cost of the debt but is not the approach adopted by IAS 32. (iv) Under IAS 32 the approach would be: on issue date
DR Cash CR Debt with the net proceeds of issue. X X

determine finance costs as total amounts repayable (interest plus redemption) less net proceeds of issue allocate finance costs to each period at a constant rate on the carrying amount of the debt by
DR Income statement CR Debt DR Debt CR Cash X X X X

with amounts paid in each period

(b)
Period y/e 30.9.X2 30.9.X3 30.9.X4 30.9.X5 30.9.X6

Carrying amount at beginning 000 4,000 4,159 4,336 4,534 4,754

Finance cost (11.476%) 000 459 477 498 520 546 2,500 000 (300) (300) (300) (300) (300 + 5,000) Payments

Carrying amount at end 000 4,159 4,336 4,534 4,754

(300 x 5 = 1,500 + 5,000 = 6,500 4,000 = 2,500)

Revised income statements for the year ended 30 September


20X5 Turnover Cost of sales Gross profit Overheads Interest payable debenture other 6,700 (3,025) 3,675 (600) (520) (75) 20X4 (restated) 6,300 (2,900) 3,400 (550) (498) (50)

141 Pearson Education Limited 2006

Barry Elliott and Jamie Elliott: Financial Accounting and Reporting (tenth edition) Instructors Manual

Profit for the financial year Retained profit brought forward, as previously stated 4,300 Prior years adjustment [159 + 177] [159 + 177 + 208] Retained profit brought forward restated Retained profit, carried forward

2,480 1,800

2,302

(336) (544) 3,756 6,236 1,464 3,766

142 Pearson Education Limited 2006

Barry Elliott and Jamie Elliott: Financial Accounting and Reporting (tenth edition) Instructors Manual

CHAPTER 14

Chapter 14: Question 1 Kathryn


(a) The original IAS 19 approach to defined benefit pension schemes needed to be reviewed for a number of reasons as follows:

Misleading balance sheet The original approach to the valuation of the asset or liability for pensions in the balance sheet was potentially misleading to users of the accounts, and did not follow the statement of principles. The income statement charge was smoothed out across all the years of service of the employees. The concept that was being followed in doing this was the accruals concept. Problems for presentation rose when surplus or deficits arose on the pension scheme. For example suppose a company had: Normal contributions Surplus Average remaining working life of staff The actuary recommended a two-year contribution holiday. The annual income statement account charge would be 3m (the surplus of 10m has been spread over the remaining working lives of 5 years), but in the balance sheet after one year would be a liability of 3m. This liability would grow to 6m in the second year, and would only return to nil by the end of year 5. This balance sheet liability could be understood by users of the financial statements to mean that the company owed the pension fund money. In fact if anything the company had overpaid into the pension scheme. This approach to pension accounting does not meet the Framework Document in two ways. Firstly the framework prioritises the balance sheet over the income statement whereas the original IAS 19 made the income statement the key statement; and secondly the balance sheet asset or liability does not meet the definition of the item. In the above example for instance the liability that is created is not an obligation to transfer economic benefits as a result of past transactions or events. Internationally inconsistent The original version of IAS 19 was out of line with the approaches in US GAAP. The revised version of the standard is nearer to the requirements to US GAAP as it follows similar valuation principles for assets and obligations, although variations still exist in recognition of gains and losses. Valuation of pension fund assets and liabilities The original IAS 19 did not use valuation principles for assets and liabilities that were internationally consistent or the most realistic methods available. Assets were valued at actuarial 5m per annum 10m 5 years

143 Pearson Education Limited 2006

Barry Elliott and Jamie Elliott: Financial Accounting and Reporting (tenth edition) Instructors Manual

value as opposed to a market value and liabilities of the pension fund were discounted at the expected rate of return on assets, not a realistic discount rate for liabilities. The amended standard has addressed this by requiring pension scheme assets to be measured at market values and liabilities to be valued using the projected unit credit method discounted at an appropriate corporate bond rate.

(b)

Income statement
Operating cost
000 Pension cost current service cost (W2) 000 (600)

Financing cost
Expected return on assets (W1) Interest cost (W2) Net return 1,155 (1,020) 135

[Under IAS 19 it is not necessary to include the income statement income or expense as operating and finance costs, it would all be acceptable under operating costs, however this split is appropriate given the nature of the income and expense items.] Balance sheet
Pension liability Present value of obligations Market value of assets (10,900) 10,700 (200)

Statement of movement in equity


Actuarial gain on the obligations (W2) Actuarial loss of the assets (W1) Loss in equity 120 (855) (735)

WORKINGS
W1 Assets of the pension fund
Market value of assets as at 1 May 2000 Expected return on assets 11% Contributions Benefits paid Actuarial gains (losses) bal. fig. Market value of assets as at 30 April 2001

000
10,500 1,155 700 (800) (855) 10,700

144 Pearson Education Limited 2006

Barry Elliott and Jamie Elliott: Financial Accounting and Reporting (tenth edition) Instructors Manual

W2

Obligations of the pension fund Present value of the obligations as at 1 May 2000 Interest cost 10% Current service cost Benefits paid Actuarial (gains) losses bal. fig. 10,200 1,020 600 (800) (120) 10,900

Present value of obligations as at 30 April 2001

145 Pearson Education Limited 2006

Barry Elliott and Jamie Elliott: Financial Accounting and Reporting (tenth edition) Instructors Manual

Chapter 14: Question 2 Donna


In 2004 the IASB amended IAS 19 Employee Benefits and introduced an extra treatment for actuarial gains and losses. Under the previous version of the standard actuarial gains and losses had to be recognised in the income statement, but only if they exceeded the greater of 10% of the present value of the obligations or 10% of the assets. In December 2004 an amendment to the standard was issued however that gave an extra treatment. If a company chooses immediate recognition of all gains and losses they can recognise the gains and losses outside the income statement, which would mean in equity. The impact of these approaches is shown below:

Workings
Change in the obligation
2002 Present value of obligation, 1 January Interest cost Current service cost Benefits paid Actuarial (gain)/loss on obligation (balancing figure) Present value of obligation, 31 December 3,500 210 150 (140) (120) 3,600 2003 3,600 180 160 (150) (290) 3,500 2004 3,500 140 170 (130) (480) 3,200

Change in the assets


Fair value of plan assets, 1 January Expected return on plan assets Contributions Benefits paid Actuarial gain/(loss) on plan assets (balancing figure) Fair value of plan assets, 31 December 3,200 320 120 (140) (100) 3,400 3,400 306 120 (150) (76) 3,600 3,600 288 130 (130) (288) 3,600

10% corridor The limits of the 10% corridor need to be calculated in order to establish whether actuarial gains or losses exceed the corridor limit and therefore need recognising in the income statement. Actuarial gains and losses are recognised in the income statement if they exceed the 10% corridor, and they are recognised by being amortised over the remaining service lives of employees.

146 Pearson Education Limited 2006

Barry Elliott and Jamie Elliott: Financial Accounting and Reporting (tenth edition) Instructors Manual

The limits of the 10% corridor are set (at 1 January each year) at the greater of: (a) 10% of the present value of the obligation before deducting plan assets; and (b) 10% of the fair value of plan assets.
2002 Limit of 10% corridor (at 1 January) Actuarial gains and losses unrecognised (1 January) Actuarial gain (loss) recognised over 10 years Cumulative unrecognised gains (losses) (1 January) Gains (losses) on the obligation Gains (losses) on the assets Cumulative gains (losses) before amortisation Amortisation in the period Cumulative unrecognised gains (losses) (31 December) 120 (100) 20 20 290 (76) 234 234 480 (288) 426 426 20 20 234 234 320 2003 340 2004 350

There would be some recognition of actuarial gains in 2005 as the unrecognised gains and losses in 2004 exceed the 10% corridor as measured at 31 December 2004. Accounts presentation The final step is to work out the balance sheet and income statement position for the company. Income statement
2002 Current service cost Interest cost Expected return on plan assets Income statement charge 150 210 (320) 40 2003 160 180 (306) 34 2004 170 140 (288) 22

147 Pearson Education Limited 2006

Barry Elliott and Jamie Elliott: Financial Accounting and Reporting (tenth edition) Instructors Manual

Balance sheet
Present value of obligation, 31 December Fair value of assets, 31 December Unrecognised actuarial gains (losses) Liability in balance sheet 3,600 (3,400) 20 220 3,500 (3,600) 234 134 3,200 (3,600) 426 26

Equity recognition approach The working will still remain the same, but the recognition of actuarial gains and losses changes. Income statement
2002 Current service cost Interest cost Expected return on plan assets 150 210 (320) 2003 160 180 (306) 2004 170 140 (288)

Income statement charge

40

34

22

Balance sheet
Present value of obligation, 31 December Fair value of assets, 31 December Liability (Assets) in balance sheet 3,600 (3,400) 200 3,500 (3,600) (100) 3,200 (3,600) (400)

Statement of movement in equity


Actuarial gains recognised in the period 20 234 426

148 Pearson Education Limited 2006

Barry Elliott and Jamie Elliott: Financial Accounting and Reporting (tenth edition) Instructors Manual

CHAPTER 15

Chapter 15: Question 1 Notes to Assist in Answering


The Government deliberately sets up special provisions to reduce taxes in order to encourage certain conduct. Examples might be allowance for payments in to pension funds or for capital investment. The running of a companys affairs to take maximum benefit of items such as these is tax planning. When a company alters its behaviour solely for tax purposes, with no commercial reason, with the intention of saving tax by using the tax system in a way not intended by Parliament, this is called tax avoidance. Tax evasion is when a company illegally hides income from the tax authorities. Tax planning is to be encouraged, and it is for an accountant to point out the opportunities to use it. Tax evasion is illegal, and an accountant is under an obligation to prevent it happening. Tax avoidance is legal; an accountant is under an obligation to ensure that steps are not taken which are illegal and that details are reported accurately to the authorities. In all dealings relating to the tax authorities, an accountant must act honestly and do nothing that might mislead the authorities, but he must do all that he can to assist his employer within these criteria.

149 Pearson Education Limited 2006

Barry Elliott and Jamie Elliott: Financial Accounting and Reporting (tenth edition) Instructors Manual

Chapter 15: Question 2


(a)
31/3/20X2 Cost Depn/allce 31/3/20X3 Depn/allce Accounts (Depreciation) 25,000.00 2,812.50 22,187.50 3,750.00 18,437.50 31/3/20X4 Depn/allce 31/3/20X5 Depn/allce 31/3/20X6 Depn/allce 3,750.00 14,687.50 3,750.00 10,937.50 3,750.00 7,187.50 Tax (capital allowances) 25,000.00 4,687.50 20,312.50 5,078.13 15,234.38 3,808.59 11,425.78 2,856.45 8,569.34 2,142.33 6,427.00 Difference (timing)

1,875.00 1,875.00 1,328.13 3,203.13 58.59 3,261.72 (893.55) 2,368.16 (1,607.67) 760.50

Tax Calculated By Deferral Method


Deferred tax charge in year 20% 375.00 30% 20% 19% 19% 398.44 11.72 (169.77) (302.57) Deferred tax provision 375.00 Balance at 31/3/20X2 773.44 785.16 615.38 310.12 Balance at 31/3/20X3 Balance at 31/3/20X4 Balance at 31/3/20X5 Balance at 31/3/20X6

31/3/20X2 31/3/20X3 31/3/20X4 31/3/20X5 31/3/20X6

1,875.00 1,328.13 58.59 (893.55) (1,607.67)

Tax Calculated By Liability Method


Difference As at (timing) Tax rate 31/3/X2 31/3/X3 31/3/X4 31/3/X5 31/3/X6 --------375.00 -------960.94 --------652.34 -------449.95 As at 31/3/X2 20% 1,875.00 3203.13 3261.72 2368.16 760.50 ---------144.69 As at 31/3/X3 30% As at 31/3/X4 20% As at 31/3/X5 19% As at 31/3/X6 19%

(b) Under the liability method the focus is on the balance sheet (the objective being to compute
the deferred tax liabilities), whereas the deferral method places the focus on the Profit and Loss Account (the objective being to show the annual effect that has arisen in the year of account).

150 Pearson Education Limited 2006

Barry Elliott and Jamie Elliott: Financial Accounting and Reporting (tenth edition) Instructors Manual

Chapter 15: Question 3 Notes to Assist in Answering


The law has been amended to allow the Inland Revenue to accept accounts drawn up in accordance with IFRS, so that two different standards will be acceptable for some years. Therefore the legislation will have to provide for different treatment of specific items under UK GAAP and IFRS. The Finance Act 2004 included legislation which ensured that companies that adopted IFRS to draw up their accounts would receive broadly equivalent tax treatment to companies that continue to use UK GAAP. The clear intention of these provisions is to defer the major tax effects of most transitional adjustments until the tax impact becomes clearer. It remains to be seen whether the taxation effects of any significant changes in profit resulting from the change from UK GAAP to IFRS will be deferred until UK GAAP becomes truly aligned with IFRS. The move towards IFRS is leading to a detailed study of accounting theory and principles, so that the accounting treatment may eventually become the benchmark standard for taxation purposes, although this will take several years to reach fruition (if it proves to be attainable).

151 Pearson Education Limited 2006

Barry Elliott and Jamie Elliott: Financial Accounting and Reporting (tenth edition) Instructors Manual

Chapter 15: Question 4 Notes to Assist in Answering


For Discounting: Most transactions take place at fair value. Rational buyers and sellers will ensure that this fair value reflects the time value of money and the risk associated with the future expected cash flows, which means that market prices generally will reflect such factors. To be consistent, these factors need also to be reflected in the other measures that can be used to determine the carrying amount of assets (in other words, value in use and net realisable value) and the carrying amount of any liabilities measured by reference to expected future cash flows. It follows that, when basing carrying amounts on future cash flows, those cash flows will need to be discounted.

Against Discounting: The reliable determination of deferred tax assets and liabilities on a discounted basis requires detailed scheduling of the timing of the reversal of each temporary difference. In many cases such scheduling is impracticable or highly complex. Therefore, it is inappropriate to require discounting of deferred tax assets and liabilities. To permit, but not to require, discounting would result in deferred tax assets and liabilities which would not be comparable between enterprises. Discounting would result in deferred tax assets and liabilities which would not be comparable between enterprises unless there was a set methodology using standard prescribed discount rates. In some cases where capital expenditure is uneven, an unexpected effect of discounting could be to turn an eventual liability into an initial asset. Discounting is not generally used in financial accounting, so its use for deferred taxation would be an exception to general accounting principles.

152 Pearson Education Limited 2006

Barry Elliott and Jamie Elliott: Financial Accounting and Reporting (tenth edition) Instructors Manual

CHAPTER 16

Chapter 16: Question 1 Universal Entrepreneurs plc


(a) The principles outlined in IAS 16 are that: A non-current asset is assessed at the year end to ensure that it has not been impaired. Fair charge is made to the income statement each year for the benefit of accruing to that accounting period for use of the asset concerned. In no way does the IAS address the notion of showing on the balance sheet under the heading of Non-current assets either the value of the assets to the enterprise or the value at which they might be sold. It was this factor that caused property investment companies to feel that they were disadvantaged by the requirements of IAS 16 to depreciate buildings when these formed the major proportion of their asset structure. They argued strongly that the assets were not USED in the business but were held, like any other investment, for their income-producing value and potential capital growth. As a result of these representations the IASC developed IAS 40. (b) (i) (ii) (iii) (iv) (v) Depreciate on the basis of the rate of extraction of growth over the 10 year period in reviewing annually. The cost of the building (4,000,000) should be depreciated over its useful life. It is not an investment property and the period of the lease granted is irrelevant. 20% per annum straight-line. Depreciate on the basis of actual flying hours. Treat as an investment property, revaluing annually but providing depreciation as it is a base of less than 20 years.

(c) The revalued amount of the buildings should be depreciated over the remainder of their useful lives, taking account of the amounts of depreciation already provided. Unless the value of the land is being consumed in some way (e.g. by mining) this should not be depreciated over the remaining period of the bases, again having account of the amounts of amortisation already provided. When the valuer is instructed in respect of the freehold properties it must be made clear that interests of land need to be distinguished from those in the buildings thereon.

153 Pearson Education Limited 2006

Barry Elliott and Jamie Elliott: Financial Accounting and Reporting (tenth edition) Instructors Manual

Chapter 16: Question 2 Mercury


(a) Identifying the method

The method of depreciation is the diminishing balance method. The following calculations show that the rate applied at 20%.
20X6 charge 20X7 charge Cumulative provision = 20% of 80,000 = 20% of 64,000 = = = 16,000 12,800 28,800

(b)

How the accumulated depreciation in line (B) was calculated

B/d from (a) above:


20X6 20X7 20X8 20% of 80,000 20% of 64,000 Balance b/f st Less: 1 disposal 20X6:15,000 x 20% = 20X7: 12,000 x 20% = Less: 2 disposal 20X6: 30,000 x 20% = 20X7: 24,000 x 20% =
nd

= =

16,000 12,800 28,800

3,000 2,400 6,000 4,800

(5,400)

(10,800)

(16,200) 12,600 4,480 10,000 1,000 28,080

20% of (80,000 45,000 disposed of) 12,600 for accumulated depreciation 20% of 50,000 replacement for 2nd disposal = Depreciation on other asset Total given in question

(c)

How the figures for 20X9 are calculated

20X9 Property, plant and equipment


Acquired 20X6 35,000 17,080 17,920 3,584 14,336 Acquired 20X7 50,000 10,000 40,000 8,000 32,000 Acquired 20X8 (balancing figure) 5,000 1,000 4,000 800 3,200 Total 20X9 90,000 28,080 61,920 12,384 49,536

Cost Depreciation to date Charge for 20X9

154 Pearson Education Limited 2006

Barry Elliott and Jamie Elliott: Financial Accounting and Reporting (tenth edition) Instructors Manual

(d)

Calculation of profit/(loss) on disposal


15,000 (5,400) (8,000) 1,600

Plant disposal I
Cost Less: Depreciation Cash Loss

Plant disposal II
Cost Less: Depreciation Cash Profit 30,000 (10,800) (21,000) 1,800

155 Pearson Education Limited 2006

Barry Elliott and Jamie Elliott: Financial Accounting and Reporting (tenth edition) Instructors Manual

Chapter 16: Question 3 Amy


(a) The figures should be the total cost of making the non-current asset usable, excluding all costs of actually using it. Therefore, 11,000 + 100 + 200 + 400 = 11,700 The additional component is cost of machine as it enhances the revenue-earning capacity of the asset. The replacement parts are cost of using machine hence the difference in treatment between the two. Maintenance is obviously a cost of usage. (b) Depreciation spreads the cost (or value) of an item over its useful life, in appropriate proportion to the benefit (usefulness). It is necessary in accordance with the matching convention allocating expense against corresponding benefit, as part of the profit calculation. (c) The straight-line method charges a constant percentage of the cost (or value) each year. The diminishing balance method charges a constant percentage of the net book value (cost less accumulated depreciation brought forward). Thus the straight-line method has a constant charge but the diminishing balance method has a charge reducing each year of the asset life. The two methods therefore make different assumptions about the usefulness, the trend or pattern of benefit, of the fixed asset concerned. (d) Objectivity implies lack of bias. It removes the need for, and the possibility of, subjectivity, of personal opinion. For an accounting figure to be objective, it must be expected that all accountants would arrive at the same figure. Clearly the figure stated on an invoice has a high degree of objectivity. However, the calculation of depreciation is based on estimates of future life and future usefulness and is therefore highly subjective. (e) This practice can claim the advantage of greater prudence, as the expense is always the higher of the two possibilities. However, it seems to lack consistency. Perhaps more importantly, it obviously fails to attempt to follow the matching convention. It makes no attempt to make the trend of expenses consistent with the trend of benefit or usefulness. If the profit figure, or profit trend, is regarded as important, then it seems an unsatisfactory practice.

156 Pearson Education Limited 2006

Barry Elliott and Jamie Elliott: Financial Accounting and Reporting (tenth edition) Instructors Manual

Chapter 16: Question 4 Small Machine Parts Ltd


(a)
Opening balance Interest at 15% Depreciation Year 1 20,000.00 3,000.00 23,000.00 5,914.43 17,085.57 Year 2 17,085.57 2,562.84 19,648.41 5,914.43 13,733.98 Year 3 13,733.98 2,060.10 15,794.08 5,914.43 9,879.65 Year 4 9,879.65 1,481.95 11,361.60 5,914.43 5,447.17 Year 5 5,447.17 817.08 6,264.25 5,914.43 349.82

The income from secondary assets is calculated at 15% of the depreciation charge less the notional interest.
Year 1 5,914.43 3,000.00 2,914.43 437.16 Year 2 5,914.43 2,562.84 3,351.59 502.74 437.16 Year 3 5,914.43 2,060.10 3,854.33 578.15 437.16 502.74 939.90 Year 4 5,914.43 1,481.95 4,432.48 664.87 437.16 502.74 578.15 1,518.05 Year 5 5,914.43 817.08 5,097.35 764.60 437.16 502.74 578.14 664.87 2,182.92

Depreciation Interest 15%

437.16

Income statement entries Cash flow


Year 1 25,000.00 5,914.43 19,085.57 19,085.57 3,000.00 22,085.57 Year 2 25,000.00 5,914.43 19,085.57 437.16 19,522.73 2,562.84 22,085.57 Year 3 25,000.00 5,914.43 19,085.57 939.90 20,025.47 2,060.10 22,085.57 Year 4 Year 5 25,000.00 25,000.00 5,914.43 5,914.43 19,085.57 19,085.57 1,518.05 2,182.92 20,603.62 21,268.49 1,481.85 817.08 22,085.57 22,085.57

Operating CF Depreciation Operating profit Income from secondary assets Interest Net profit

(b) The annuity method is recommended because it attempts to show the effect of the loss of interest suffered as a result of investing the funds in non-current assets within the organisation. It does this by charging notional interest in addition to the depreciation charge with a reduction for the estimated secondary income on the difference between the depreciation charge and the notional interest. The method suggested of charging the annual average cost is frequently met in practice but is less accurate in that it fails to take account of the opportunity cost of the interest foregone.

157 Pearson Education Limited 2006

Barry Elliott and Jamie Elliott: Financial Accounting and Reporting (tenth edition) Instructors Manual

Chapter 16: Question 5 Calculation of Depreciation


(a)
Year 1
Straight-line (SF800,000 SF104,000)/4 Reducing balance 40% of SF696,000 = SF174,000 = SF278,400

(b) Comment to include:


Directors responsible under IAS 16 for selecting an appropriate method. Little guidance given as to how to exercise the choice but the following matters may be relevant: risk of technological change incidence of repairs extent to which the asset characteristics favour a particular method, e.g. a lease would be amortised evenly over its life.

158 Pearson Education Limited 2006

Barry Elliott and Jamie Elliott: Financial Accounting and Reporting (tenth edition) Instructors Manual

Chapter 16: Question 6 AB


(a) IAS 36 (i) Indicators (assuming significant in all cases) Market value lower than book value Lower expected cash flows affecting the value in use Rates of return have increased adversely affecting the recoverable amount Adverse change in the environment e.g. technological, economic or legal or in physical state of asset e.g. obsolescence or damage Adverse change in the use to which asset is put e.g. reorganisation programme Evidence that the economic performance of the asset will be worse than expected The asset has suffered considerable physical change, or obsolescence or physical damage

Cost of construction over-run making asset less profitable (ii) Recognition and measurement IAS 36 Impairment of Assets says that if indicated under the above, then undertake a review to establish the extent of any impairment. Criteria in HCA model An asset should not be valued at an amount greater than its cost or recoverable amount The recoverable amount being the higher of net selling price and value in use (net present value of future cash flows).

Criteria in Revaluation model Compare the carrying value of the asset with its net selling price or value in use. If the net selling price OR value in use exceeds the carrying value, no write-down is necessary If the recoverable amount is lower than the carrying value, the asset is impaired and the carrying amount of the asset should be reduced to its recoverable amount.

Recognition Any recognition is an impairment loss to be recognised as an expense immediately in the income statement.

What if it is not possible to estimate the recoverable value of an individual asset? This can occur if the asset does not generate independent cash flows and in such a case, the recoverable amount of the assets cash generating unit should be calculated together with value in use on the same basis.

159 Pearson Education Limited 2006

Barry Elliott and Jamie Elliott: Financial Accounting and Reporting (tenth edition) Instructors Manual

An impairment loss is only recognised where its recoverable amount is less than the carrying amounts of the items in that unit. Allocation of impaired amount where the HCA model is being followed Any specific impairment of assets should be dealt with initially then allocate first to goodwill, then to intangible assets which have no active market, then to assets whose Net Selling Price is less than their carrying value and finally to other assets on a pro rata basis. Allocation of impaired amount where the Replacement model is being followed An impairment loss relating to a revalued asset is treated as a revaluation decrease and therefore charged to revaluation account. Where the impairment loss is greater than the carrying amount of the asset, a liability should only be recognised where it is required by other International Standards. After recognition of an impairment loss, the depreciation charge should be adjusted to allocate the revised carrying amount (less residual value) systematically over its remaining life. An enterprise should review the balance sheet to assess whether a recognised impairment loss still exists or has decreased. Any reversal of an impairment loss should be recognised in the income statement. (b) AB (i) Impairment of machinery Indicators are the inventory losses and the taxi business problems. Procedure: Compare the carrying value ($290,000) with its recoverable amount which has to be calculated. The calculation is to determine the higher of an assets net selling price ($120,000) and its value in use. The value in use is $100,000 discounted at 10% for three years, i.e. $248,600 approx. Thus the recoverable amount would be deemed to be $248,600. AB would therefore write down the asset from $290,000 (carrying value) to $248,600 (its value in use) and recognise the loss of $41,400 in the income statement. (ii) Impairment of the car taxi business treated as a CGI Impairment losses should be recognised if the recoverable amount of the cash generating unit is less than the carrying value of the items of that unit.
At 1 February 20X1 1.1.X1 Goodwill Intangible assets Vehicles Sundry net assets $000 40 30 120 40 230 Impairment loss $000 (15) (30) (45) 1.2.X1 $000 25 30 90 40 185

An impairment loss of $30,000 is recognised first for the specific asset (i.e. the stolen vehicles) and the balance ($15,000) is attributed to goodwill.

160 Pearson Education Limited 2006

Barry Elliott and Jamie Elliott: Financial Accounting and Reporting (tenth edition) Instructors Manual

Goodwill Intangible assets Vehicles Sundry net assets

1.2.X1 $000 25 30 90 40 185

At 1 March 20X1 Impairment $000 (25) (5) (30)

1.3.X1 $000 25 90 40 155

Note the tricky point that is that the net selling price of the sundry net assets has not fallen. It is therefore not permissible to reduce the sundry net assets.

161 Pearson Education Limited 2006

Barry Elliott and Jamie Elliott: Financial Accounting and Reporting (tenth edition) Instructors Manual

CHAPTER 17

Chapter 17: Question 1 Grabbit plc


(a) PV of future lease payments = 350,000 (cost of asset)
The interest rate to apply to the six payments of 92,500 to equate to a PV of 350,000 is 15%. This is calculated as follows: 80,435 + 69,943 + 60,820 + 52,887 + 45,989 + 39,990 = 350,064
Balance sheet Assets Cost of leased asset Depreciation for year Liabilities PV of future lease payments 80,435 + 69,943 + 60,820 + 52,887 + 45,989 = 310,074, say Income statement Interest on leasing obligation (350,000 15%) Depreciation on leased asset Liability at year end (NB 350,000 52,500 402,500 (92,500) 310,000 Payment Bal. c/f) Cost Interest 52,500 43,750 96,250 310,000 350,000 43,750 306,250

(b) The obligations under a long-term lease are in substance no different from those under a
loan but prior to the introduction of the leasing standard they did not appear on the balance sheet. This made the balance sheet unreliable as one could not be confident that there were not undisclosed liabilities such as leases. Further, where a business had essentially the control of an asset for a substantial period of time they had most if not all of the benefits and risks associated with ownership. It was considered a deficiency that these asset rights were not reflected in the balance sheets, even if they were different in nature to outright ownership. Therefore the leasing standard attempted to capture the assets and liabilities which occurred when longer-term ownership like contracts were entered into.

162 Pearson Education Limited 2006

Barry Elliott and Jamie Elliott: Financial Accounting and Reporting (tenth edition) Instructors Manual

(c) The advantage would be that the obligation would no longer involve the lessee in a longterm commitment although they would probably have long-term access. This would take the relationship out of the financial lease category and put it in as an operating lease. As a consequence, under existing standards it would not have to be capitalised but rather would be disclosed under operating leases. The ratios would be better but only if influential investors did not adjust their figures for the long-term obligations that would probably arise.

163 Pearson Education Limited 2006

Barry Elliott and Jamie Elliott: Financial Accounting and Reporting (tenth edition) Instructors Manual

Chapter 17: Question 2


(a)
The title to the goods acquired on a hire purchase agreement or lease finance remains legally vested in the lessor. Yet in commercial substance, at the point of getting custody of the asset the lessee acquires substantially all the risks and rewards of owning the asset. Hence accountants have preferred to overlook the legal form and focus more on the commercial substance. For example, they have accounted for assets acquired on HP terms or on finance lease terms as if the title passes to the lessee at the date of transfer of the custody of the asset. IAS 17, while endorsing the accounting practice, changes the conceptual basis for this accounting practice. It emphasises that what the lessor capitalises, at the point of acquiring custody of the leased assets, is not the asset itself (which admittedly he does not own yet), but his own right to use that asset.

The amount at which he capitalises this right is the present value of the minimum lease payments he commits himself to.

(b)

In accordance with IAS 17:


A finance lease is a lease that transfers to the lessee substantially all the risks and rewards of owning an asset. All other leases are operating leases. The Standard sets out a presumption that a lease transfers substantially all the risks and rewards of ownership to the lessee IF at the inception of a lease the risks and rewards of ownership are transferred to the lessee and if at the inception of a lease the present value of the minimum lease payments, including any initial payment, amounts substantially to all of the fair value of the leased asset.

Other criteria such as transference of legal title and the right to use the asset for all its life are also considered.

(c)

(1) Smarty plc


Machine acquired under a finance lease

1.8.X7

Obligation

15,000

Provision for depreciation on machine 31.3.X8 31.3.X9 31.3.X0 31.3.X1 Depreciation Depreciation Depreciation Depreciation 2,000 3,000 3,000 3,000

164 Pearson Education Limited 2006

Barry Elliott and Jamie Elliott: Financial Accounting and Reporting (tenth edition) Instructors Manual

Finance charges account 31.3.X8 Obligation under fin. lease 31.3.X9 Obligation under fin. lease 31.3.Y0 Obligation under fin. lease 31.3.Y1 Obligation under fin. lease 28 Obligation under finance lease on machinery account Cash 31.3.X8 Balance c/d Cash 31.3.X9 Balance c/d Cash 31.3.Y0 Balance c/d Cash 4,000 12,200 16,200 6,000 7,429 13,429 6,000 1,972 7,972 2,000 2,000 1.4.Y1 31.3.Y1 Balance b/d Finance charges a/c 1.4.X9 31.3.Y0 Balance b/d Finance charges a/c 1.4.X8 31.3.X9 Balance b/d Finance charges a/c 1.8.X7 31.3.X8 Machinery a/c Finance charges a/c 15,000 1,200 16,200 12,200 1,229 13,429 7,429 543 7,972 1,972 28 2,000 543 31.3.Y1 Income statement 28 1,229 31.3.Y0 Income statement 543 1,200 31.3.X9 Income statement 1,229 31.3.X8 Income statement 1,200

(2)

Income statement entries for y/e 31 March 20X8 extracts:


Finance charge Depreciation 1,200 2,000

Balance sheet as at 31 March 20X8 extracts: Non-current asset Depreciation 15,000 (2,000) 13,000

Liability

12,200

165 Pearson Education Limited 2006

Barry Elliott and Jamie Elliott: Financial Accounting and Reporting (tenth edition) Instructors Manual

Chapter 17: Question 3 The Mission Company Ltd


(a) Income statement
20X6 Item Y Item Z As an operating lease each annual payment is simply rent. 20,000 30,000 20X7 20,000 30,000

(b)
Workings (Item Y) Total lease price: 10 20,000 Purchase price Finance charge Period of lease: 10 years Sum of the digits = 10 + 9 + 8 + 7 + 6 + 5 + 4 + 3 + 2 + 1 = 55 Therefore Charge to 20X6 = 4/55 40,000 Charge to 20X7 = 3/55 40,000 Finance charge to income statement 20X6 Item Y Working (Item Z) Total lease price: 10 30,000 Purchase price = Sum of the digits = 55 Therefore Charge to 20X6 = 6/55 66,000 Charge to 20X7 = 5/55 66,000 Finance charge to income statement 20X6 Item Z 7,200 20X7 6,000 = 300,000 234,000 66,000 2,909 20X7 2,182 = = = 200,000 160,000 40,000

166 Pearson Education Limited 2006

Barry Elliott and Jamie Elliott: Financial Accounting and Reporting (tenth edition) Instructors Manual

Depreciation charge to income statement 20X6 Item Y Item Z 16,000 19,500 20X7 16,000 19,500

(c)

Balance sheet 20X7


Non-current assets Item Y Item Z Cost 160,000 234,000 Current liabilities Item Y Item Z Non-current liabilities Item Y Item Z Item Y 20X8 20X9 20Y0 20Y1 40,000 20,000 20,000 19,273 82,800 Item Z 30,000 30,000 30,000 30,000 120,000 12,000 108,000 2,182 37,818 Accumulated Depn NBV 128,000 117,000 32,000 117,000

[20,000

2 40,000] 55

18,545 25,200

Note: Minimum lease commitments at 31 December 20X7 in relation to finance leases were:

Less: Finance charge allocated to future periods Workings: to liabilities Item Y Year 20X0 20X1 20X2 20X3 147,273 133,818 119,636 B/forward Cost 160,000 Repayment 20,000 20,000 20,000 20,000

Interest 7,273 6,545 5,818 5,091

C/forward 147,273 133,818 119,636 104,727

167 Pearson Education Limited 2006

Barry Elliott and Jamie Elliott: Financial Accounting and Reporting (tenth edition) Instructors Manual

20X4 20X5 20X6 20X7 20X8 20X9

104,727 89,091 72,727 55,636 37,818 19,273

20,000 20,000 20,000 20,000 20,000 20,000 = 20,000 1,455 = 18,545 = 37,818 18,545 = 19,273 Item Z

4,364 3,636 2,909 2,182 1,455 727 40,000

89,091 72,727 55,636 37,818 19,273 -

Therefore current liability non-current liability

Year 20X2 20X3 20X4 20X5 20X6 20X7 20X8 20X9 20Y0 20Y1 Therefore

B/forward 216,000 196,800 176,400 154,800 132,000 108,000 82,800 56,400 28,800 current liability non-current liability

Cost 234,000

Repayment 30,000 30,000 30,000 30,000 30,000 30,000 30,000 30,000 30,000 30,000

Interest 12,000 10,800 9,600 8,400 7,200 6,000 4,800 3,600 2,400 1,200 66,000

C/forward 216,000 196,800 176,400 154,800 132,000 108,000 82,800 56,400 28,800

= 30,000

4,800 = 25,200

= 108,000 25,200 = 82,800

168 Pearson Education Limited 2006

Barry Elliott and Jamie Elliott: Financial Accounting and Reporting (tenth edition) Instructors Manual

Chapter 17: Question 4 X Ltd


(a) Workings to provide figures for income statement and balance sheet entries
Period Liability at start of period 1.1.X4 1.7.X4 1.1.X5 1.7.X5 1.1.X6 1.7.X6 1.1.X7 1.7.X7 1.1.X8 1.7.X8 100,000 91,831 83,306 74,410 65,127 55,440 45,331 34,782 23,774 12,287 Rental payment period 12,000 12,000 12,000 12,000 12,000 12,000 12,000 12,000 12,000 12,000 120,000 Depreciation is over useful economic life, using the historical cost of the leased asset 100,000 = 12,500 per year 8 Income statement Finance 20X4 20X5 20X6 20X7 (1,451 + 992) (3,831 + 3,475) Depreciation Charge 7,306 5,821 4,204 2,443 12,500 12,500 12,500 12,500 19,806 18,321 16,704 14,943 Total Liability during 88,000 79,831 71,306 62,410 53,127 43,440 33,331 22,782 11,774 287 Finance charge 3,831 3,475 3,104 2,717 2,313 1,891 1,451 992 513 12 Liability at end 91,831 83,306 74,410 65,127 55,440 45,331 34,782 23,774 12,287 299

(4.3535%) of period

The balance sheet is as follows: Assets 20X4 Equipment under finance lease Depreciation 100,000 12,500 87,500 100,000 25,000 75,000 100,000 32,500 62,500 100,000 50,000 50,000 20X5 20X6 20X7

169 Pearson Education Limited 2006

Barry Elliott and Jamie Elliott: Financial Accounting and Reporting (tenth edition) Instructors Manual

Liabilities Within 25 years Within 1 year 65,127 *18,179 83,306 45,331 19,796 65,127 23,774 21,557 45,331 299 23,475 23,774

* The 18,179 = 24,000 interest (3,104 + 2,717)

170 Pearson Education Limited 2006

Barry Elliott and Jamie Elliott: Financial Accounting and Reporting (tenth edition) Instructors Manual

Chapter 17: Question 5 Bridge Finance plc


Workings for the lease receivable account $ 1 Jan 5 31 Dec 5 Cost Direct expense Balance 10% interest Cash 1 Jan 6 31 Dec 6 1 Jan 7 31 Dec 7 Balance Interest Cash Balance Interest Cash Balance 37,200 708 37,908 3,791 10,000 31,699 3,170 10,000 24,869 2,487 10,000 17,356

As this is a finance lease the accounts in the lessors books would be; Income Statement Interest revenue Balance sheet Asset Lease receivable $17,356 Current Non-current This can be reconciled as follows: Gross cashflows receivable Less interest Net amount $20,000 2,644 17,356 8,265 9,091 $2,487

171 Pearson Education Limited 2006

Barry Elliott and Jamie Elliott: Financial Accounting and Reporting (tenth edition) Instructors Manual

CHAPTER 18

Chapter 18: Question 1 Environmental Engineering plc


(i) Referring to IAS 38 criteria in (ii) below, only (c) might qualify for deferral as development expenditure.
(a) is applied research. (b) is development cost (1.2m) that has not yet been incorporated into a specific, separate viable project. However, the line between categories is often indistinct in practice, e.g. between development and production costs. Looked at in general, all three relate to a specific project to which it appears expenditure can be separately allocated. However, the outcome is not reasonably certain as to either technical feasibility or commercial viability. We have no idea or projections of sales volume or price/revenue in total and whether it will exceed costs. It is assumed that a plc would have the necessary resources to complete the project but there is no evidence of this. Item (c) would not stand out from (a) and (b) and it is recommended that all be written off as expenses. It could be capitalised later when evidence is produced as to criteria for proceeds.

(ii)

IAS 38 criteria (para. 45):


(a) Technical feasibility (b) Intention to complete and use or sell (c) Ability to use or sell (d) Asset will generate possible future income demonstrate existence (e) of a market, availability of technical, financial and other resources to complete the development or to use or sell.

(iii)
Amortisation should begin with the commencement of production. Any write-off should be over the period the product is expected to be sold. This implies that the amortisation costs can be included in stocks being produced for sale. Deferred development expenditure should be reviewed at the end of each accounting period and, to the extent that it is not considered recoverable, it should be written off.

172 Pearson Education Limited 2006

Barry Elliott and Jamie Elliott: Financial Accounting and Reporting (tenth edition) Instructors Manual

Chapter 18: Question 2 Italin NV


IAS 38: Pure and applied research, always written off in period incurred; development expenditure may be carried forward in certain circumstances.
Income Statement for the year ended 30 September (extract) 20X1 Research expenditure 200 Development cost Depreciation 300 50 300 50 300 50 300 50 300 50 300 50 300 20X2 20X3 20X4 20X5 20X6 20X7

Balance Sheet as at 30 September (extract) 20X1 Intangible fixed assets Tangible fixed assets 300 20X2 250 20X3 200 1,600 20X4 150 1,300 20X5 100 1,000 20X6 50 700 20X7

2,200 1,900

Projects must be reviewed each year. Treatment of fixed assets used in R&D as for any assets.

Disclosure Accounting policy Consistency and application of IAS 38 amounts written off in the period pure and applied research is written off development expenditure is capitalised and written off over six years Movement on development costs capitalised Fixed assets used are depreciated in the normal way over their useful life of 7 years.

173 Pearson Education Limited 2006

Barry Elliott and Jamie Elliott: Financial Accounting and Reporting (tenth edition) Instructors Manual

Chapter 18: Question 3 Oxlag plc


(i)
Research and Development Costs Account 000 Capital costs b/f at start of year (project C) Costs incurred in year: Project A Project C Project D Depreciation: Laboratory: Project C Equipment: Project A Project C Project D Capitalised costs b/f Project C 10 15 20 633 500 633 20 25 265 78 Costs written off to Profit & loss account: Project A Project D 35 98 133 200 Capitalised costs c/f Project C 500 000

(consists of 200 b/f + 265 costs incurred + 20 laboratory depreciation and 15 equipment depreciation) Fixed Assets: Specialized Laboratory Account

000 Cost b/f at start of year Depreciation c/f at end of year 500 45 545 Cost b/f at start of year 500 Depreciation b/f at start of year Depreciation b/f at start of year Depreciation charge for year20 Cost c/f at end of year

000 25 500 545 45

174 Pearson Education Limited 2006

Barry Elliott and Jamie Elliott: Financial Accounting and Reporting (tenth edition) Instructors Manual

Fixed Assets: Specialized Equipment Cost 000 Cost b/f at start of year: Project C Project D Additions: Project A Project D Depreciation c/f at end of year Cost b/f at start of year 50 50 70 295 225 Depreciation b/f at start of year 75 50 Depreciation b/f at start of year Project C Project D Depreciation provided in year: Project A Project C Project D Cost c/f at end of year 10 15 20 225 295 70 15 10 000

Market Research Costs Account 000 Costs b/f at start of year Costs in year 250 75 325 Costs b/f at start of year 325 325 Costs c/f at end of year 000 325

Assumption is that this is a contract that will continue in future years.

(ii)

Amount to be charged as research costs charged in the income statement for the year ended 31 January 20X2
Fees Per T a/c Project A: Project D: Costs Depn Costs Depn 25 10 78 20 98 133 35 Fees

(iii)

Basis of amortisation:
Any reasonably systematic basis of amortisation per IAS 38. Amount spent and written off reconciled with opening and closing balances in the balance sheets. Most likely basis here will be expected sales of the new drug with amortisation being calculated as the proportion of total sales sold during each year.

175 Pearson Education Limited 2006

Barry Elliott and Jamie Elliott: Financial Accounting and Reporting (tenth edition) Instructors Manual

Disclosure Accounting policy stating basis of capitalisation and basis of write-off

(iv)

Balance sheet amounts


000

Fixed assets Intangible assets: Deferred development expenditure (recovery assured by projected future sales) Tangible assets: Land and buildings: Specialised laboratory Plant and machinery: Specialised laboratory equipment Current assets Inventories: Long-term work-in-progress

500 455 155

325

(v)

Disclosures about new improved drug sales

Identify as non-adjusting post balance sheet event which requires disclosure if material in accordance with IAS 10, having arisen between the end of year 31.1.20X2 and the date of signing the accounts on 14.7.20X2. This does appear to nbe material, therefore the accounts will need to disclose: date of new drug going on sale success of new drug expectation that the sales of the new drug will significantly increase following years profits.

176 Pearson Education Limited 2006

Barry Elliott and Jamie Elliott: Financial Accounting and Reporting (tenth edition) Instructors Manual

Chapter 18: Question 4 Goodwill and Intangible Assets


(a) IFRS 3 defines goodwill as any excess of the cost of the acquisition over the acquirers interest in the fair value of the identifiable assets, liabilities and contingent liabilities acquired at the date of the exchange transaction (i.e. the date the assets were acquired). The cost of the acquisition is the cash or cash equivalent paid or the fair value, at the date of exchange, of the other purchase consideration given in exchange for control over the net assets of the other enterprise plus any costs directly attributable to the acquisition. Fair value is defined as the amount for which an asset could be exchanged or a liability settled between knowledgeable, willing parties in an arms length transaction. The fair value of the assets or liabilities at the date of purchase can include a provision for reorganisation costs, provided a plan has been developed within three months of the date of acquisition. The costs included in the provision can only include costs relating to the acquired business for employee redundancy, closing facilities, eliminating product lines and terminating onerous contracts. (b) IFRS 3 requires an impairment test to be carried out. Unless goodwill is impaired continue to carry at cost.
(a) Total net assets per balance sheet 58,234 (i) No adjustment as no readily ascertainable market and no information to verify directors estimate (but see (vi) below) (ii) Valuation Other tangible fixed assets Total fair value Value in balance sheet 23,000 18,000 41,000 38,300

Revaluation
(iii) Inventories at net realisable value (less than replacement cost) 21,600 Loss (iv) Bank loan Loan at 31.5.09: 12,1001.1 Value in balance sheet Increase in liability (v) No adjustment required (as the reorganisation was decided before acquisition and future losses cannot be
3

2,700
20,000

(1,600)

16,105 13,146 12,100 (1,046)

Discounted to 31.5.06 at 7%

177 Pearson Education Limited 2006

Barry Elliott and Jamie Elliott: Financial Accounting and Reporting (tenth edition) Instructors Manual

included in the provision) (vi) Impairment of brand names Fair value of Yukon at acquisition Goodwill: Cost of acquisition 2.5 2.25 10,000 80% Fair value of net assets acquired 80% 52,268 Goodwill Valuation of goodwill Impairment 45,000 41,814 3,186 1,000 2,186 (6,020) 52,268

178 Pearson Education Limited 2006

Barry Elliott and Jamie Elliott: Financial Accounting and Reporting (tenth edition) Instructors Manual

Chapter 18: Question 5 The Brands Debate


Many of the arguments for including brands in the balance sheet are given in Section 18.12 of Chapter 18. Including brands in the balance sheet increases shareholders funds and thus reduces gearing. A reduction in gearing reduces investors and banks perception of the risk of the company and it is likely to increase the companys ability to borrow funds. Also, including brands in the balance sheet shows investors and management the value of the companys brands, thus providing more information to those users of accounts and enabling them to make more rational decisions. A brand which has been purchased by a company can be included in the balance sheet. Under IAS 38 Intangible Assets, the brand is included at cost and amortised over its useful life. The allowed alternative treatment enables the brand to be revalued and reductions in the brands valuation below its original cost are charged to the income statement. IAS 38 says that internally generated goodwill should not be recognised as an asset (para. 36). However, the cost of developing a brand could be taken as development expenditure and this cost subsequently capitalised in the balance sheet and amortised in the income statement. So, it is possible to capitalise internally generated brands. However, the cost of developing a successful brand is likely to be considerably less than its market value (if the brand was purchased, it would be shown in the balance sheet at its market value). So, the treatment of purchased and internally generated brands is different, and in most situations internally generated brands will not be included in the balance sheet (whereas purchased brands would be included). It would be possible to include an internally generated brand in the balance sheet at its current market value, provided it was initially included as development expenditure and the allowed alternative treatment (of IAS 38) of including the brand at its fair value was included for the balance sheet valuation. However, IAS 38 does require the fair value to be determined by reference to an active market. On the subject of separability of brands, when a business is acquired, it is likely to be difficult to distinguish between brands and other goodwill. The total amount of goodwill, being the difference between the purchase consideration and the fair value of assets acquired, can be determined. However, dividing this total goodwill between brands and other goodwill will be difficult and is likely to be subjective. Also, in acquiring a company, a number of brand names may be acquired, and it is likely to be very difficult (and subjective) to say how much each of the brands is worth. On purchased vs home-grown brands, the different accounting treatment has been discussed above. In most situations, purchased brands will be included as an asset in the balance sheet, whereas home-grown brands will not. As both purchased and home-grown brands have value, this different accounting treatment is not consistent. However, this different accounting treatment arises because accountants are prepared to include an item in assets when its purchase price is known (as with purchased brands) but are reluctant to include it as an asset when it has been internally generated (and its market value is not certain). For investors, ideally they would like the value of both purchased and home-grown brands to be included in the balance sheet, but this creates the risk that directors may artificially inflate the value of home-grown brands and thus mislead investors.

179 Pearson Education Limited 2006

Barry Elliott and Jamie Elliott: Financial Accounting and Reporting (tenth edition) Instructors Manual

As with land and buildings, some companies argue that brands have an infinite life. Current expenditure on advertising and marketing the product (e.g. a Mars Bar) maintain the value of the brand, so no amortisation of the brands value is justified. IAS 38 says that intangible assets should be amortised over their life, which should not normally exceed 20 years. One can see that some brands have a life of significantly less than 20 years. For instance, a 1 GHz microprocessor has a life of, at most, only a few years as it is superseded by faster processors. Although the brand of Intel may have a relatively long life, the company must continue to develop its products (i.e. make the microprocessors faster) in order to keep its brand alive. However, other brands, like the Mars Bar, have a life of significantly more than 20 years the Mars Bar existed more than 60 years ago. For most brand names, a life of 20 years is a realistic maximum (many brands have a life of less than 20 years), but some brands may have a significantly longer life. However, although a brand may have had a life of more than 20 years, there is no certainty that it will continue to exist for another 20 years. Many computer companies which were successful 20 years ago no longer exist (e.g. Commodore, Sinclair). How many of todays well-known brands will no longer exist in 10 or 20 years time? All buildings eventually fall down or are demolished, and all brands will eventually die. So, we would argue that the cost of brands should be amortised in the income statement. It is wrong not to amortise the cost of brands, as eventually they will be worthless. There is a further argument that even if the brand continues to be reported in the balance sheet at the existing value, the reality is that expenditure has been currently incurred which effectively replaces the original brand value. This means that there has been a substitution of a new brand for the old rather than a maintenance of the old brand.

180 Pearson Education Limited 2006

Barry Elliott and Jamie Elliott: Financial Accounting and Reporting (tenth edition) Instructors Manual

CHAPTER 19

Chapter 19: Question 1 Sunhats Ltd


1. The principle usually followed is to include in the stock valuation only those expenses which relate to the bringing of the product to its present condition and location. In practice, this often entails: including factory (or production) expenses excluding selling, finance and administration expenses.

2. Sunhats Ltd factory expenses to be included in inventory valuation: Wages of storemen and foremen Salary of production manager Rent and rates, repairs and depreciation; proportion relating to factory and stores would be included e.g. electric power. 3. Expenses to be excluded from inventory valuation: Salaries of sales manager and salespeople, advertising and carriage outwards. These expenses are excluded as they relate to selling and distributing the goods, not to the production of them. Bad debts and bank interest*: these finance charges are excluded as they relate to the business as a whole and not merely to production. Salaries of personnel officer*, buyer*, accountant* and company secretary*, and directors fees*: these administration expenses are excluded, as they similarly relate to the business as a whole. Development expenditure: this is excluded as it is clearly not relevant to the cost of existing stock.

*The items marked with an asterisk are marginal. It can be argued that part of these expenses relate to production and should therefore be regarded as factory overheads. 4. It is important to ensure that the overhead expenses included in the inventory valuation are: appropriate in the circumstances of the business, and included on a consistent basis from year to year.

181 Pearson Education Limited 2006

Barry Elliott and Jamie Elliott: Financial Accounting and Reporting (tenth edition) Instructors Manual

Chapter 19: Question 2 Inventory Valuation Methods


(i)
Date Quantity FIFO 1/7 10/7 12/7 14/7 15/7 20/7 30/7 50 100 9.6 9.4 480 940 20 50 20 9.8 9.6 9.4 196 480 188 2,648 100 9.8 980 20 80 10 9.8 200 784 50 100 80 9.6 9.4 9.4 480 940 752 100 10 1,000 80 10 800 100 20 100 20 10 10 9.8 9.8 1,000 200 980 196 Rate Quantity Rate Quantity Rate Receipts Issues Balance

Cost of goods sold LIFO 1/7 10/7 12/7 14/7 15/7 20/7 30/7 50 100 9.6 9.4 480 940 90 9.4 100 9.8 980 100 9.8 100 10 1,000 80 10

100 800 980 20 100 20 50 100 846 20 50 10 Cost of goods sold 2,626

10 10 9.8 10 9.8 9.4 10 9.6 9.4

1,000 200 980 200 480 940 200 480 94

Weighted average 1/7 10/7 12/7 14/7 15/7 20/7 30/7 50 100 9.6 9.4 480 940 90 9.5 855 2,638 100 9.8 980 100 9.83 983 100 10 1,000 80 10 800 100 20 100 120 20 50 100 170 80 Cost of goods sold 10 10 9.8 9.83 9.83 9.6 9.4 9.5 9.5 1,000 200 980 1,180 196.7 480 940 1,615 760

182 Pearson Education Limited 2006

Barry Elliott and Jamie Elliott: Financial Accounting and Reporting (tenth edition) Instructors Manual

(ii)
FIFO

Advantages and disadvantages

The movement of some stock follows this pattern in reality e.g. perishables. However, the charge to cost of sales will still represent out-of-date prices. This means that a distribution policy based on profits calculated using this method will reduce the operating capital base. The balance sheet value will value stock at approaching current values.

LIFO The movement of stock does not follow this pattern and detailed records will be required to track costs. The charge to cost of sales will represent prices prevalent at date of sale. This means that a distribution policy based on profits calculated using this method will tend to maintain the operating capital base. However, the balance sheet value will value stock at out-of-date values.

Average cost This is a common compromise between the two methods. The advantage is that the average represents a compromise between the FIFO and LIFO methods. However, there is a disadvantage in that the average cost has to be recalculated after each purchase.

(iii)
FIFO

Effect of a physical shortage of inventory

Closing inventory 75 @ 9.4 Cost of sales increased by 5 @ 9.4 LIFO Closing inventory 15 @ 10.0 50 @ 9.6 10 @ 9.4 150 480 9.4 724 47 705

183 Pearson Education Limited 2006

Barry Elliott and Jamie Elliott: Financial Accounting and Reporting (tenth edition) Instructors Manual

Cost of sales increased by 5 @ 10 Weighted average Closing inventory 75 @ 9.5 Cost of sales increased by 5 @ 9.5 47.5 712.5 50

184 Pearson Education Limited 2006

Barry Elliott and Jamie Elliott: Financial Accounting and Reporting (tenth edition) Instructors Manual

Chapter 19: Question 3 Alpha Ltd


Principles The basis on which the stocks are valued in this solution is the one which is most commonly used by companies, i.e. the lower of cost and net realisable value. The term cost includes those overheads which have been incurred in bringing the stocks to their existing condition, namely manufacturing overheads. Selling and distribution expenses have been excluded from cost as it is assumed that these are not incurred until the units are sold.
Valuation details

Raw materials: 100 tons cost 140 per ton = 14,000 The net realisable value is assumed to be greater than this amount as the finished units (which incorporate the steel) sell at a profit, as follows
Selling price Less: Selling and distribution expenses Net realisable value Manufacturing costs (see workings below) Profit per unit 500 60 440 350 90

The current replacement price has not been taken, as it is not within the basis of valuation stated above. However, as the replacement price has fallen this is a suitable time to consider whether the client should be advised to amend the basis of stock valuation to the lower of cost, replacement price and net realisable value, which is more conservative. On this basis the stock would be valued at 130 per ton. Finished units: 100 cost 350 = 35,000 The cost comprises:
Per unit Materials Labour Manufacturing overheads 100% of labour Net realisable value is greater than the cost: Selling price Less: Selling and distribution expenses Net realisable value 500 60 440 50 150 150 350

185 Pearson Education Limited 2006

Barry Elliott and Jamie Elliott: Financial Accounting and Reporting (tenth edition) Instructors Manual

Damaged, finished units: 10 240 = 2,400 These units have been valued at cost less the amount of the loss that will be incurred when the units have been rectified, as follows:
Per unit Cost of finished units Cost to rectify Total cost Less: Net realisable value Loss Amount per unit included in the balance sheet 350 200 550 440 110 110 240 Valuation 350

Semi-finished units: 40 cost 250 = 10,000 The cost comprises:


Per unit Materials Labour Manufacturing overheads 100% of labour Total cost per unit so far 50 100 100 250

An estimate should be made of the cost required to finish the work. If the total estimated cost exceeds the net realisable value, then the excess must be provided for by deducting it from the 250 cost; this is similar in principle to the treatment of the damaged units. For example:
Per unit Total cost per unit so far (as above) Estimated costs to complete Estimated total costs to completion Less: Net realisable value Estimated loss on completion Valuation: Total cost per unit so far Less: Estimated loss on completion 250 30 220 250 220 470 440 30

186 Pearson Education Limited 2006

Barry Elliott and Jamie Elliott: Financial Accounting and Reporting (tenth edition) Instructors Manual

Chapter 19: Question 4 Beta Ltd


1. As the raw materials will realise more than cost, they have obviously been valued at the standard cost, namely 30,000. 2. A review of the price variance account shows that, in total, the actual cost of materials has consistently been well above the standard costs. 3. Consequently, the 30,000 standard cost of raw materials in stock is significantly below the actual cost; and, unless the stock figure is adjusted to the actual cost, this years profit will be understated. (Moreover, the understatement of stocks this year will result in next years profits being artificially inflated.) 4. Therefore, the figure to be included in the balance sheet should not be the standard cost but a figure which is reasonably close to actual cost. This could be done in one of the following ways: (a) Value each item at the actual cost paid for it, by referring to the purchase invoices concerned. However, this may be too laborious, in which case method (b) or (c) should be considered. (b) If the company has revised the standard costs for use in the following year, then it may be suitable to use these revised costs for valuing the stocks in the balance sheet. (Presumably the revised standards are based on the cost applicable around the year-end.) (c) If methods (a) and (b) are impracticable, a rough and ready method may be used, as follows:
Balance on raw materials control account This is equal to the goods purchased in October, November and December, when the price variances totalled Value of raw materials at year-end 2,700 32,700 30,000

Care is needed in using this method, as the price variances may have arisen over a narrow range of materials, in which case the calculations of the adjustment needed should embrace only those materials. Conclusion Standard costs are used mainly as a tool of management control; their use in the valuation of stocks for accounts purposes is merely identical. Standard costs should not be used for stock valuation unless they are reasonably close to actual costs.

187 Pearson Education Limited 2006

Barry Elliott and Jamie Elliott: Financial Accounting and Reporting (tenth edition) Instructors Manual

Chapter 19: Question 5 Bottom


Bottom income statement using FIFO inventory valuation $000 Revenue Cost of sales Gross profit Other operating expenses Profit from operations Investment income Finance cost Profit before tax Income tax expense Net profit for the period (4,000) 24,400 (7,000) 17,400 75,000 (37,600) 37,400 (9,000) 28,400

The change from LIFO to FIFO would be a change of accounting policy. Under IAS 8 (revised) the effects of such a change should be applied retrospectively and comparative figures restated, with the opening balance of retained profits adjusted. Working cost of sales
$000 As originally stated Increase to opening Inventory Increase to closing Inventory As restated 38,000 00,500 (900) 37,600

188 Pearson Education Limited 2006

Barry Elliott and Jamie Elliott: Financial Accounting and Reporting (tenth edition) Instructors Manual

CHAPTER 20

Chapter 20: Question 1 MACTAR


(m) Costs to date Future costs Contract sum Expected profit/(loss) M1 2.1 0.3 2.4 3.0 0.6 M6 0.3 1.1 1.4 2.0 0.6 M62 2.3 0.8 3.1 2.75* (0.35)

* assumes a 10% increase in contract price is allowed and negotiated. Recognised profit Value of work certified % of work certified to total So Recognised profit 1.8 60% 0.36 0.1 5% Note 1 1.3 47% Note 2

Note 1 Contract M6 is probably at too early a stage of completion to recognise any profit. Note 2 The anticipated loss on contract M62 must be recognised in full on the grounds of prudence.
m Recorded as revenue C of S balance Profit M1 1.8 1.44 0.36 Balance sheet work-in-progress Costs to date Plus recognised profits Less: recognised losses Less: progress billings Closing balance Receivables Progress billings Less: Payment received Closing balance 1.75 (1.5) 0.25 0.1 nil 0.1 1.0 (0.75) 0.25 (1.75) 0.71 (0.1) 0.2 2.1 0.36 0.3 (0.35) (1.0) 0.95 2.3 M6 0.1 0.1 nil M62 1.3 1.65 (0.35)

189 Pearson Education Limited 2006

Barry Elliott and Jamie Elliott: Financial Accounting and Reporting (tenth edition) Instructors Manual

Thus the overall profit on these three contracts is 0.01m (10,000) for the year. This is of course a prudent view as one of the projects (M6) has only just started, one project is set for a cost over-run (M62) and one contract is very nearly complete (M1).

190 Pearson Education Limited 2006

Barry Elliott and Jamie Elliott: Financial Accounting and Reporting (tenth edition) Instructors Manual

Chapter 20: Question 2 Lytax Ltd


Contract No. Contract price Costs incurred to date Estimated further cost to complete Estimated cost of postcompletion work Estimated profit/(loss) On contracts Recognised profit/(loss) 580/800 300 470/620 330 218 250 (135) (160) (40) 300 330 (135) (160) (40) 30 800 10 620 45 1,535 20 1,460 5 1,240 106 75 680 800 165 1 1,100 664 2 000 950 535 3 000 1,400 810 4 000 1,300 640 5 000 1,200 1,070 000

Notes 1 2 Recognised profit on the profitable contracts is taken as the proportion that costs taken to revenue bear to total anticipated costs. Other sensible proportions would be acceptable. Losses on unprofitable contracts are recognised in full.

The balance sheet work-in-progress balance will show:


1 000 Costs incurred to date Recognised profits less foreseeable losses Progress billings: Received Awaited Retained Closing balance (615) (60) (75) 132 (680) (40) (80) (15) (615) (25) (60) (25) (385) (200) (65) (170) (722) (34) (84) 190 664 218 2 000 535 250 3 000 810 (135) 4 000 640 (160) 5 000 1,070 (40)

The positive balances on contracts 1 and 5, totalling 322,000, will be presented as an asset. The negative balances on contracts 2, 3 and 4, totalling 210,000, will be presented as a liability. The difference between total progress billings and total receipts will be shown as a receivable.

191 Pearson Education Limited 2006

Barry Elliott and Jamie Elliott: Financial Accounting and Reporting (tenth edition) Instructors Manual

Chapter 20: Question 3 Beavers Ltd


(a)
Wages Materials Other costs HO costs (see Note) Plant (12/15 8,000) Recognised profit

Contract Account
91,000 36,000 18,000 6,000 6,400 40,500 197,900 Progress billings 150,000 Materials c/f WIP c/f 3,000 44,900 197,900

Note It is assumed that the Head Office costs are associated with the provision of contract-related services that cannot be directly allocated to a specific contract. In these circumstances IAS 11 allows their inclusion in contract costs. Any apportionment of general Head Office costs would not be permitted under IAS 11.

(b)

Balance sheet extract as at 30 June 20X7


3,000 44,900

Current assets Inventory materials Long-term contract balance

(c)

Calculation of profit
240,000 154,400 31,600 186,000 54,000

Contract price Costs to date [157,400 3,000 material on site] Estimated further costs to complete [10,000 + 12,000 + 8,000 + 1,600 plant] Estimated profit Recognised profit (say) 180,000/240,000 54,000 = 40,500 Amount recoverable on long-term contract (180,000 150,000) 30,000

192 Pearson Education Limited 2006

Barry Elliott and Jamie Elliott: Financial Accounting and Reporting (tenth edition) Instructors Manual

Chapter 20: Question 4 Newbild SA


(a) Contract Account

Materials Stores Materials Site Wages Site expenses Administration Plant Depreciation Subtotal Recognised profit 13,407 73,078 39,894 4,815 3,742 5,160 140,096 23,099 163,195 163,195 Materials Bal c/f WIP Bal c/f 5,467 22,928 Progress billing (see Note)

134,800

Note The progress billing is the amount received from the customer grossed up by the 15% retention.
Calculation of profit Contract price Costs to date [140,096 5,467] Estimated further costs to complete Guarantee work [2.5% of 780,000] Estimated profit 134,629 490,000 19,500 644,129 135,871 780,000

Recognised profit The work certified as complete has a value of 134,800 ($114,580/0.85). Therefore the contract is around 17% complete (134,800/$780,000 is 17.28%). Therefore recognised profit could be 17% 135,871 = 23,099.

(b)

Income statement
134,800 111,701 23,099

Revenue Cost of sales Profit Balance sheet extract Current assets Inventory materials Long-term contract balance Receivables amount recoverable on long-term contract

5,467 22,928 20,220

193 Pearson Education Limited 2006

Barry Elliott and Jamie Elliott: Financial Accounting and Reporting (tenth edition) Instructors Manual

Chapter 20: Question 5 Good Progress SpA


Note that the contract is 25% complete at the end of 20X0 and 40% complete by the end of 20X1. Therefore 15% of the contract was completed in 20X1.
Income statement entries: 20X1 Revenue Cost of goods sold 112,500 Profit [15%/25%] Balance sheet entries 20X1 Current assets Long-term contracts balance [117,000 112,500] 4,500 [265,000 262,500] 2,500 20X0 37,500 187,500 62,500 150,000 20X0 250,000

194 Pearson Education Limited 2006

Barry Elliott and Jamie Elliott: Financial Accounting and Reporting (tenth edition) Instructors Manual

Chapter 20: Question 6


Solution (a) Water Away
The following extract from the Balfour Beatty 2003 Annual Report deals with such a situation: Revenue on Aberdeen Environmental Services Limited (AES) is related to the volume and quality of the wastewater processed by the plant. AES therefore takes demand risk and has a fixed asset, which is depreciated over the life of the concession. The revenue is recognised as turnover as it is earned.

Solution (b) Learn A head


The following extract from the Balfour Beatty 2003 Annual Report deals with such a similar situation: Hospitals and Schools Balfour Beatty's hospitals and schools concessions receive income based on the availability of the asset, rather than their actual usage. The costs of constructing the asset are therefore accounted for as amounts recoverable on contracts during the course of construction and reclassified to contract debtors when construction is complete. The contract debtor gives rise to an interest income calculation based on an appropriate rate of return for the asset concerned. The income is split into two elements: that relating to the contract debtor, and that relating to the provision of other services such as cleaning and catering. The element of revenue relating to the contract debtor is split between principal repayments, reducing the amount owed to the concession, and interest income, which is credited to the profit and loss account as it is earned. The revenue relating to services is recognised as turnover as it is earned, reflecting the continuing provision of services to the concession.

195 Pearson Education Limited 2006

Barry Elliott and Jamie Elliott: Financial Accounting and Reporting (tenth edition) Instructors Manual

CHAPTER 21

Chapter 21: Question 1 Parent Ltd


(a) Parent Ltd balance sheet as at 1 January 20X1
Ordinary Shares of 1 each Retained earnings Investment in Daughter Ltd Cash[20,000 10,800] Other net assets 40,500 4,500 45,000 10,800 9,200 25,000 45,000 Note: The investment is shown as its fair value of 10,800 and the cash has been reduced by consideration. Consolidated balance sheet as at January 20X1 Parent Ordinary shares Retained earnings Investment in Daughter Cash Other net assets 10,800 9,200 25,000 55,800 2,000 8,800 10,800 10,800 11,200 33,800 55,800 Q 10,800 11,200 33,800 45.000 40,500 4,500 55,800 Daughter 9,000 1,800 10,800 Add 49,500 6,300 55,800 Eliminate (Dr)/Cr (9,000) (1,800) 40,500 4,500 45,000 CBS

Note: Because the cash paid exactly equalled the value of the net assets acquired, there was no difference on consolidation i.e. no positive or negative goodwill.

(b)

Parent Ltd balance sheet as at 1 January 20X7


45,900 5,400 4,500 55,800

Ordinary Shares of 1 each (40,500+(10,800/2) Share premium Retained earnings

196 Pearson Education Limited 2006

Barry Elliott and Jamie Elliott: Financial Accounting and Reporting (tenth edition) Instructors Manual

Investment in Daughter Ltd Cash Other net assets

10,800 20,000 25,000 55,800

Note: The investment is shown as its fair value of 10,800 and the shares are issued at their fair value of 5,400 par value and 5,400 premium.
Consolidated balance sheet as at January 20X7 Parent Ordinary shares Share premium Retained earnings 45,900 5,400 4,500 55,800 Investment in Daughter Cash Other net assets 10,800 20,000 25,000 55,800 10,800 10,800 10,800 20,000 35,800 66,800 ___ Q 10,800 20,000 35,800 45.000 Daughter 9,000 5,400 1,800 10,800 6,300 66,600 (1,800) Add 54,900 Eliminate (Dr)/Cr (9,000) 40,500 5,400 4,500 55,800 CBS

Note: Because the value of the shares issued exactly equalled the value of the net assets acquired, there was no difference on consolidation i.e. no positive or negative goodwill.

197 Pearson Education Limited 2006

Barry Elliott and Jamie Elliott: Financial Accounting and Reporting (tenth edition) Instructors Manual

Chapter 21: Question 2 Parent Ltd


(a) Parent Ltd balance sheet as at 1 January 20X7
Ordinary Shares of 1 each Retained earnings Investment in Daughter Ltd Cash [20,000 16,200] Other net assets 40,500 4,500 45,000 16,200 3,800 25,000 45,000

Note: The investment is shown as its fair value of 16,200 and the cash has been reduced by consideration.
Consolidated balance sheet as at January 20X7 Parent Ordinary shares Retained earnings Investment in Daughter Cash Other net assets 16,200 3,800 25,000 55,800 2,000 8,800 10,800 16,200 5,800 33,800 55,800 ___ Q 10,800 5,400 5,800 33,800 45.000 40,500 4,500 55,800 Daughter 9,000 1,800 10,800 Add 49,500 6,300 55,800 Eliminate (Dr)/Cr (9,000) (1,800) 40,500 4,500 45,000 CBS

Note: Because the cash paid exceeded the value of the net assets acquired, there was a difference on consolidation of 5,400 which appears in the consolidated balance sheet as an asset goodwill this will be reviewed for possible impairment.

(b)

Parent Ltd balance sheet as at 1 January 20X7


Ordinary Shares of 1 each (40,500+(16,200/3) Share premium Retained earnings Investment in Daughter Ltd Cash Other net assets 45,900 10,800 4,500 61,200 16,200 20,000 25,000 61,200

198 Pearson Education Limited 2006

Barry Elliott and Jamie Elliott: Financial Accounting and Reporting (tenth edition) Instructors Manual

Note: The investment is shown as its fair value of 16,200 and the shares are issued at their fair value of 5,400 par value and 10,800 premium. Consolidated balance sheet as at January 20X7 Parent Ordinary shares Share premium Retained earnings 45,900 10,800 4,500 61,200 Investment in Daughter Cash Other net assets 16,200 20,000 25,000 55,800 10,800 10,800 1,800 10,800 Daughter 9,000 Add 54,900 10,800 6,300 72,000 16,200 20,000 35,800 72,000 ___ Q 10,800 (1,800) Eliminate (Dr)/Cr (9,000) CBS 45,900 10,800 4,500 61,200 5,400 20,000 35,800 61,200

Note: Because the value of the shares issued exceeded the value of the net assets acquired, there was a difference on consolidation.

199 Pearson Education Limited 2006

Barry Elliott and Jamie Elliott: Financial Accounting and Reporting (tenth edition) Instructors Manual

Chapter 21: Question 3 Parent Ltd


(a) Parent Ltd balance sheet as at 1 January 20X7
Ordinary Shares of 1 each Retained earnings Investment in Daughter Ltd Cash [20,000 16,200] Other net assets 40,500 4,500 45,000 16,200 3,800 25,000 45,000

Note: The investment is shown as its fair value of 16,200 and the cash has been reduced by consideration.
Consolidated balance sheet as at January 20X7 Parent Ordinary shares Retained earnings Investment in Daughter Cash Other net assets 16,200 3,800 25,000 55,000 2,000 8,800 10,800 16,200 5,800 33,800 55,800 1,200 Q 10,800 (1,200) 4,200 5,800 35,000 45.000 Revaluation increase 40,500 4,500 55,800 Daughter 9,000 1,800 10,800 Add 49,500 6,300 55,800 Eliminate (Dr)/Cr (9,000) (1,800) 40,500 4,500 45,000 CBS

Note: 1. The net assets in the CBS will be increased by 1,200. 2. The fair value of the shares issued (16,200) exceeded the fair value of the net assets acquired (12,000). This difference on consolidation will be reported as goodwill and reviewed for impairment.

(b)

Parent Ltd balance sheet as at 1 January 20X7


Ordinary Shares of 1 each (40,500+(16.200/3) Share premium Retained earnings Investment in Daughter Ltd 45,900 10,800 4,500 61,200 16,200

200 Pearson Education Limited 2006

Barry Elliott and Jamie Elliott: Financial Accounting and Reporting (tenth edition) Instructors Manual

Cash Other net assets

20,000 25,000 61,200

Note: The investment is shown as its fair value of 16,200 and the shares are issued at their fair value of 5,400 par value and 10,800 premium.
Consolidated balance sheet as at January 20X7 Parent Ordinary shares Share premium Retained earnings 45,900 10,800 4,500 61,200 1,800 10,800 Daughter 9,000 Add 54,900 10,800 6,300 72,000 (1,800) Eliminate (Dr)/Cr (9,000) 45,900 10,800 4,500 61,200 CBS

Daughter Cash Other net assets

Investment in 16,200 20,000 25,000 55,800 10,800 10,800

16,200 20,000 35,800 72,000

10,800 (1,200) 1,200 Q 4,200 20,000 37,000 61,200

Revaluation increase

Note: 1. The net assets in the CBS will be increased by 1,200. 2. The fair value of the shares issued (16,200) exceeded the fair value of the net assets acquired (12,000). This difference on consolidation will be reported as goodwill and reviewed for impairment.

201 Pearson Education Limited 2006

Barry Elliott and Jamie Elliott: Financial Accounting and Reporting (tenth edition) Instructors Manual

Chapter 21: Question 4 Parent Ltd


Parent Ltd balance sheet as at 1 January 20X7 Ordinary Shares of 1 each Retained earnings Investment in Daughter Ltd Cash [20,000 6,000] Other net assets 40,500 4,500 45,000 6,000 14,000 25,000 45,000

Note: The investment is shown as its fair value of 6,000 and the cash has been reduced by consideration.
Consolidated balance sheet as at January 20X7 Parent Ordinary shares Retained earnings 40,500 4,500 55,800 Investment in Daughter Cash Other net assets 6,000 14,000 25,000 55,800 2,000 8,800 10,800 Daughter 9,000 1,800 10,800 Add 49,500 6,300 55,800 6,000 16,000 33,800 55,800 ___ Q 10,800 Eliminate (Dr)/Cr (9,000) (1,800) 40,500 4,500 45,000 (4,800) 16,000 33,800 45.000 CBS

Note: Because the cash paid was less than the value of the net assets acquired, there was a credit difference on consolidation i.e. negative goodwill which will be credited to the retained earnings.

202 Pearson Education Limited 2006

Barry Elliott and Jamie Elliott: Financial Accounting and Reporting (tenth edition) Instructors Manual

Chapter 21: Question 5 Parent Ltd


Parent Ltd balance sheet as at 1 January 20X7 Ordinary Shares of 1 each Retained earnings Investment in Daughter Ltd Cash [20,000 9,000] Other net assets 40,500 4,500 45,000 9,000 11,000 25,000 45,000

Note: The investment is shown as its fair value of 9,000 and the cash has been reduced by consideration.
Consolidated balance sheet as at January 20X7 Parent Ordinary shares Retained earnings Minority interest 40,500 4,500 Daughter 9,000 1,800 Add 49,500 6,300 Eliminate (Dr)/Cr (6,750) a (2,250) b (1,350) a (450) b 2,250 b 450 b 45,000 Investment in Daughter Cash Other net assets 9,000 11,000 25,000 45,000 2,000 8,800 10,800 9,000 13,000 33,800 55,800 ___ Q 6,750 a 1,350 a 900 13,000 33,800 47,700 10,800 55,800 2,700 47,700 4,500 40,500 CBS

Note: Because the cash paid was more than the value of the net assets acquired, there was a debit difference on consolidation of 900. (a) represents the elimination of the shares and reserves of the company acquired against the investment in the company acquired. (b) Represents the transfer to the minority shareholders their 25% interest in the net assets of 10,8700 in Daughter Ltd.

203 Pearson Education Limited 2006

Barry Elliott and Jamie Elliott: Financial Accounting and Reporting (tenth edition) Instructors Manual

Chapter 21: Question 6


(a)
Property Plant and equipment Current assets Inventory Trade receivables Cash Current liabilities Trade payables Income tax Net current assets 63,700 6,440 70,140 67,573 123.293 Share capital Retained earnings 56,000 67,293 123.293 Reserves at date of acquisition Investment Less shares Goodwill Reserves Step 1: Calculate the % ownership Proposed dividend in CBS Proposed dividend in Parent Minority dividend Dividend receivable by Parent Step 2: Calculate the retained earnings balance Consolidated balance Less Parent 151,200 50,400 100,800 85,680 15.120 51,324 22,829 63,500 137,713

Balance sheet as at 30 June 20X2


43,400 12,320

11,760 11,200 560 5,040 10% 90%

79,884 35,280 44,604

204 Pearson Education Limited 2006

Barry Elliott and Jamie Elliott: Financial Accounting and Reporting (tenth edition) Instructors Manual

Add Profit on stock (4,200 3,360) Add Minority interest (10% of 50,493 or 1/9 of 45,444)) Add Pre-acquisition Parent (90% o 16,800) Minority (10% of 16,800) Subsidiary retained earnings

840 45,444 5,049 15,120 1,680 67,293

Step 3: Reconcile the minority interest Shares Retained earnings post-acquisition Retained earnings pre-acquisition

5,600 5,049 1,680 12,329

Worksheet
Non-current assets Group Property Plant Current assets Inventory Receivables Cash Current liabilities Payables Income tax 140,420 27,160 80,920 20,720 59,500 6,440 4,200 63,700 6,440 121,604 70,429 24,360 71,120 51,800 50,484 18,629 24,360 840 4,200 39,200 51,324 22,829 63,560 127,400 62,720 Parent 84,000 50,400 Subsidiary 43,400 12,320 Adjustment 43,400 12,320

205 Pearson Education Limited 2006

Barry Elliott and Jamie Elliott: Financial Accounting and Reporting (tenth edition) Instructors Manual

Chapter 21: Question 7 Rouge plc


Rouge plc Balance Sheet as at 1 January 20X0 ASSETS Non-current assets Property, plant and equipment [100 + 60] Goodwill [132 100] Current assets [80 + 70] Common 10 shares Retained earnings Share capital and reserves Current liabilities 160 32 150 342 200 52 252 90 342 Million

206 Pearson Education Limited 2006

Barry Elliott and Jamie Elliott: Financial Accounting and Reporting (tenth edition) Instructors Manual

Chapter 21: Question 8 Ham plc


(a)
Ham plc Balance sheet as at January 20X0 ASSETS Non-current assets Property, plant and equipment [250 + 100] Goodwill [90 110] Current assets [100 + 70] 350 (20) 170 500 Common 5 shares Retained earnings Share capital and reserves Current liabilities 200 160 360 140 500 000

(b)

See discussion in chapter.

Following recent changes in the treatment of goodwill, negative goodwill will be taken to income statement immediately.

207 Pearson Education Limited 2006

Barry Elliott and Jamie Elliott: Financial Accounting and Reporting (tenth edition) Instructors Manual

Chapter 21: Question 9 Berlin plc


Berlin plc Balance sheet as at 1 January 20X0 (a) Cash acquisition ASSETS Non-current assets Property, plant and equipment Investment in Hanover Current assets 250 100 50 400 Common 5 shares Additional paid-in capital Retained earnings Share capital and reserves Current liabilities 200 80 280 120 400 250 100 150 500 250 50 80 380 120 500 000 (b) Share exchange 000

208 Pearson Education Limited 2006

Barry Elliott and Jamie Elliott: Financial Accounting and Reporting (tenth edition) Instructors Manual

Chapter 21: Question 10 Bleu plc


Bleu plc Balance sheet as at January 20X0 ASSETS Non-current assets Property, plant and equipment [150 + 120] Goodwill [210 (80% 180) ] Current assets [108 + 105] 270 66 213 549 Common 10 shares Retained earnings Share capital and reserves Minority interest [20% 180] Current liabilities [90 + 45] 300 78 378 36 135 549 Million

209 Pearson Education Limited 2006

Barry Elliott and Jamie Elliott: Financial Accounting and Reporting (tenth edition) Instructors Manual

Chapter 21: Question 11 Base plc


Base plc Balance sheet as at January 20X0 ASSETS Non-current assets Property, plant and equipment [250 + 120] Goodwill [90 (60% 110) + (40% 20)] Current assets [100 + 70] Total assets Common 5 shares Retained earnings Share capital and reserves Minority interest [(40% 110) + (40% 20)] Current liabilities [80 + 60] Total equity and liabilities 370 12 170 552 200 160 360 52 140 552 000

210 Pearson Education Limited 2006

Barry Elliott and Jamie Elliott: Financial Accounting and Reporting (tenth edition) Instructors Manual

CHAPTER 22

Chapter 22: Question 1 Sweden


Sweden Balance Sheet as at 31 December 20X1 ASSETS Non-current assets Property, plant and equipment Goodwill Current assets [160 + 140] Total assets Common Kr10 shares Revaluation reserve Retained earnings [104 +10 2] Share capital and reserves Current liabilities [100 + 60] Total equity and liabilities [264 + 120] [200 (110 + 10 + 70) 2] 384 8 300 692 400 20 112 532 160 692 Krm

211 Pearson Education Limited 2006

Barry Elliott and Jamie Elliott: Financial Accounting and Reporting (tenth edition) Instructors Manual

Chapter 22: Question 2 Summer plc


Summer plc Balance Sheet as at 31 December 20X1 ASSETS Non-current assets Property, plant and equipment [200 + 200] Goodwill [141 60% (20 + 35 + 160) 1] Current assets [100 + 140] 400 11 240 651 Common 5 shares Additional paid-in capital Retained earnings [161 + 60% (40 35) 1] Share capital and reserves Minority interest [40% 220] Current liabilities [80 + 120] 175 25 163 363 88 200 651 000

212 Pearson Education Limited 2006

Barry Elliott and Jamie Elliott: Financial Accounting and Reporting (tenth edition) Instructors Manual

Chapter 22: Question 3 Gold plc


Gold plc Balance Sheet as at 31 December 20X1 ASSETS Non-current assets Fixed assets, including land [82,300 + 108,550 + 3,000] Goodwill Current assets Inventories [23,200 +10,000 300] Other current assets [5,000 + 7,500] Total assets Common 10 shares Preferred shares Additional paid-in capital Retained earnings [(75,000 + 75% (21,200 16,000)) 300 310)] Share capital and reserves Minority interest (Note 2) 78,290 148,290 26,950 26,500 32,900 12,500 240,490 55,000 10,000 5,000 (Note 1) 193,850 1,240

Non-current liabilities [12,500 +14,000] Current liabilities Bond interest payable [625 + 700] Other current liabilities [18,550 + 18,875] 1,325 37,425

38,750 240,490

Note 1:

Goodwill 46,000 18,000 6,000 3,500 27,500 75% 3,600 75% 3,000 75% 16,000 2,700 2,250 12,000 44,450 1,550

Investment in Silver Acquired 75% 24,000 30% 20,000 20% 17,500

Goodwill

213 Pearson Education Limited 2006

Barry Elliott and Jamie Elliott: Financial Accounting and Reporting (tenth edition) Instructors Manual

Impairment @ 20% Goodwill at 31.12.201 Note 2: Minority interest = 1,550 310 = 1,240

= 310

25% 24,000 70% 20,000 25% 3,600 25% 3,000 25% 21,200 6,000 14,000 900 750 5,300 26,950

214 Pearson Education Limited 2006

Barry Elliott and Jamie Elliott: Financial Accounting and Reporting (tenth edition) Instructors Manual

CHAPTER 23

Chapter 23: Question 1 Bill plc


Bill SA Consolidated Income Statement for the year ended 31 December 20X1

Sales [300,000 + 180,000 12,000] Cost of sales [90,000 + 90,000 12,000 + 2,000] Gross profit Expenses [88,623 + 60,000] Impairment of goodwill Profit before taxation Taxation [21,006 + 9,000] Profit after taxation Minority interest [(20% (21,000 4,500)) + (90% x 4,500)] Profit attributable to the group Dividend paid Retained profit for the year Retained profit brought forward [104,004 + 80% (81,000 45,000)] Retained profit carried forward 132,804 181,825 468,000 170,000 298,000 148,623 3,000 146,377 30,006 116,371 7,350 109,021 60,000 49,021

215 Pearson Education Limited 2006

Barry Elliott and Jamie Elliott: Financial Accounting and Reporting (tenth edition) Instructors Manual

Chapter 23: Question 2 Morn Ltd


Morn Ltd Consolidated Income Statement for the year ended 31 December 20X1 Gross profit [360,000 + 180,000)] Expenses [120,000 + 110,000] Profit before taxation Taxation [69,000 + 18,000] Profit after taxation Minority interest [10% 52,000] Profit after taxation belonging to the group Dividends paid Retained profit for the year 540,000 230,000 310,000 87,000 223,000 5,200 217,800 120,000 97,800

216 Pearson Education Limited 2006

Barry Elliott and Jamie Elliott: Financial Accounting and Reporting (tenth edition) Instructors Manual

Chapter 23: Question 3 River plc


River A/S Consolidated Income Statement for the year ended 31 December 20X1 Sales [100,000 + [(9/12 60,000)] Cost of sales [30,000 +[(9/12 30,000]) Gross profit Expenses [20,541 + (9/12 15,000)] Interest payable on 5% bonds [9/12 (5,000 500)] Impairment of goodwill Profit before taxation Taxation [7,002 + (9/12 3,000)] Profit after taxation Minority Interest [10% 7,000 9/12] Profit after taxation belonging to the group Dividends paid Profit and loss account balance brought forward from previous years 34,668 58,225 145,000 52,500 92,500 31,791 3,375 4,000 53,334 9,252 44,082 525 43,557 20,000 23,557

217 Pearson Education Limited 2006

Barry Elliott and Jamie Elliott: Financial Accounting and Reporting (tenth edition) Instructors Manual

Chapter 23: Question 4 Mars plc


Balance Sheet as at 31 December 20X2

Assets
Non-current assets [330,000 + 157,500] Goodwill Current assets Inventories [(225,000 + 67,500) 3,000] Trade receivables [180,000 + 90,000] Bank [36,000 + 18,000] Total assets Equity and liabilities Capital and reserves Issued capital General reserve [245,000 + 10,800] Retained earnings [222,000 + 44,000] Minority interest Current liabilities Trade payables [283,500 + 40,500] Taxation [31,500 + 13,500] 324,000 45,000 369,000 1,138,100 W1: Cancel inter-company balances 196,000 255,800 266,000 717,800 51,300 289,500 270,000 54,000 613,500 1,138,100 487,500 37,100

Current accounts of 22,500 Dividends receivable in Mars of 9,000 cancels with 9,000 of the dividends payable in Jupiter, leaving 2,250 payable to the minority interest.
Consolidated Income Statement for the year ending 31 December 20X2

Sales [1,440,000 + 270,000 18,000] Cost of sales [1,045,000 + 135,000 18,000 + 3,000] Gross profit Expenses [123,500 + 90,000] Profit before tax Taxation [31,500 + 13,500] 1,692,000 1,165,000 527,000 213,500 313,500 45,000

218 Pearson Education Limited 2006

Barry Elliott and Jamie Elliott: Financial Accounting and Reporting (tenth edition) Instructors Manual

Profit after tax Minority interest [20% 31,500] Dividend paid Retained profit Accumulated profit brought forward [(156,000 + 80% (114,750 80,000))

268,500 6,300 262,200 180,000 82,200 183,800 266,000

W2:

Goodwill 187,500 72,000 64,000 14,400 150,400 37,100

Investment in Jupiter 1 Ordinary shares [80% 90,000] Accumulated profits [80% 80,000] General Reserve [80% 18,000] Goodwill W3: Unrealised profit on inter-company sales

50/150 18,000 = 6,000. Only half the stock is unsold at the year end so 6,000/2 is the provision required against the closing stock figure.
W4: The income statement of Jupiter Balance at 31/12/20X2 as per the balance sheet Pre-acquisition profit held by Mars Minority interest [20% 135,000] 64,000 27,000 91,000 44,000 W5: The income statement of Mars 135,000

Balance at 31/12/20X2 as per the balance sheet Less: Provision for unrealised profit 225,000 3,000 222,000

219 Pearson Education Limited 2006

Barry Elliott and Jamie Elliott: Financial Accounting and Reporting (tenth edition) Instructors Manual

W6:

The minority interest

20% 90,000 Shares 20% 31,500 General reserve 20% 135,000

18,000 6,300 27,000 51,300

W7:

Jupiter general reserve

Balance at 31/12/20X2 as per balance sheet Less: Mars share of pre-acquisition 80% 18,000 = Minority interest 20% 31,500

31,500 (14,400) (6,300) 10,800

220 Pearson Education Limited 2006

Barry Elliott and Jamie Elliott: Financial Accounting and Reporting (tenth edition) Instructors Manual

Chapter 23: Question 5 Red Ltd


Consolidated Balance Sheet as at 31 December X2 Assets Non-current assets Tangible [120,000 + 70,000] Intangible Goodwill (W1) Current assets Inventories [100,000 + 30,000 1,500] (W2) Trade receivables [80,000 + 40,000] Bank [16,000 + 8,000] Total assets Equity and liabilities Capital and reserves Issued capital General reserve [20,000 + (75% (14,000 8,000))] Retained earnings [(100,000 + (75% (60,000 30,000) 1,500))] Minority interest 25% (114,000 1,500) 121,375 321,875 28,125 176,000 24,500 128,500 120,000 24,000 272,500 514,000 190,000 51,500 $ $

Current liabilities Creditors [125,996 + 18,000] Taxation [14,004 + 6,000] 143,996 20,004 164,000 514,000 Income Statement for the year ending 31 December X2

Sales [200,000 + 120,000 12,000] 308,000 109,500 198,500 (99,082) 99,418 20,004 79,414 3,125 76,289 Dividend paid 40,000 Cost of sales [60,000 + 60,000 12,000 + 1,500 Gross profit Expenses [59,082 + 40,000] Profit before tax Taxation [14,004 + 6,000] Profit after tax Minority interest [25% (14,000 1,500]

221 Pearson Education Limited 2006

Barry Elliott and Jamie Elliott: Financial Accounting and Reporting (tenth edition) Instructors Manual

Retained profit Accumulated profit brought forward (W3)

36,289 85,086 121,375

W1:

Goodwill $

Investment in Pink $1 Common shares [75% 40,000] Accumulated profits [75% 30,000] General Reserve [75% 8,000] 30,000 22,500 6,000

110,000

58,500 Goodwill 51,500

W2:

Unrealised profit on inter-company sales

Mark-up = 9,000 1/3 = $3,000 Only half the stock is unsold at the year end so $3,000/2 is the provision required against the closing stock figure = $1,500
W3: Red Pink [75% (51,000 30,000)] 69,336 15,750 85,086

222 Pearson Education Limited 2006

Barry Elliott and Jamie Elliott: Financial Accounting and Reporting (tenth edition) Instructors Manual

Chapter 23: Question 6 Try plc


Try 000 Profit before tax Tax Minority interest [25%] 38 Dividends 20 80 42 38 Hard 000 56 28.60 27.40 6.85 20.55 . CPL 000 136.00 70.60 65.40 6.85 58.55 20.00

Carried forward

18

20.55

38.55

223 Pearson Education Limited 2006

Barry Elliott and Jamie Elliott: Financial Accounting and Reporting (tenth edition) Instructors Manual

Chapter 23: Question 7 Mother plc


(a) Consolidated income statement for the year ended 31 December 20x6
Mother 000 Profit before tax Less Intra-company dividend Income tax Profit after tax Intra-group dividend Transfer to general reserve Dividends Retained Balance b/f at 1.1.20x6 300 (20) (120) 160 20 180 (30) (50) 100 50 150 Consolidated balance sheet as at 31 December 20x6 Mother 000 Non-current assets Investment in Daughter plc Net current assets 300 200 280 780 Share capital General reserve Retained earnings 500 130 150 780 110 260 200 20 40 260 390 840 500 150 190 840 Daughter 000 150 CBS 000 450 30 10 40 (30) 70 (20) 50 (20) 230 (50) (50) 130 60 190 Daughter 000 100 CPL 000 400 (20) (150) 230

224 Pearson Education Limited 2006

Barry Elliott and Jamie Elliott: Financial Accounting and Reporting (tenth edition) Instructors Manual

(b)

The consolidated balance sheet:


The parent company has 100% control of the subsidiary. The subsidiary was acquired at the date of incorporation when the net assets were equal to share capital with both stated at 200,000, there was therefore no goodwill. The CBS is formed by the aggregation of the assets and the reserves. Only the share capital of the parent company is included.

The consolidated income statement:


Items are aggregated except for dividend. Intra-group dividends are cancelled. Only the dividend of the parent company is shown. Reserves are aggregated because they are post-acquisition

225 Pearson Education Limited 2006

Barry Elliott and Jamie Elliott: Financial Accounting and Reporting (tenth edition) Instructors Manual

Chapter 23: Question 8 Mother plc


(a) Consolidated income statement for the year ended 31 December 20x6
Mother 000 Profit before tax Less Intra-company dividend Income tax Profit after tax Less minority interest 164 Intra-group dividend Transfer to general reserve Dividends Retained Balance b/f at 1.1.20x6 16 180 (30) (50) 100 50 150 80% of 10 24 8 80% of 40 32 300 (16) (120) 164 (30) 70 20% of 70 (14) 56 (16) 50 80% of 20 (16) 220 (46) (50) 124 58 182 Daughter 000 100 CPL 000 400 (16) (150) 234 (14) 220

Consolidated balance sheet as at 31 December 20x6


Mother 000 Non-current assets Goodwill (200,000-160,000) Investment in Daughter plc Net current assets 200 280 780 Share capital General reserve Retained earnings Minority interest 20% of 260,000 780 260 500 130 150 110 260 200 20 40 390 880 500 146 182 52 880 300 Daughter 000 150 CBS 000 450 40

226 Pearson Education Limited 2006

Barry Elliott and Jamie Elliott: Financial Accounting and Reporting (tenth edition) Instructors Manual

(b)

The consolidated balance sheet:


The parent company holds 80% of the shares There is a 20% minority interest In CPL it is 20% of post tax profit In the CBS it is 20% of the closing net assets of 260,000

The intragroup dividend is 80% of the Daughter plc dividend i.e. 80% of 20,000 The retained earnings is then 80% of the closing balance i.e. 80% of 40,000. Goodwill has arisen because Mother paid more than the fair value of the net assets i.e. 200,000 for net assets of 160,000. The CBS is formed by the aggregation of the assets and the reserves. Only the share capital of the parent company is included.

The consolidated income statement:

Items are aggregated except for dividend. Intra-group dividends are cancelled. Only the dividend of the parent company is shown. Reserves are aggregated because they are post-acquisition

227 Pearson Education Limited 2006

Barry Elliott and Jamie Elliott: Financial Accounting and Reporting (tenth edition) Instructors Manual

Chapter 23: Question 9 Mother plc


(a) Consolidated income statement for the year ended 31 December 20X6
Mother 000 Retained Balance b/f at 1.1.20X6 150 50 150 (Note 1) Daughter 000 30 5 35 CPL 000 130 55 185

Consolidated balance sheet as at 31 December 20X6 000 Non-current assets Net current assets 450 390 840 Share capital Negative goodwill [Cost 200,000 - net assets 220,000] General reserve Retained earnings 20 135 185 840 (Note 2) [only includes post-acquisition] 500

(b)
Note 1. The group share of Daughter's retained profit is 100% of the post-acquisition profits i.e. 100% of 40,000 5,000 that existed at 1.1.20X2. Note 2. The price paid is less than the fair value of the net assets. This can be attributed either to a bargain purchase perhaps because the vendor needs to achieve a quick sale or to expectation of future losses whereby the purchase price has been reduced to take account of future costs, such as reorganisation costs, or losses that do not represent identifiable liabilities at the balance sheet date.

228 Pearson Education Limited 2006

Barry Elliott and Jamie Elliott: Financial Accounting and Reporting (tenth edition) Instructors Manual

Chapter 23: Question 10 Mother plc


(a) Consolidated income statement for the year ended 31 December 20X6
Mother 000 Retained Bal b/f at 1.1.20X6 100 50 150 [80% 5] (Note 1) Daughter 000 24 4 28 CPL 000 124 54 178

Note 1. The group share of Daughter's retained profit is 80% of the post-acquisition profits i.e. 80% of 40,000 5,000 that existed at 1.1.20X2. Consolidated balance sheet as at 31 December 20X6 000 Goodwill [200,000 80% of 220,000] Non-current assets Net current assets 24 450 390 864 Share capital General reserve [130,000 + 80% of 5,000] Income statement Minority interest [20% of 260,000] 500 134 178 52 864

(b) In this case a minority interest is recorded representing the minority's 20% interest in the net
assets at the balance sheet date which are under the control of the majority shareholder.

229 Pearson Education Limited 2006

Barry Elliott and Jamie Elliott: Financial Accounting and Reporting (tenth edition) Instructors Manual

Chapter 23: Question 11 Tailor Ltd


Plant [207,900184,900] 23,000 12,000 7,300 2,000 9,300 20,300 Less: Payables Overdraft [58,000 56,000] [6,0000] 2,000 6,000 8,000 12.300 Capital Reserves 7,000 5,300 12,300 Reconcile the reserves: (1) Cost of control Cash for investment - per balance sheet Capital reserve - per Note 2 in question Less: Shares Reserves as the difference Reserves of 3,600 is a 90% interest (i.e. 6300/7000 100) Therefore 100% interest was 4,000 (2) Calculate distributable profits Minority interest per consolidated balance sheet Less Shares Post-dividend reserve Dividend (1/9 of 900 in parent balance sheet) Pre dividend reserve Less Pre-acquisition (1/9 of 3,600 from Working 1 above) 1,200 (700) 500 100 600 (400) 200 10% is 200, therefore 100% is 2,000 9,100 800 9,900 (6,300) 3.600 11,000

Depreciation [44,00032,000] Inventory [70,000 63,000 + 300 unrealised profit] Receivables [25,00023,000]

230 Pearson Education Limited 2006

Barry Elliott and Jamie Elliott: Financial Accounting and Reporting (tenth edition) Instructors Manual

(3)

Calculate the reserve of subsidiary Profits were from (2) above Add profit on stock (inter-group) Less dividend Retained Balance b/f Balance c/f 2,000 300 2,300 (1,000) 1,300 4,000 5.300

(4)

Treatment of reserves on consolidation Balance Less: Inventory adjustment Minority interest Pre-acquisition Post-acquisition 300 500 3,600 900 5.300 5,300

231 Pearson Education Limited 2006

Barry Elliott and Jamie Elliott: Financial Accounting and Reporting (tenth edition) Instructors Manual

Chapter 23: Question 12 H and S Ltd


(a) % of S Ltd owned by H Ltd Derived from the minority interest figures Income statement 170/680 100 Balance sheet 555/2,220 100 = = = 75% 25% 25%

(b)

Inter-company sales = 500 The amount eliminated in the Income Statement Parent 4,000 + subsidiary 2,200 - group 5,700 Inventory unrealised profit = 45 Parent 410 + subsidiary 420 - group 785 Inter-company receivables and payables offset/eliminated Receivables: Parent 535 + subsidiary 220 - group 595 = 160 Payables: Parent (300) + subsidiary (260) - group 355 = 225 S Ltd retained earnings on acquisition = 960 Comprising: Cost Less Share capital Goodwill Reserves attributable to 75% Total reserves 720/75 100

(c)

(d)

(e)

1,700 570 410 980 720 960

232 Pearson Education Limited 2006

Barry Elliott and Jamie Elliott: Financial Accounting and Reporting (tenth edition) Instructors Manual

CHAPTER 24

Chapter 24: Question 1 Swish


(a)
Sales

Income Statement for the year ended 31 December 20X3


[300,000 + 160,000 16,000] [90,000 + 80,000 16,000 + 3,200] 444,000 157,200 286,800 145,000 141,800

Cost of Sales Expenses

[95,000 + 50,000]

Group profit before taxation Share of Associated Company profits [25% 30,000] Profit before taxation Taxation Group [30,000 + 7,000] Associate [25% 8,000] 37,000 2,000

7,500 149,300 39,000 110,300 2,300 108,000 40,000 68,000 137,800 205,800

Minority Interest (10% 23,000) Dividends paid Retained profit for the year Retained earnings brought forward [94,000 + ( 90% 47,000) + (25% 6,000)]

(b)

Consolidated Balance Sheet as at 31 December 20X3


Cost Depn 500,000 270,000 Net 17,600 230,000 46,000 293,600

Non-current assets Intangible: Goodwill in subsidiary Tangible [120,000 + 110,000] [18,000 + 28,000] Investment in Associate Current Assets Inventories [120,000 + (60,000 3,200)] [130,000 + 70,000]

176,800 200,000

Trade receivables

233 Pearson Education Limited 2006

Barry Elliott and Jamie Elliott: Financial Accounting and Reporting (tenth edition) Instructors Manual

Current account Handle (Note 1) Bank [24,000 + 7,000]

3,000 31,000 410,800

Current liabilities Trade payables Taxation payable [132,000 + 25,000] 157,000 37,000 194,000 Financed by 1 Common shares General reserve [30,000 + 3,600 + (25% 4,000] (Note 2) Retained earnings [150,000 3,200 + 54,000 + (25% 20,000) (Note 2) 205,800 490,400 Minority interest 20,000 510,400 250,000 34,600 216,800 510,400

Notes 1. The inter-company current account balance with the associated company has not been cancelled because the associated company is not a member of the group. 2. The groups share of the retained earnings and general reserve is calculated on the postacquisition accumulated profits and general reserve of Handle, i.e.
General reserve per Handle balance sheet is Pre-acquisition (see question) Post-acquisition Retained earnings per Handle balance sheet Pre-acquisition (see question) Post-acquisition 12,000 8,000 4,000 50,000 30,000 20,000

3. Goodwill in Broom
Cost of investment Less 90% of share capital General reserve Retained earnings 90% Goodwill 140,000 60,000 16,000 60,000 136,000 122,400 17,600

234 Pearson Education Limited 2006

Barry Elliott and Jamie Elliott: Financial Accounting and Reporting (tenth edition) Instructors Manual

4. Unrealised profit in Inventory


Swish sold Broom Remaining unsold at year end Profit element 25% 16,000 12,800 3,200

235 Pearson Education Limited 2006

Barry Elliott and Jamie Elliott: Financial Accounting and Reporting (tenth edition) Instructors Manual

Chapter 24: Question 2 Stop Group


(a)
Sales Cost of sales Expenses Share of profits of Associate Taxation Group Associate [11,250 30% 0.75] [43,750 30% 0.75] 25,000 2,531 27,531 82,563 Dividends paid Retained profit for the year Balance brought forward 50,000 32,563 112,500 145,063

Consolidated income statement for year to 31 December 20X1

375,000 125,000 250,000 149,750 100,250 9,844 110,094

(b)

Consolidated Balance Sheet at 31 December 20X1

375,000 125,000 250,000 7,000 [204,375 30%] 61,313

Fixed assets at cost Depreciation Goodwill Investment in Start Ltd Current assets Inventory Trade receivables Current a/c Start Ltd Bank 160,000 165,000 15,000 12,500 352,500

236 Pearson Education Limited 2006

Barry Elliott and Jamie Elliott: Financial Accounting and Reporting (tenth edition) Instructors Manual

Current liabilities Trade payables Taxation 187,500 25,000 212,500 140,000 458,313 Financed by Ordinary shares of 1 General reserves Retained earnings [31,500 + (2,500 30%)] 281,000 32,250 145,063 458,313

Working 1
Investment in Start [63,250 750] (see Note) Shares acquired Retained earnings [102,500 30%] General reserve [20,000 30%] Goodwill 18,750 30,750 6,000 55,500 7,000 62,500

Note: The 750 which has correctly been credited to Stops investment in Start represents the share of the dividend receivable out of the profits of Start before it became an associate i.e. 10,000 30% 3/12 = 750. This amount is not a profit made by the group and must not therefore be in the group profit and loss. In effect this adjustment restores the net assets purchased on acquisition to their full amount.

237 Pearson Education Limited 2006

Barry Elliott and Jamie Elliott: Financial Accounting and Reporting (tenth edition) Instructors Manual

Chapter 24: Question 3 Ant Co


(a) Consolidated Income Statement for the year ended 31 December 20X9
$
Sales [225,000 + 120,000 12,000] [67,500 + 60,000 118,200 214,800 Expenses [70,500 + 37,500] 108,000 106,800 Group profit before taxation Share of Associated Company profits [25% 22,500] Profit before taxation Taxation Group [22,500 + 5,250] Associate [25% 6,000] Minority interest Dividends paid Retained profit for the year Retained earnings brought forward [70,500 + (80% 35,250*) + (25% 4,500*) 99,825 149,550 * Post-acquisition profits brought forward [20% 17,250] 27,750 1,500 29,250 83,175 3,450 79,725 30,000 49,725 5,625 112,425 12,000 + 2,700] 333,000 Cost of sales

(b)

Consolidated Balance Sheet as at 31 December 20X9


Cost $ Depn $ Net $ 8,400 375,000 202,500 172,500 34,500 215,400

Non-current assets Intangible: Goodwill in subsidiary Tangible: [90,000 + 82,500] Investment in Associate Current assets Inventories [105,000 + 45,000 2,700] 147,300 [21,000 + 13,500]

238 Pearson Education Limited 2006

Barry Elliott and Jamie Elliott: Financial Accounting and Reporting (tenth edition) Instructors Manual

Trade receivables

[98,250 + 52,500]

150,750 2,250 22,500 322,800

Current account Nit (Note 1) Bank [17,750 + 5,250] Current liabilities Trade payables Taxation payable [99,000 + 18,750] 117,750 27,750

145,500 Financed by $1 common shares General reserve [22,500 + 2,400 + (25% 3,000)] (Note 2) Retained earnings

177,300 392,700 187,500 25,650 149,550 362,700

Minority interest (20% 150,000)

30,000 392,700

Notes 1. The inter-company current account balance with the Associated company has not been cancelled because the Associated company is not a member of the group. 2. The groups share of the Associated retained earnings and General Reserve is calculated on the post-acquisition retained earnings and General reserve of Nit, i.e.
General reserve per Nit balance sheet is Pre-acquisition (see question) Post-acquisition Retained earnings per Nit balance sheet Pre-acquisition (see question) Post-acquisition 9,000 6,000 3,000 37,500 22,500 15,000

239 Pearson Education Limited 2006

Barry Elliott and Jamie Elliott: Financial Accounting and Reporting (tenth edition) Instructors Manual

Chapter 24: Question 4 Twist plc


(a)
Sales Cost of sales Expenses Share of profits of Associate Taxation Group Associate [14,000 30% 0.25] [74,000 30% 0.25] 30,000 1,050 31,050 103,500 Minority interest Attributable to the shareholders of the parent Dividends paid Retained earnings brought forward 10,000 93,500 60,000 33,500 136,000 169,500

Consolidated Income Statement for year to 31 December 20X3


450,000 150,000 300,000 171,000 129,000 5,550 134,550

(b)

Consolidated Balance Sheet at 31 December 20X3


450,000 150,000 300,000

Property, plant and equipment Depreciation

Investment in Turn [(244,000 30%) + 3,300] (W1) Current assets Inventories Trade receivables Current a/c Turn Bank 180,000 207,000 18,000 18,000 423,000

76,500

240 Pearson Education Limited 2006

Barry Elliott and Jamie Elliott: Financial Accounting and Reporting (tenth edition) Instructors Manual

Current liabilities Trade payables Taxation 225,000 30,000 255,000 168,000 544,500 Financed by Common shares of 1 each General reserve Retained earnings [168,000 + (20,000 25% 30%)] Minority interest 200,000 37,000 169,500 406,500 138,000 544,500 Working 1 Investment in Turn Shares acquired Retained earnings [137,000 30%] General reserve [32,000 30%] Goodwill 21,000 41,100 9,600 71,700 3,300 75,000

241 Pearson Education Limited 2006

Barry Elliott and Jamie Elliott: Financial Accounting and Reporting (tenth edition) Instructors Manual

CHAPTER 25

Chapter 25: Question 1 Fry Ltd


The profit or loss on foreign exchange in these cases will be as follows:
American Name of account: Texas Inc Alamos Inc Chicago Inc Payable Foreign currency at exchange rate on date of initial transaction 15,385 Foreign currency at exchange rate on date of settlement Foreign currency at exchange rate on date of balance sheet Profit/(loss) on foreign exchange (1,282) 3,709 (2,640) (5,357) 5,357 000 000 $40,000 2.40 16,667 23,077 $30,000 2.40 12,500 $30,000 2.10 14,286 41,667 $80,000 2.30 34,783 $20,000 2.10 9,524 $90,000 2.10 42,847 $90,000 2.10 42,857 37,500 37,500 $40,000 2.60 Receivable $60,000 2.60 Creditor $100,000 2.40 bank Loan creditor $90,000 2.40 $90,000 2.40 Dollar bank account

242 Pearson Education Limited 2006

Barry Elliott and Jamie Elliott: Financial Accounting and Reporting (tenth edition) Instructors Manual

Chapter 25: Question 2 Walpole Ltd


(a) Translate the financial statements of Paris SA into sterling

Income statement Paris SA 000 Sales Purchases Other expenses Interest Taxation Opening inventories Closing inventories Depreciation Dividend paid Net profit carried forward 200,000 (90,000) (7,000) (3,000) (15,000) 85,000 (22,000) 12,000 (30,000) (10,000) 35,000 3.0 2.5 4.5 3.5 5.0 28,333 (8,800) 2,667 (8,571) (2,000) 11,629 Note 3 Note 4 Note 1 Note 2 Exchange rate 000 Translated

Notes 1 Sales, purchases and expenses have been translated at an average which is an approximation of the rate when they were originally recorded. This requirement of the IAS has been rather more loosely interpreted in the case of interest and taxation, which have been translated at the rates or approximate rates when they originally accrued. Under the wording of the IAS it might be more strictly correct if the taxation and interest were translated at the date they were first recorded in the books. Translated at the actual date of acquiring the inventories. The rate is that applicable to the date of revaluation rather than that at the date of acquiring the fixed assets. The closing rate has been taken as the actual rate in this case.

2 3 4

Balance sheet Non-current assets Current assets Inventories Receivables Cash 12,000 40,000 11,000 4.5 5.0 5.0 2,667 8,000 2,200 Note 5 Note 6 150,000 3.5 42,857 Note 3

243 Pearson Education Limited 2006

Barry Elliott and Jamie Elliott: Financial Accounting and Reporting (tenth edition) Instructors Manual

Current liabilities Payables Walpole Ltd Taxation Non-current assets bonds Total assets less liabilities Equity Share capital Additional paid-in capital Accumulated Profit [66,000 35,000] Revaluation reserve Profit for the year Loss on exchange Total equity 158,000 31,000 111,000 12,000 35,000 3.5 see above Balancing figure 30,229 70,229 3,428 11,629 (40,562) 44,724 Note 9 Note 10 Note 11 Note 12 Note 13 60,000 20,000 2.0 2.0 30,000 10,000 Note 7 Note 8 (18,000) (12,000) (15,000) (10,000) 158,000 5.0 5.0 5.0 5.0 (3,600) (2,400) (3,000) (2,000) 44,724

Notes 5 6 7 As inventories are non-monetary assets they are translated at actual. The receivables and all the following assets in the balance sheet are monetary items and therefore retranslated at the closing rate. Share capital issued should be translated at the date of acquiring the subsidiary or at the date of issue if later. At this stage of the process we are attempting to find the figure of profit or loss on exchange differences up to the beginning of the current year so that we can isolate the profit or loss on exchange in the current year. One way to do this is to split the share capital and reserves into the amount arising this year and the balance at the end of the previous year. In this question the balance at the end of the previous year can be calculated as 111,000 (see Note 10). The rest (profit for the year 35,000 and revaluation reserve 12,000) arose in the present year. If we can then translate the 111,000 into pounds sterling at the end of the previous year, we can eventually find this years profit or loss on exchange. In practice we could obtain the sterling equivalent of the 111,000 from the workings for the previous years consolidated accounts. The additional paid-in capital (share premium) is translated at the same rate as the shares to which it relates. The accumulated profit at the end of the previous year can be taken as the balancing figure after translating the 111,000. The accumulated profit will have been translated at many different rates over the years. In practice the translated figure would be available from the previous years consolidated accounts.

8 9

10 The 111,000 represents a mixture of monetary and non-monetary assets. The amount can be translated by applying the exchange rates used in the balance sheet in the previous year. This is calculated as follows:

244 Pearson Education Limited 2006

Barry Elliott and Jamie Elliott: Financial Accounting and Reporting (tenth edition) Instructors Manual

Non-current assets Depreciation Revaluation Opening inventory Balance = Monetary Liabilities at 31/12/20X1 Net assets at 31/12/20X1

150,000 30,000 180,000 (12,000) 168,000 22,000 190,000 (79,000) 111,000 3.5 2.0 2.5 84,000 8,800 92,800 (22,571) 70,229

11 The revaluation reserve is translated at the rate when the revaluation took place in the current year. 12 The income statement was translated to give the profit figure in sterling. 13 The loss on exchange differences is the balancing figure but can be found directly, as shown below. Calculating the exchange difference for the year (i) The monetary net liabilities at the beginning of the year of 79,000 (see Note 10 above) have been retranslated into sterling at 31 December 20X2 and a gain of 6,771 has been made since translation at the end of the previous year.
Opening net monetary liabilities: 79,000 @ opening rate Opening net monetary liabilities: 79,000 @ closing rate Gain 3.5 = 5.0 = 22,571 15,800 6,771

(ii) Any profit made during the year initially goes into monetary net assets. The translation of these at the year-end rate will give a profit in this case of the difference between the actual or average rate (as an approximation of the actual rate) used for translating the income statement items and the year-end rate used for translating monetary items in the balance sheet.
Translated at average rate at closing rate Loss 85,000 @ 3.0 = 28,333 85,000 @ 5.0 = 17,000 11,333

(iii) The fixed assets were acquired when the rate of exchange was 2 euros to the and this was the rate applied to them at 31 December 20X1. On 1 January 20X2 the fixed assets were revalued when the rate was 3.5 euros to the and this was therefore the rate applied to these fixed assets at 31 December 20X2. This produced a loss of 36,000 being (168,000/2.0) (168,000/3.5). Total loss on changing exchange rates (11,333 + 36,000 6,771) = (40,562)

245 Pearson Education Limited 2006

Barry Elliott and Jamie Elliott: Financial Accounting and Reporting (tenth edition) Instructors Manual

(b)

Prepare consolidated accounts Walpole

Balance Sheet as at 31 December 20X2


ASSETS Non-current assets Goodwill (W1) Other [94,950 + 42,857] Current assets (W2) Inventories [60,000 + 2,667] Receivables [59,600 + 8,000] Bank [11,000 + 2,200] Total assets Equity and liabilities Common share capital Additional paid-in capital Revaluation reserve [10,000 + 3,085] (W3) Retained earnings [67,000 3,334] (W4) Minority interest (W5) Non-current liabilities (W2) Bonds [40,000 + 2,000] Current liabilities Payables [45,000 + 3,600] Taxation [21,000 + 3,000] Total equity and liabilities W1: Goodwill Investment in Paris Common share in Paris [90% 30,000] Retained earnings [90% 5,000*] Additional paid-in capital [90% 10,000*] Goodwill = 2) 000 27,000 4,500 9,000 40,500 550 48,600 24,000 114,600 281,824 000 41,050 42,000 80,000 6,000 13,085 63,666 162,751 4,473 62,667 67,600 13,200 143,467 281,824 550 137,807 138,357 000

* Pre-acquisition retained earnings translated at the exchange rate on the day of acquisition (1

246 Pearson Education Limited 2006

Barry Elliott and Jamie Elliott: Financial Accounting and Reporting (tenth edition) Instructors Manual

W2: Cancel inter-company balances Current accounts of 2,400 W3: Paris SA revaluation reserve 000 Balance at 31/12/20X2 as per balance sheet Minority interest 10% 3,428 Consolidated balance sheet W4: Retained earnings of Paris 000 Balance at 31/12/20X2 as per the balance sheet [11,629 + 30,229 40,562 loss on exchange] Pre-acquisition profit Minority interest [10% 1,296] 4,500 130 4,630 (3,334) W5: The minority interest 000 Common shares [10% 30,000] Retained earnings [10% 1,296] Additional paid-in capital [10% (9,800 +10,000)] Revaluation reserve [10% 3,429] 3,000 130 1,000 343 4,473 1,296 3,428 343 3,085

The consolidated income statement for the year ended 31/12/20X2 includes the subsidiary figures using the exchange rates as shown in the question.
Sales [317,200 + (200,000/3)] Cost of sales [170,000 + (8,800 + 90,000/3 2,667)] Gross profit Depreciation [30,000 + 8,571] Expenses [15,000 + 7,000/3] Loss on foreign exchange Interest [6,000 + 3000/3] Profit before tax Taxation [21,000 + 15,000/3] Profit after tax Minority interest [10% (11,629 + Div 2,000 40,562)] Net profit for the year
247 Pearson Education Limited 2006

383,867 (206,133) 177,734 (38,571) (17,334) (40,562) (7,000) 74,267 26,000 48,267 2,693 50,960

Barry Elliott and Jamie Elliott: Financial Accounting and Reporting (tenth edition) Instructors Manual

Dividend paid Retained profit Retained earnings brought forward (W6)

(20,000) 30,960 32,706 63,666

Group statement of changes in equity for the year ended 31/12/20X2 This statement will appear as follows: Retained earnings brought forward (W6) Net profit for the year Dividend paid Accumulated profit carried forward W6: Group retained earnings brought forward at 1/1/20X2 Walpole [67,000 this year 57,000] Paris [90% (30,229 Pre-acquisition 5,000 (W1))] 10,000 (22,706) 32,706 32,706 50,960 (20,000) 63,666

248 Pearson Education Limited 2006

Barry Elliott and Jamie Elliott: Financial Accounting and Reporting (tenth edition) Instructors Manual

Chapter 25: Question 3 Paris SA


Walpole Question 3

(a)

Income statement
Paris SA Exchange rate See sol to Qn 2 Translated 11,629

As in solution 2 Balance sheet Net assets Share capital Additional paid in capital Retained earnings (66,000 35,000) Revaluation reserve Profit for the year Loss on exchange Total equity

35,000

158,000 60,000 20,000 31,000 111,000 12,000 35,000 158,000

5.0 2.0 2.0 Balancing figure 3.5 3.5 See above

31,600 30,000 10,000 (8,286) 31,714 3,428 11,629 (15,171) 31,600

Proof of the loss on translation


(i) Opening net assets 111,000 Closing net assets 111,000 (ii) Income statement 85,000 @ opening rate 3.5 @ closing rate 5.0 @ average rate 3.0 @ closing rate 5.0 (iii) Opening inventories 22,000 @ @ (iv) Closing inventories 12,000 @ @ 2.5 5.0 4.5 5.0 = = Loss = = Loss = = Profit = = Loss 31,714 22,200 (9,514) 28,333 17,000 (11,333) 8,800 4,400 4,400 2,667 2,400 (267)

249 Pearson Education Limited 2006

Barry Elliott and Jamie Elliott: Financial Accounting and Reporting (tenth edition) Instructors Manual

(v)

Depreciation

30,000

@ @

3.5 5.0 3.5 5.0

= = Profit = = Loss

8,571 6,000 2,571 3,428 2,400 (1,028) (15,171) (330)

(vi)

Revaluation Reserve 12,000

@ @

Total loss [ (9,514) + (11,333) +(267) + (1,028) 2,571 4,400] = Add loss on retranslation of goodwill 550 (@2) 220 (@5) =

(b) Prepare consolidated accounts Walpole


Balance sheet as at 31 December 20x2
Non-current assets Goodwill Other (94,950 + 150,000/5) Current assets Inventories (60,000 + 12,000/5) Receivables (59,600 + 8,000) Bank Total assets Equity and liabilities Common share capital Additional paid in capital Revaluation reserve (10,000 + 3,085) (W3) Retained earnings Minority interest Non-current liabilities Bonds (40,000+2,000) Current liabilities Payables (45,000 + 3,600) Taxation (21,000 + 3,000) Total equity and liabilities 48,600 24,000 114,600 268,370 (67,000 15,475) (W4) (W5) (W2) 42,000 80,000 6,000 13,085 51,525 150,610 3,160 (11,000 + 2,200) (W2) 62,400 67,600 13,200 143,200 268,370 (W1) 220 124,950 125,170

250 Pearson Education Limited 2006

Barry Elliott and Jamie Elliott: Financial Accounting and Reporting (tenth edition) Instructors Manual

W1 Goodwill Investment in Paris Common shares in Paris (90% of 30,000) Retained earnings (90% of 5,000) Additional paid in capital (90% of 10,000* Goodwill 27,000 4,500 9,000 40,500 550 41,050

Pre-acquisition retained earnings translated at the exchange rate on the day of acquisition (1 = 2) Restated at the rate on 31/12/20X2 as goodwill is treated as the asset of the subsidiary. Restated at 5.0 = 330.

W2 Cancel inter-company balances Current accounts of 2,400 W3 Paris SA revaluation reserve 000 Balance at 31.12.20x2 per balance sheet Minority interest 10% of 3,428 Consolidated balance sheet W4 Retained earnings of Paris 000 Balance at 31.12.20x2 per the balance sheet (11,629 8,286 15,171) Pre-acquisition profit Minority interest 10% of (11,829) Loss on restatement of goodwill 4,500 (1,183) (3317) (15,145) (330) (15,475) W5 The minority interest 000 Common shares (10% of 30,000) Retained earnings (W4) Additional paid in capital (10% of 10,000) Revaluation reserve (10% of 3,428) 3,000 (1,183) 1,000 343 3,160 (11,828) 000 3,428 343 3,085

The consolidated income statement for the year ended 31.12.20x2 includes the subsidiary figures using the exchange rates as shown in the question

251 Pearson Education Limited 2006

Barry Elliott and Jamie Elliott: Financial Accounting and Reporting (tenth edition) Instructors Manual

Sales Cost of sales Gross profit Depreciation Expenses Interest Profit before tax Taxation Profit after tax Minority interest Net profit for the year Dividend paid Retained profit Group statement of changes in equity for the year ended 31.12.20x2 [10% of (11,629 + Div 2,000)] (21,000 + 15,000/3) (30,000 + 8,571) (15,000 + 7,000/3) (6,000 + 3,000/3) (317,200 + (200,000/3) [170,000 + (8,800 + 0,000/3 2,667)]

383,867 (206,133) 177,734 (38,571) (17,333) (7,000) 114,830 26,000 88,830 (1,363) 87,467 (20,000) 67,467

Retained earnings brought forward (W6) Net profit for the year Dividend paid Loss on translation (90% of 15,171) Loss on restatement of goodwill Retained earnings carried forward W6 Group retained earnings brought forward at 1.1.20x2 Walpole Paris (67,000 this year 57,000) [90% of ( (8,286) pre-acquisition 5,000 (W1)] 10,000 (11,957) (1,957) (1,957) 87,467 (20,000) 65,510 (13,655) (330) 51,525

Note that in this example the loss on translation has been taken to retained earnings. In practice such gains or losses might be taken to a separate foreign currency reserve.

252 Pearson Education Limited 2006

Barry Elliott and Jamie Elliott: Financial Accounting and Reporting (tenth edition) Instructors Manual

CHAPTER 26

Chapter 26: Question 1 Alpha plc


(i)
Step 1 Theoretical ex-rights calculation The shareholders get an element of bonus at the same time as the company receives additional capital. The bonus element may be quantified by the calculation of a theoretical ex-rights price, which is compared with the last market price prior to the issue; the difference is a bonus. The theoretical ex-rights price is calculated as follows:
4 shares at fair value of 1 each prior to rights issue 1 share at discounted rights issue price of 80p each 5 shares at fair value after issue (i.e. ex-rights) The theoretical ex-rights price is 4.80/5shares The bonus element is fair value 1 less 96p = = = = = 4.00 0.80 4.80 0.96 0.04

Step 2 The time-weighted average number of shares is calculated for the current year
No. of shares Shares to date of rights issue Shares Increase by bonus fraction 2,000,000 Bonus : ((2,000,000 100/96) 2,000,000) Time adjustment 9/12 9/12 3/12 = = = 1,500,000 62,500 625,000 2,187,500

Shares from date of issue 2,500,000 Weighted average number of shares

Step 3 Calculate BEPS for current year BEPS for 20X1 is then calculated as 5,000,000/2,187,500 shares = 2.29

253 Pearson Education Limited 2006

Barry Elliott and Jamie Elliott: Financial Accounting and Reporting (tenth edition) Instructors Manual

(ii)
Step 4 Adjusting the previous years BEPS for the bonus element of a rights issue This bonus element will affect the comparison with the previous years BEPS which will need to be reduced to ensure comparability. The approach is to reduce the prior year by multiplying it by: Theoretical ex-rights fair value per share Fair value per share immediately before the exercise of rights = 0.96 1.00

Assuming that the earnings for 20X0 and 20X1 were 4.5m and 5m respectively, the 20X0 BEPS figures will be reported as follows: As reported in the 20X0 accounts 4.5m/2m As restated in the 20X1 accounts (4.5m/2m) (0.96/1.00) = = 2.25 2.16

The same effect is achieved by increasing the number of shares in the denominator by 100/96 for 20X0: Earnings/(Number of shares Bonus fraction ) 4,500,000 / (2,000,000 (100/96) = 2.16

254 Pearson Education Limited 2006

Barry Elliott and Jamie Elliott: Financial Accounting and Reporting (tenth edition) Instructors Manual

Chapter 26: Question 2 Beta Ltd


Beta Ltd weighted average number of shares
Time apportion 1 January 31 March 1,000,000 1 April 30 April 1,500,000 1 May 31 August 2,250,000 4/12 7/6 = 875,000 1/12 3/2 7/6 = 218,750 3/12 3/2 7/6 = 437,500 Bonus adjustment Bonus element in rights issue

1 September 31 October 3,250,000 2/12 7/6 = 631,944

1 November 31 December 4,333,333 2/12 = 722,222 2,885,416

Weighted average number of shares

Note: bonus element in rights issue calculated as follows: 3 shares at $5.60 1 share at $2.40 Fair value of 4 shares Theoretical ex-rights price Fair value Bonus factor = 5.6/4.8 = 7/6 = = 16.80 2.40 19.20 $4.80 $5.60

255 Pearson Education Limited 2006

Barry Elliott and Jamie Elliott: Financial Accounting and Reporting (tenth edition) Instructors Manual

Chapter 26: Question 3 Nottingham Industries plc


(a) EPS complying with IAS 33 definition of earnings:

Earnings for EPS calculation is profit of the period after tax, minority interests and extraordinary items and after preference dividends.
Basic EPS calculation: Equity earnings: Profit after tax and extraordinary items Preference dividend [10% of 1,000,000] Weighted average number of ordinary shares (25p) Actual no. 1.4.X5 in issue 1.7.X5 bonus issue 1.10.X5 Purchase 31.3.X6 in issue 16,000,000 3,200,000 19,200,000 (500,000) 18,700,000 6/12 9,350,000 18,950,000 Basic EPS for 20X6 Comparative for 20X5 480,000/18,950,000 = 0.022 5/6 = = 0.0253 0.0183 3/12 4,800,000 Weight time 3/12 Bonus factor 6/5 Weighted average 4,800,000 580 (100) 480 000

(b)

Diluted EPS calculation


000

Equity earnings: As for Basic EPS 480

The computation of Basic and Diluted EPS is as follows:


Per share Net profit for 20X6 Weighted average shares during 20X6 Basic EPS (480,000/18,950,000) Number of shares under option Number that would have been issued At fair value (200,000 1.00)/ 1.10 Diluted EPS 0.0253 480,000 (181,818) 18,968,182 0.02533 200,000 Earnings 480,000 18,950,000 Shares

256 Pearson Education Limited 2006

Barry Elliott and Jamie Elliott: Financial Accounting and Reporting (tenth edition) Instructors Manual

(c)

Usefulness of EPS figures

It is helpful to users to have a standardised EPS figure. This is provided by applying the IIMR calculation as follows. IIMR headline EPS Headline earnings per share are based upon the headline earnings figure stated in accordance with the Institute of Investment Management and Research Statement of Practice No. 1 The Definition of Headline Earnings and accordingly exclude profit on sale of the major operation.
000 Equity earnings: Profit after tax and extraordinary items Exclude capital items such as profit on sale of a major operation: 120,000 less tax 38,000 IIMR Headline EPS Less: preference dividend (82) 498 (100) 398 580

Even when standardised the ASB considers that there is too much emphasis on a single profit figure and encourages users to refer to the information set as a whole when appraising performance and predicting future earnings. Nevertheless, the EPS figure has remained an important figure in the eyes of many investors and analysts.

257 Pearson Education Limited 2006

Barry Elliott and Jamie Elliott: Financial Accounting and Reporting (tenth edition) Instructors Manual

Chapter 26: Question 4 Simrin plc


(a) Calculation of Basic EPS

As per IAS 33: EPS = = Profit after tax, Minority interest, Preference dividends and extraordinary items Number of ordinary shares 79,000 9,000 100,000

Basic Earnings per Share = 70p per share

(b)

Calculation of the Diluted EPS


Subscription monies received Notional number at fair value: 64,000/1.1 (fair value of a share) Notional number at no value = = 58,182 5,818 64,000 Profit after tax and preference dividend MI & extraordinary items Number of shares: At 1 January 20X0 From warrants at no value Total number of shares Diluted EPS = 70,000/105,818 = 66.15p per share 100,000 5,818 105,818 70,000 = 1.28 50,000 = 64,000

(c)

(i) Need to disclose DEPS


Company able to finance projects using convertible securities which carried fixed interest rate and also future benefits causing dilution of shares in the future on conversion. Trend revealed by diluted EPS is more meaningful to shareholders as it enables them to identify the final effect on companys EPS by using convertible debt.

258 Pearson Education Limited 2006

Barry Elliott and Jamie Elliott: Financial Accounting and Reporting (tenth edition) Instructors Manual

(ii) Relevance to shareholders


Relevance is questionable. It shows dilution of future earnings per share and it is reasonable that existing shareholders should be given a warning of the potential dilution.

(d)

Reliance on EPS as single most important indicator of financial performance


There is no one correct answer for this, but discussion of the Institute of Investment Management and Research headline figure is required.

259 Pearson Education Limited 2006

Barry Elliott and Jamie Elliott: Financial Accounting and Reporting (tenth edition) Instructors Manual

Chapter 26: Question 5 Gamma Ltd


There are two steps in arriving at the Diluted EPS, namely: Step 1 Determine the increase in earnings attributable to ordinary shareholders on conversion of potential ordinary shares Step 2 Determine the potential ordinary shares to include in the Diluted Earnings per Share

(a)

Convertible preference shares receive a dividend of 2.50

Step 1 Determine the increase in earnings attributable to ordinary shareholders on conversion of potential ordinary shares
Increase in earnings Convertible preference shares Increase in net profit 50,000 shares 2.50 Incremental shares 50,000/1 10% Convertible bond Increase in net profit 250,000 0.10 (1 0.4) Incremental shares 250,000/1000 500 125,000 0.12 15,000 50,000 2.50 125,000 Increase in number Earnings per of ordinary shares incremental share

Step 2 Determine the potential ordinary shares to include in the computation of diluted earnings per share
Net profit attributable to continuing operations As reported 10% Convertible loan Convertible preference shares 5,000,000 15,000 5,015,000 125,000 5,140,000 shares 1,000,000 125,000 1,125,000 50,000 1,175,000 4.37 dilutive 4.46 dilutive Ordinary Per share 5.00

260 Pearson Education Limited 2006

Barry Elliott and Jamie Elliott: Financial Accounting and Reporting (tenth edition) Instructors Manual

(b)

Convertible preference shares receive a dividend of 6 per share.

Step 1 Determine the increase in earnings attributable to ordinary shareholders on conversion of potential ordinary shares
Earnings per Increase in earnings Convertible preference shares Increase in net profit 50,000 shares 6.00 Incremental shares 50,000/1 10% Convertible bond Increase in net profit 250,000 0.10 (1 0.4) Incremental shares 250,000/1000 500 125,000 0.12 15,000 300,000 50,000 6.00 Increase in number of ordinary shares incremental share

Step 2 Determine the potential ordinary shares to include in the computation of diluted earnings per share
Net profit attributable to continuing operations As reported 10% convertible loan Convertible preference shares 5,000,000 15,000 5,015,000 300,000 5,315,000 Ordinary shares 1,000,000 125,000 1,125,000 50,000 1,175,000 4.52 anti-dilutive 4.46 dilutive 5.00 Per share

Since the Diluted EPS is increased when taking the convertible preference shares into account (from 4.46p to 4.52p) , the convertible preference shares are anti-dilutive and are ignored in the calculation of Diluted EPS. The lowest figure is selected and the Diluted EPS will, therefore, be disclosed as 4.46p.

261 Pearson Education Limited 2006

Barry Elliott and Jamie Elliott: Financial Accounting and Reporting (tenth edition) Instructors Manual

Chapter 26: Question 6 Delta NV


(a) Calculate theoretical ex-rights value of a share

Market value of a share prior to rights issue was 1.10. 4 shares at 1.10 per share 1 share at 60p 5 shares Theoretical ex-rights value = = = = 4.40 .60 5.00 1.00

(b) (c)

Bonus issue factor BEPS 20X8


440,000/(4,000,000 11/10)

=110/100

= 0.10

previously calculated as: 440,000/4,000,000 = 0.11

(d)

BEPS 20X9
= = = = 2,200,000 2,500,000 4,700,000 0.106

Uplift shares prior to issue by 110/100 4,000,000 (110/100) 6/12 months Weight shares after issue: 5,000,000 6/12 months Total shares for BEPS calculation BEPS = 500,000/4,700,000

262 Pearson Education Limited 2006

Barry Elliott and Jamie Elliott: Financial Accounting and Reporting (tenth edition) Instructors Manual

Chapter 26: Question 7 X Ltd


(a)
Ordinary shares Net profit after tax Less preference dividend 40,000 Options (W1) Convertible preference shares Convertible loan stock Interest [6% 20m s 0 .67] Discount Shares converted [(20m/200) 23] 2,300 45,900 _____ 19,164 41.8p anti-dilutive 804 200 3,200 43,600 160 18,160 41.7p dilutive 400 40,400 18,000 44.6p dilutive Profit 18,160 (160) 18,000 45p EPS Effect


W1

Since the loan stock is anti-dilutive it is ignored in the calculation of Diluted Earnings per Share Diluted EPS will be reported as 41.7p.

Fair value of one ordinary share Number of options Exercise price Proceeds from exercise of options Number of shares assumed to be issued at fair value Number of shares issued for no consideration (2m 1.6m)

1.50 2,000,000 1.20 2,400,000 1,600,000 = 400,000

(b) An option is treated as if there was an issue of shares for full market value/fair value; and an issue for no consideration (a bonus issue). The bonus element is treated as being the dilutive effect. IAS 33 is saying that by issuing options to directors/employees the company is making a bonus issue of shares plus a full issue of shares, the latter being assumed not to have a dilutive effect.

263 Pearson Education Limited 2006

Barry Elliott and Jamie Elliott: Financial Accounting and Reporting (tenth edition) Instructors Manual

Only potential ordinary shares that would dilute EPS should be taken into account and any anti-dilutive potential ordinary shares will be ignored. This procedure essentially means that certain categories of potential ordinary shares will not be used in the calculation. Thus the calculation will be based on the concept of prudence rather than on the substance of what is realistically going to occur. All items of income or expense that would cease on conversion are to be added back. Prudent disclosure.

As regards the ranking of potential ordinary shares from most to least dilutive and the subsequent calculations, an alternative solution would be to disclose both the fully diluted EPS and the maximum dilution of EPS. This would essentially mean that the more realistic calculation and the prudent calculation of IAS 33 would be disclosed.

264 Pearson Education Limited 2006

Barry Elliott and Jamie Elliott: Financial Accounting and Reporting (tenth edition) Instructors Manual

CHAPTER 27

Chapter 27: Question 1 Example Ltd


000 Cash flows from operating activities Net profit before tax Adjustments for: Depreciation Profit on sale of plant Interest expense Operating profit before working capital changes Increase in trade and other receivables Increase in inventories Decrease in trade payables Cash generated from operations Interest paid Income taxes paid Net cash used in operating activities Cash flows from investing activities Purchase of property, plant and equipment Proceeds from sale of equipment Payment to acquire government securities Net cash used in investing activities Cash flows from financing activities Proceeds from issuance of share capital Redemption of debentures Dividends paid Net cash from financing activities Net increase in cash and cash equivalents Cash and cash equivalents at the beginning of the period Cash and cash equivalents at the end of the period 300 (50) (120) 130 (540) 72 (468) (560) 241 (20) (339) 102 (13) 20 609 (260) (400) (40) (91) (20) (220) (331) 500 000

265 Pearson Education Limited 2006

Barry Elliott and Jamie Elliott: Financial Accounting and Reporting (tenth edition) Instructors Manual

Chapter 27: Question 2 Martel plc


(a) Cash flows from operating activities Net profit before tax Adjustments for: Depreciation Profit on sale of plant Interest expense Operating profit before working capital changes Increase in trade and other receivables Increase in inventories Increase in trade payables Cash generated from operations Interest paid Taxes paid Net cash used in operating activities Cash flows from investing activities Purchase of property, plant and equipment Proceeds from sale of equipment Purchase of government securities Net cash used in investing activities Cash flows from financing activities Proceeds from issuance of share capital Proceeds from 9% debenture issue Dividends paid Net cash from financing activities Net increase in cash and cash equivalents Cash and cash equivalents at the beginning of the period Cash and cash equivalents at the end of the period Note 1: Cash and cash equivalents 20X1 Bank Overdraft Cash and cash equivalents (184) (184) 20X0 22 22 150 82 (76) 156 (206) 22 (184) (714) 20 (40) (734) 292 (8) 52 763 (132) (174) 46 503 (52) (79) 372 427 000 000

266 Pearson Education Limited 2006

Barry Elliott and Jamie Elliott: Financial Accounting and Reporting (tenth edition) Instructors Manual

(b) Martel plc has invested heavily in fixed assets during the year and although it has raised additional capital it has had to rely on a bank overdraft. The acid test ratio is lower in the current year (304 : 642). However, we do not have information on the projected cash flows that supported the capital investment decisions this is where narrative information within the annual report could be helpful in identifying the companys strategic planning for future years, e.g. new markets, new products, greater productive efficiency.

267 Pearson Education Limited 2006

Barry Elliott and Jamie Elliott: Financial Accounting and Reporting (tenth edition) Instructors Manual

Chapter 27: Question 3 Flow Ltd


Cash flow Statement for the year ended 31 December 20x6 for Flow Ltd
Net cash inflow from operating activities Cash flows from investing activities Payment to acquire non-current assets [1,983,750 + 25,5001,743,750] Receipts from sale of noncurrent assets Net cash paid on investing activities Cash flows from financing activities Issue of common shares Dividends paid (could be shown as operating cash flow) Net cash inflow from financing activities Net increase in cash and cash equivalents [75,0008,250] Net cash inflow from operating activities: Profit (176,625 (387,000300,00045,000)) Non cash items Depreciation [619,125551,250 +(25,5009,375)] 84,000 3,150 Loss on disposal [9,3756,225] Changes in working capital Decrease in inventory Increase in trade receivables Decrease in trade payables Net cash inflow from operating activities 15,750 (22,500) (24,000) 191,025 134,625 66,750 (45,000) 135,000 180,000 6,225 (259,275) (265,500) 191,025

268 Pearson Education Limited 2006

Barry Elliott and Jamie Elliott: Financial Accounting and Reporting (tenth edition) Instructors Manual

Chapter 27: Question 4 Blue Ting plc


(a)
Cash Flow Statement for the year ended 31 May 20X5 m Cash flows from operating activities Profit before tax Adjustments for: Depreciation expense Amortisation of development expenditure Interest income Interest expense Loss (gain) on disposal of property, plant and equipment Operating profit (loss) before working capital changes Increase in accounts receivable Decrease in inventories Increase in payables Cash generated from (used in) operations Interest paid Interest income Interest element in finance lease rental payment Tax paid [10 + 22 16] Dividends paid [12 + 8] Net cash from operating activities Cash flows from investing activities: Proceeds from disposal of property, plant and equipment Purchase of property, plant and equipment Net cash used in investing activities Cash flows from financing activities: Proceeds from issuing shares Share issue costs Purchase of own shares Capital repayments under finance leases Net cash used in financing activities Net increase (decrease) in cash and cash equivalents Cash and cash equivalents at beginning of year Cash and cash equivalents at end of year 14 (1) (12) (7) (6) 91 3 94 21 (104) (83) 37 1 (3) 7 (2) 136 (20) 15 89 220 (4) 3 (3) (16) (20) 180 96 m

(Note: Dividends paid have been treated in their optional position in operating activities and the cash and liquid resources have been assumed to be cash and cash equivalents.)
269 Pearson Education Limited 2006

Barry Elliott and Jamie Elliott: Financial Accounting and Reporting (tenth edition) Instructors Manual

(b)

Published forecast cash disadvantages to users

flow

information:

advantages

and

Advantages Provides the benefit of: managements knowledge of future cash flow their views as to future cash flows committing management to future planning committing to considering going concern issues. Reduces the benefits of insider dealing as information would be in the public domain. Makes it more possible to evaluate managerial performance Aids investors and creditors to assess the ability of the company to meet its obligations in the future.

Disadvantages They are uncertain. They are subjective as based on the opinions of management. They can be manipulated by management although a poor history of accuracy will become apparent over time.

270 Pearson Education Limited 2006

Barry Elliott and Jamie Elliott: Financial Accounting and Reporting (tenth edition) Instructors Manual

Chapter 27: Question 5 Carver plc


Cash Flow from Operations for the year ended 30 September 20X4 000 Profit before tax 1,985 Adjustments for: Depreciation Investment income Interest payable Share of profit of associate Profit on sale of machinery Operating profit before working capital changes Increase in inventory Increase in trade receivables Increase in trade payables Cash generated from operations Interest paid Taxes paid Net cash from operating activities Cash from investing activities Purchase of machinery Sale of machinery Purchase of subsidiary Dividends received from associate Dividends received from fixed asset investments Cash used in investing activities Cash flow from financing Proceeds from the issue of share capital Issue of loan stock Capital payments under finance leases Dividends paid to group shareholders Dividends paid to minority interests W12 W9 W10 W11 2,453 920 (270) (300) (48) W7 W8 W6 (1,085) 500 98 250 155 (82) W5 W2 W3 W4 W1 325 (155) 150 (495) (100) 1,710 (943) (547) 152 372 (100) (250) 22 000

271 Pearson Education Limited 2006

Barry Elliott and Jamie Elliott: Financial Accounting and Reporting (tenth edition) Instructors Manual

Cash flow from financing Increase in cash and cash equivalents Cash and cash equivalents brought forward Cash and cash equivalents carried forward Workings W1: Depreciation charge Buildings Machinery Closing aggregate amount Less: Opening aggregate amount(1,100) 100 Add: depreciation on disposal 100 1,200

2,755 2,695 1,820 4,515

125

200 325

W2: Inventory Closing balance Less: Opening balance Arising on acquisition 1,000 32 (1,032) 943 W3: Trade receivables Closing balance Less: Opening balance Arising on acquisition 1,275 28 (1,303) 547 W4: Trade payables Closing balance Less: Opening balance Arising on acquisition 280 68 (348) 152 W5: Tax Opening balances Income tax Deferred tax Transfer from profit and loss account Closing balances 217 13 495 500 1,850 1,975

272 Pearson Education Limited 2006

Barry Elliott and Jamie Elliott: Financial Accounting and Reporting (tenth edition) Instructors Manual

Income tax Deferred tax Acquisition tax

(462) (30) 233 17 250

W6: Investment in machinery Cost at 30.9.20X4 Less: Cost at 1.10.20X3 Add: Disposal Less: Arising from acquisition Leased Cash outflow W7: Cash Cash acquired from acquisition Less: Cash consideration Cash inflow W8: Dividends received from associate Opening balance Add: Share of profit Less: Tax Closing balance Cash inflow W9: Shares Closing balances Shares Premium Less: Opening balances Shares Premium (2,000) (2,095) 3,940 2,883 6,823 495 (145) 350 1,350 (1,100) 250 1,000 112 (14) 98 3,000 (1,400) 1,600 500 2,100 (165) (850) 1,085

273 Pearson Education Limited 2006

Barry Elliott and Jamie Elliott: Financial Accounting and Reporting (tenth edition) Instructors Manual

Non-cash consideration Shares Premium Cash inflow W10: Loans Closing balance Less: Opening balance Less: Increase finance cost Cash inflow W11: Lease capital payments Opening balances [200 + 170] Add: new lease commitment Less: Closing balances [240 + 710] Cash outflow W12: Minority interests Opening balance Add: Profit for year Arising from acquisition Closing balance Cash inflow 100 63 163 (115) 48 370 850 1,220 (950) 270 1,460 (500) 960 (40) 920 (220) (55) 2,453

274 Pearson Education Limited 2006

Barry Elliott and Jamie Elliott: Financial Accounting and Reporting (tenth edition) Instructors Manual

CHAPTER 28

Chapter 28: Question 1 Saddam Ltd


(a) Profitability ROCE
Camel Ltd is the most profitable of the three companies. An inspection of the secondary ratios shows that this is due to efficient utilisation of assets since its net profit ratio is well below that of the other two companies. Examination of gross profit percentages confirms the observation that Camel Ltd seems a high volume, low margin business compared with the others.

Liquidity Ali Ltd has a current ratio which is out of line with the other two, being very much higher suggesting surplus investment in working capital. The acid test ratio reinforces this view and also indicates that Baba Ltd appears to have a liquidity problem with current liabilities considerably greater than cash and debtors (despite having the greatest number of weeks debtors outstanding of the three companies). Baba Ltd also has considerably more weeks of stock outstanding than the other two companies which may be linked with the high level of creditors. Ali Ltd also has stock levels well in excess of Camel Ltd explaining in part at least the high current ratio.

Dividends Camel Ltd is paying a higher proportion of profits out in dividends, which may have the effect of raising shareholder loyalty and the bid price. Conclusion Baba Ltd appears to have considerable liquidity problems arising out of excess investment in stock. Camel Ltd is a lean enterprise able to survive on a lower gross profit margin due to superior asset utilisation. Why is the gross profit margin low?

Before a final decision is made the absolute figures in the financial statements should be studied and questions raised such as: Are the activities of the firms really the same? What are the relative turnovers? What is the growth over a period of years? What are the trends of all the ratios? How old are the assets? Are asset ages distorting ROCE comparisons between the companies?
275 Pearson Education Limited 2006

Barry Elliott and Jamie Elliott: Financial Accounting and Reporting (tenth edition) Instructors Manual

Also need to assess managerial skills, product potential etc. which are not shown in the financial statements.

(b) Why balance sheet is unlikely to show the true market value of the business
The accounting policy in the UK is to state fixed assets at cost less depreciation or at historical cost modified by revaluation of all or selected classes of fixed assets. The true market value of a listed company is available from the market capitalisation figure based on current share prices. The true market value of an unquoted company is not readily available and would require the future cash flows to be evaluated.

276 Pearson Education Limited 2006

Barry Elliott and Jamie Elliott: Financial Accounting and Reporting (tenth edition) Instructors Manual

Chapter 28: Question 2 Esrever Ltd


Forecast profit and loss account for year ended 30 June 20X1 Turnover Opening stock Purchases Closing stock Cost of sales Gross profit Depreciation buildings fixtures etc. [132,000 2%] [96,750 20%] [50,000 12%] (S6) (S6) (S7) (S8) 2,640 19,350 6,000 33,655 61,645 Profit before tax Corporation tax Profit after tax Dividends Profit retained Profit retained b/f Retained profit c/f Forecast balance sheet as at 30 June 20X1 Fixed assets (NBV) Land and buildings Fixtures, fittings Current assets Stock Debtors [(271,897 42.6/365) 1.15] (S15) 31,355 (S16) 36,494 67,849 [132,000 2,640] [96,750 19,350] 129,360 77,400 (S14) 206,760 [20,290 20/80] [181,808 11.16%] [200,000 2.5p] (S10) (S11) (S12) (S13) (S9) (S1) 25,362 5,072 20,290 5,000 15,290 66,518 81,808 [184,890 61.9/365] [271,897 68%] [20,290 100/23.32] [87,007 100/32] 22,040 (S5) 194,205 216,245 (S5) 31,355 (S4) (S2) 184,890 87,007 (S3) 271,897

Loan interest

Credit expenses (balancing figure)

277 Pearson Education Limited 2006

Barry Elliott and Jamie Elliott: Financial Accounting and Reporting (tenth edition) Instructors Manual

Creditors: amounts falling due in less than one year Bank overdraft (a balance figure based on note 2) Creditors [(194,205 + 33,655) (29.7/365) 115%] Other creditors [5,072 tax + 5,000 dividends + 1,652 VAT] Net current assets Total assets less current liabilities (per Note 3) Creditors: amounts falling due in more than one year 12% loan (S23) 50,000 181,808 Ordinary shares Profit and loss account (balancing figure) (S21) (S22) 100,000 81,808 181,808 VAT: Output tax Input tax Net amount for year 6,606 0.25 [271,897 15%] [(194,205 + 33,655) 15%] 40,785 34,179 6,606 1,652 (S18, S19) 11,724 42,801 25,048 231,808 (S20) 9,756 (S17) 21,321

Approach to Esrever profit and loss account (S1) (S2) (S3) (S4) (S5) Start with post-tax profit i.e. 11.16% of (231,808 50,000) per notes 3 & 4 = 20,290 From post-tax profit 20,290 derive gross profit as 100/23.32 20,292 based on Note 4 = 87,007 Next, derive turnover as 100/32 87,007 based on Note 6. Cost of goods sold = 68% of turnover. Therefore turnover = 100/32 gross profit = 271,897 From sales and gross profit derive cost of goods sold as 271,897 87,007 = 184,890 You can now find components of cost of sales (184,890) as: (a) Opening stock 22,040 (given in question) (b) Purchases 194,205 (balance figure) 216,245 (c) Closing stock (31,355) (61.9 184,890) 365 Total costs of goods sold 184,890 Note: Start with closing stock 61.9 days based on Note 7; all other figures are derived and the opening stock is given as 22,040.

278 Pearson Education Limited 2006

Barry Elliott and Jamie Elliott: Financial Accounting and Reporting (tenth edition) Instructors Manual

(S6)

2% 132,000 for buildings = 2,640 20% 96,750 for fixtures etc. = 19,350 based on Note 1 and opening asset given (S7) Loan interest is 12% of 50,000 = 6,000 (S8) Expenses this is a balancing figure as we already have all the other figures in the profit and loss account = 33,655 (S9) Taxation charge is 20/80 20,290 based on Note 5 = 5,072 (S10) Dividend see Note 9 (200,000 2.5p) = 5,000 (S11) Retained profit = 15,290 (S12) Retained profit b/forward is a balancing figure = 66,518 (S13) Retained profit c/down (see S22 below) = 81,808 Approach to Esrever balance sheet Projected balance sheet as at 30/6/20X1 is built up as follows: (S14) Fixed assets are derived from the opening figure less depreciation (S15) Stock has already been computed at (S16) Debtors, based on Note 10, assuming 42.6 days credit, are 42.6/365 271,897 = 31,734 1.15 to cover VAT (S17) Creditors, assuming credit of 29.7 days, are 29.7/365 227,860 1.15 (S18) Other creditors (dividends 5,000 + tax 5,072) (S19) VAT 15% net of sales purchases and expenses is 15% (271,897 194,205 33,655) x 0.25 (S20) Overdraft is balancing figure based on Note 2 Current liabilities Total assets less current liabilities per Note 3 (S21) Share capital given in question (S22) Retained profit (balancing figure) (S23) 12% loan = = = = = = = 206,760 31,355 36,494 (21,321) (10,072) (1,652) (11,724) (9,756) 42,801 231,808 100,000 81,808 50,000 231,808

Depreciation:

Note: Retained profit is the balancing figure to make up 231,808. The bank overdraft of 9,756 is the overall balance sheet balancing figure.

279 Pearson Education Limited 2006

Barry Elliott and Jamie Elliott: Financial Accounting and Reporting (tenth edition) Instructors Manual

Chapter 28: Question 3 Amalgamated Engineering plc


(a) Cash flow statement for year ended 31 December 20X6
000 Net cash inflow from operating activities Returns on investing and servicing of finance Interest paid Net cash outflow Taxation Tax paid Capital expenditure Payment to acquire plant Receipt from sale of investments Equity dividends paid Net cash outflow before financing Financing Increase in overdraft (225) (450) (450) (450) 300 (150) 375 (75) 195 195 300 000 495

Reconciliation of operating profit to net cash inflow from operating activities Operating profit Depreciation Increase in stocks Increase in debtors Increase in creditors 795 300 (375) (300) 75 495

(b)
Liquid ratio Current assets inventory Current liabilities

20X5

20X6

1,125 1,125 = 1 =

1,125 1,575 0.71

280 Pearson Education Limited 2006

Barry Elliott and Jamie Elliott: Financial Accounting and Reporting (tenth edition) Instructors Manual

Interest cover Profit before interest Interest charge 885 135 = 6.56 times Return on average shareholders funds Profit after tax Av. shareholders funds (Could revalue property) Gearing ratio Long-term loans Shareholders funds or Long-term loans Long-term loans and shareholders funds = 24.7% Stock turnover ratio Cost of sales Inventory 4,410 1,125 = 3.92 times 4,680 1,500 = 3.12 times = 24.4% 1,500 (1,500 + 4,575) 1,500 (1,500 + 4,650) 1,500 4,575 = 32.8% 1,500 4,650 = 32.3% .375 (4,575 + 4,425) / 2 = 8.3% ..300 (4,650 + 4,575) / 2 = 6.5% 795 195 = 4.08 times

(c)

Main points in report should cover the following. Most important points are with an asterisk.

Profitability *(1) Given unchanged sales volume (NB cannot tell from historical cost accounts without date on specific price movements), price rises have been below the level of general inflation (4.8%). Is this deliberate policy or just poor management? If deliberate appears not to have improved sales.

(2) Cost of materials and labour also increased below level of inflation (5% and 5.6% respectively). More efficient use? (3) Overheads increased 10% in line with inflation (both production and administrative) led to falling margins (gross and net). (Further information by product might help see if one particular area is a problem or if it is right across the board.) (4) Increased interest has caused profit before tax to fall 20% although interest cover still looks OK. (NB Is this relevant? Interest is paid from cash.)

281 Pearson Education Limited 2006

Barry Elliott and Jamie Elliott: Financial Accounting and Reporting (tenth edition) Instructors Manual

*(5)

Trends are worrying falling margins and rising interest seem to indicate problems soon. How long can firm continue to hold the dividend? (Need more years data longterm picture. Is this a recent trend or not?)

Solvency/liquidity (6) Working capital rising trade receivables and inventories are up a lot. *(7) *(8) Reflected in worsening liquid ratio quite a large fall. (Again, need more years data. What is norm?) Inventory turnover is getting worse 3.85 months inventory on hand (20X5 3.06). Need more information here slow-moving inventory? Or is it just poor management of working capital? Trade receivables turnover ratio has got worse (20X5 7.64; 20X6 5.87). In their state they need to be collecting more quickly. Is there one or a few debts causing this, or is it general sloppiness? Flow of funds company is investing in new equipment so is presumably not contracting operations. Need information as to use equipment is being put to, and future capital expenditure plans. *Purchases of assets (+ payment of tax + dividend) have been partly paid for by selling off short-term investments. This is a one-off bad sign. Could use previous 5 years funds flow statements trends quite important. *(11) The increased overdraft is financing the increased stocks and debtors. (12) Gearing ratio is OK but the problem is one of liquidity at the moment. Could argue the overdraft appears to be a permanent feature of this firm. The gearing ratio looks worse if the overdraft is included (+ an overdraft of 1,500,000 makes it look even more unhealthy). (Gearing ratios calculated using book values may not be too useful could recalculate using market values of debt and equity, where quoted.) General points *(13) Why does firm want to increase the overdraft? Seems to be to finance working capital. Could be risk for the bank if the firms profitability is in a long-term decline (Does not mean dont lend could charge more interest.) *(14) Or could secure the overdraft market value of the land and buildings is well in excess of the debentures. *(15) How will firm pay off the overdraft? Need to ask for cash forecasts for next few years (firm should have if not, poor management). (NB Historical cost accounts generally of little help with respect to forward-looking data.) (16) More data on management. Old, young? Likely to let firm stagnate? Also need to see strategic plans in what direction is firm going? Do they know?

*(9)

(10)

282 Pearson Education Limited 2006

Barry Elliott and Jamie Elliott: Financial Accounting and Reporting (tenth edition) Instructors Manual

(d) Response to director


(i) Debt Service Coverage Ratio This ratio requires the figures for interest, tax, depreciation and amortisation charge to calculate EBITDA. The ratio gives the bank an indication of the companys ability to meet its capital debt repayments as well as annual interest payments from its cash flow from operations.

(ii) Cash flow from operations to current liabilities This ratio requires the cash flow from operations figure in the cash flow statement. The ratio gives additional information to the current and acid test ratios which are static in the sense that both the numerators and denominators are based on year-end figures which are capable of manipulation or management e.g. running down stocks or exceptional cash receipts at the year-end. Cash recovery ratio This ratio requires the figures for cash flow from operations and proceeds of sale of fixed assets from the cash flow statement. The ratio gives an indication of the payback time i.e. how quickly the company will recoup its investment in fixed assets from its cash flow. The manager would naturally regard a shorter period as less risky.

(iii)

283 Pearson Education Limited 2006

Barry Elliott and Jamie Elliott: Financial Accounting and Reporting (tenth edition) Instructors Manual

Chapter 28: Question 4 Sally Gorden


(a) EPS
Earnings: Profit Pref. div. Number of shares: 1.7.X3 30.9.X3: 1,500,000 3/12 1.10.X3 31.6.X4: 2,000,000 9/12 = = 375,000 1,500,000 1,875,000 190,000 / 1,875,000 100 = 240,000 / 3,000,000 100 = 10.13p 8p 280,000 (90,000) 190,000 Ruby Sapphire

(b) Price/earnings ratio


475/10.13 = 480/8 47 times 60 times

(c) PE ratio of Sapphire plc is almost twice that of Ruby plc.


This would reveal that there is much higher demand for shares in Sapphire. This in turn indicates greater confidence the investing public has in that company. This confidence may be based on the type of industry growth potential, growth rate track record of past performance diversity of its products quality of management customer attachment and so on.

(d) Other matters that should be considered


(i) What Sapphires EPS would have been if there had been no bonus issue: 240,000/2,000,000 100 Sapphire appears better than Ruby 12p

284 Pearson Education Limited 2006

Barry Elliott and Jamie Elliott: Financial Accounting and Reporting (tenth edition) Instructors Manual

(ii) Return on capital employed (ROCE) PBIT/capital employed 100 588,000 / 2,710,000 100 = 445,000 / 2,450,000 100 = (iii) Return on equity capital (ROEC) PBT less pref. Div./equity cap + reserves 354,000/1,310,000 100 = 385,000/1,950,000 100 = Ruby provides a better return on equity but its EPS is not quite so favourable it is more geared, and had borrowed at a significantly higher cost than Sapphire. (iv) Gearing ratio Prior charge capital/total CE 100 1,400,000/2,710,000 100 = 500,000/2,450,000 100 = 51.66% 20.4% 27% 19.7% 21.7% 18.2%

Though both companies are geared, Ruby is highly geared. This means that any fall in profit will affect equity shares more than in proportion.

(e) Advantages of gearing


Equity shareholders benefit if the return on investment exceeds the cost of borrowing. There is no dilution of the existing shareholders interest if funds are raised by borrowing rather than by an issue to new shareholders. Loan interest is allowable for tax relief. Lenders normally obtain some form of security in the form of either a charge on assets or prior rights on liquidation. This means that their risk is lower and therefore their required rate of interest is lower.

Disadvantages of gearing Impact on companys funding if loan covenants are breached e.g. may be required to renegotiate the loan at a higher rate of interest or even by issuing additional ordinary shares to the lenders in recognition of their increased risk. Impact on companys funding if equity shareholders perceive that there is a greater risk to equity funds if there is high gearing and as a result require a higher return on their investment. Adverse impact on amount available for distribution to shareholders if profits fall.

285 Pearson Education Limited 2006

Barry Elliott and Jamie Elliott: Financial Accounting and Reporting (tenth edition) Instructors Manual

Chapter 28: Question 5 Segmental Reporting


(i) The case for segmental reporting two arguments for
It will reveal in more detail how well management has performed. Management will not be able to hide its failures behinds its successes. Both will be disclosed and shareholders will be better able to judge the performance of directors. In addition, disclosure of segment results may encourage management to exercise greater care when making investment decisions and be more positive in correcting any mistakes. The first argument is, then, that segmental reporting will result in improved managerial performance. The data provided by segmental reporting will be more useful for the investors. This is because many financial statement users have said that consolidated financial information, while important, would be more useful if supplemented with disaggregated information to assist them in assessing those uncertainties that surround the timing and amount of expected cash flows. This would allow them, therefore, to assess the risks related to a personal investment in or a loan to an enterprise that may well operate in different industries or in different areas of work. The results of a diversified enterprise are composed of the results of its parts and the financial users consequently regard financial information on a segmental basis as also important.

(ii) The case against segmental reporting


The case against segmental reporting arises from a consideration of cost and reliability. An important consideration in assessing the desirability of disclosing segmental data is a comparison of the benefits arising from and the costs incurred by any such disclosure. If the benefits exceed the costs, then the disclosure is desirable. This comparison is difficult to make in practice because the benefits are enjoyed by the users while the costs are incurred by the statement providers. It is not surprising that users express a need for segmental data because it costs them nothing. Equally, it is not surprising that the statement providers do not want to incur the costs of disclosing segmental data because they are unlikely to receive any benefits and, even worse, they run the risk of their managerial deficiencies being revealed. the costs of collecting and processing the information, the costs of audit,

The costs that may be incurred by the statement providers include

286 Pearson Education Limited 2006

Barry Elliott and Jamie Elliott: Financial Accounting and Reporting (tenth edition) Instructors Manual

the costs of disseminating it to those who must receive it and the costs of disclosure in the form of a loss of competitive advantage vis--vis trade competitors or trade unions with a consequent effect on wage demands.

It follows that a comparison of the private costs incurred by the providers and the private benefits enjoyed by the users is likely to be inconclusive. A more fruitful, but again difficult, approach would be to compare the social costs with the social benefits. The social costs would be the resources consumed in the gathering, processing and publication of the segmental data. The social benefits would be the improved allocation and more efficient use of resources. The second major objection to the provision of segmental data is their reliability. It is argued that segmental data are not sufficiently reliable to justify disclosure. If this is true the unreliable data may be just as misleading as no segmental data at all. The unreliability is due to the fact that there is the necessity to make arbitrary allocations of both costs and revenues amongst the various segments of the business. The degree of arbitrariness will depend upon the nature and size of the reporting segments and the amount of detail disclosed for each segment. There are other specific objections to the disclosure of segmental data that may be made. These include: Investors invest in a company and not its individual segments. Whilst this is correct it cannot be denied that data about the operations of individual segments may permit investors to make better informed decisions about investments. The data are difficult to interpret and may confuse readers or be misunderstood with inappropriate inferences being drawn. It is usually assumed, however, that the statement users are technically competent and able to understand accounting data. Segmental data cannot be prepared with sufficient reliability and it is beyond the scope of external financial reporting to provide such analytical or interpretive data. It is true then that there are reliability problems with producers of segmental data, but whether those problems are sufficient to warrant non-disclosure of the data is a matter of judgement. It is sometimes maintained that the disclosure of segmental data constitutes analysis and interpretation and is, therefore, beyond the scope of financial reporting. However, this is a matter of opinion. Whilst analysis and interpretation do usually involve the study or reordering of existing published data, segmental reporting provides additional data not otherwise available. It is difficult to argue, therefore, that the provision of segmental data constitutes analysis and interpretation. There may be a negative impact on corporate innovation and experimentation. If mistakes are disclosed, management may be inclined to minimise risk to avoid mistakes, and innovation may suffer. This argument is difficult to assess. In the long run, of course, a lack of innovation will lead to poor performance and dissatisfaction with management. It seems likely that investors will be sufficiently sophisticated to realise that continued success requires innovation, which means that some risks must be taken. The costs of providing segmental information are too high.
287 Pearson Education Limited 2006

Barry Elliott and Jamie Elliott: Financial Accounting and Reporting (tenth edition) Instructors Manual

The objection relates primarily to a fear that disclosure of segmental data may weaken the firms competitive position. This objection has been fairly widely researched and the general conclusion seems to be that researchers found that companies rarely if ever, encounter(ed) any real loss of competitive advantage as a result of segment reporting.

288 Pearson Education Limited 2006

Barry Elliott and Jamie Elliott: Financial Accounting and Reporting (tenth edition) Instructors Manual

(b)
(a) (b)

Chapter 28: Question 6 Filios Products plc


Refer to Chapter 28: Question 5. Segmental statement (m)
Beer & pub Hotel business Other drinks & leisure 368 Total operations 1,028

Classes of business Turnover Turnover Profit Segment profit (W1) Common costs Operating profit Interest Published net profit Net assets Segment net assets (W2) Unallocated assets (W2) Published net assets Workings W1 Sales Cost of sales Administration Distribution costs Segment profit W2 Net assets

508

152

85

45

18

148 15 133 14 119

1,127

391

403

1,921 82 1,839

508 316 43 64 423 85*

152 81 14 12 107 45

368 287 18 25 350 18

Fixed assets book value


Stocks and debtors Bank Less Current liabilities Net current assets

890 230 73 303 66 237

332 84 15 99 40 59

364 67 28 95 56 39

77

12 12 31 (19)

289 Pearson Education Limited 2006

Barry Elliott and Jamie Elliott: Financial Accounting and Reporting (tenth edition) Instructors Manual

10% Debentures Net assets 1,127*

(140) 391 403 (82)

* Excluding interest and interest bearing loans as appropriate

(c) Analysis calculations


Accounting ratios: Operating divisions Beer & Pub % Operating profit % Asset turnover Return on assets 16.7 45.1 7.5 Hotel business % 29.6 38.9 11.5 % 4.9 91 4.5 % 13.3 46.0 6.2 % 40.0 50.0 20.0 Other Competitors Dean Clarke

(i) Possible comments: The best performing segment, based on the primary accounting ratio, the return on assets, is the hotel division. The superior performance of the hotel division is attributable to the fact that it is able to generate higher operating margins than either of the other segments, and this outweighs the fact that it has the lowest asset turnover. It would appear that Filios beer and pub division pursues a policy of higher selling prices and margins while also endeavouring to maintain asset turnover. It would appear that the hotel business division is performing poorly both in terms of cost control and use of assets, and each of these areas requires detailed investigation. The third division other drinks and leisure is making a contribution to profit of 18m but performance is mediocre by all measures, including a return on net assets of just 4.5%.

(ii) In order to interpret effectively the performance of the company, the results achieved by each division need to be compared with its direct competitor. The beer and pub division achieved a return on assets higher than that of its competitor, Dean. The beer and pub division has achieved a far higher profit margin that has more than compensated for the marginally inferior asset turnover. Filios hotel business division performs poorly compared with its competitor, producing a return on assets of not much more than half of that achieved by Clarke. The divisions profit margin and rate of asset turnover are both lower than those of its competitor.

290 Pearson Education Limited 2006

Barry Elliott and Jamie Elliott: Financial Accounting and Reporting (tenth edition) Instructors Manual

The performance of each of the two divisions for which competitor information is available is not encouraging. The position is even worse when it is recognised that there are unallocated common costs of 15m and interest of 14 million. All the indications are that Filios is not being managed in the most effective manner. The rate of return on assets earned by Filios Products as a whole (6.5%) is marginally better than that achieved by its competitor Dean, but far below that of Clarke.

291 Pearson Education Limited 2006

Barry Elliott and Jamie Elliott: Financial Accounting and Reporting (tenth edition) Instructors Manual

Chapter 28: Question 7 Chaldon District Council


Report To From Date Subject Client Services Committee Accountant Roofing Contract: Financial Appraisal of Tenderers

Introduction

1.1 Four tenderers, including CDS, have been short-listed for appraisal. The tenders have been submitted by: Tender A B C D Name Nutfield & Sons Chaldon Direct Services (CDS) Tandridge Tilers Limited Redhill Roofing Contractors plc

Objective to determine to whom the roofing contract should be awarded. Basis of appraisal Tenderers will be appraised on financial and qualitative grounds. Accounting ratios will be employed to assess profitability, solvency (long and short term), speed of cash collection and payment. Details are provided in Appendix A. Reference to limitations of approach: analysis is indicative only, not definitive analysis is based on historical information need for several years figures in order to consider trends.

Interpretation of ratios

2.1 Profitability Despite having the lowest profit margin, As ROCE is the highest at 77% due to its very high asset turnover of 13 times per annum. This is probably a reflection of the nature of the business a small family concern; this probably also accounts for the firms relatively low stockholding. The asset turnover of the other two companies is similar, and Cs higher ROCE is due to its higher margins. Ds stock turnover is considerably higher than its competitors which could be a cause for concern. 2.2 Long-term security A has no long-term debt and, therefore, does not bear any interest charges. The other two companies are highly geared with Cs long-term debt being equivalent to its equity finance which is a cause for concern.

292 Pearson Education Limited 2006

Barry Elliott and Jamie Elliott: Financial Accounting and Reporting (tenth edition) Instructors Manual

2.3 Short-term security D has the best current ratio, although the quick ratio of the three businesses is similar, C having the lowest. Interest cover is only relevant for C & D and does not appear problematic in either given current profit levels. 2.4 Cash flow ratios A takes longer to settle its creditors and collect from its debtors than the other two companies whose ratios are similar.

3 Other factors
3.1 An analysis of the make-up of the tenders is as follows
Labour % A B C D 59 63 75 57 Materials % 35 25 20 34 Overheads % 6 12 5 9

The variation between the components of the various tenders does not provide for any meaningful comparison, although CDS (B) does have the highest proportion of its bid for overheads and profit. Nutfield and Sons (A) have been employed by the Council for small contracts which they have performed satisfactorily. However, this contract is substantially larger than others they have won and, given a workforce of only six, they may not be able to fulfil a contract of this scale. CDS (B) is obviously well known to the authority and its management have striven to improve its financial position recently in order to achieve a satisfactory rate of return this year. Tandridge Tilers Limited (C) have not performed satisfactorily on other contracts that they have carried out for the Council. Redhill Roofing Contractors plc have not been employed by this authority and the standard of their work is not known.

Conclusions

Although the financial standing of Nutfield and Sons (A) does not give cause for concern, and although it has submitted the lowest tender bid, there are doubts as to whether it is capable of carrying out a contract of this scale. Include a comparison of uses in private and public sector. Main points should include:

293 Pearson Education Limited 2006

Barry Elliott and Jamie Elliott: Financial Accounting and Reporting (tenth edition) Instructors Manual

Private sector Weaknesses of historical cost (HC) accounts in times of high inflation; undervaluing assets; overstating profits; not providing for maintenance of capital. Can give a better indication of actual profits earned, separates holding gains from earned profits. Can give better indication of value of individual assets to the business. Based on concept of providing useful information for users of accounts. Not accepted in public sector; accountants not able to agree on bases and methods of adjustments required, or capital maintenance to use.

Public Sector Need to show effective use of public assets. Real-terms measure seen as more appropriate: Many public sector organisations have very long lived assets, HC is particularly misleading as result. Financial objectives of many public sector bodies are stated in real terms and test discount rates used to evaluate capital projects based on real rates of inflation.

HC is objective and services stewardship function. HC can provide information to enable users of accounts to make their own adjustments, comparisons etc., but fuller disclosure of information would be required. There is a wide range of external information available to users of private sector company accounts. This is not the case with many public sector bodies. CCA-adjusted figures argued to be more useful bases of assessing performance. Does this imply the government views performance evaluation as more important for public sector bodies than investors do for private companies? CDS (B) submitted the second lowest tender. There is no reason to suspect that it will not be able to deliver the contract to the appropriate standard. The longer-term financial security (gearing) of Tandridge Tilers Limited (C), the second highest tenderer, and the quality of its work give major causes for concern. The highest bid was submitted by Redhill Roofing Contractors plc and, although its financial standing does not cause concern, its quality is unknown.

294 Pearson Education Limited 2006

Barry Elliott and Jamie Elliott: Financial Accounting and Reporting (tenth edition) Instructors Manual

Recommendation

5.1 It is recommended that the contract be awarded to CDS.


Appendix A: Accounting ratios Profitability A Return on capital employed (%) (ROCE) Profit margin (%)(PM) Asset turnover (times p.a.)(AT) Stock turnover (days) (ST) Long-term solvency Gearing (%) 0.0 50.0 37.0 77.1 6.0 12.8 17 B 21.2 19.5 1.1 46 C 16.5 17.3 1.0 94

Short-term solvency Interest cover (times) Current ratio Quick ratio Cash flow ratios Creditors settlement period (days) Debtors settlement period (days) Notes ROCE PM AT ST Gearing Interest cover Current ratio Quick ratio Creditors settlement period Debtors settlement period = = = = = = = = = =

n/a 0.9:1 0.7:1 59 41

4.0 1.3:1 0.6:1 37 27

5.6 1.9:1 0.7:1 40 29

(Operating profit/net assets) 100 (Operating profit/sales) 100 (Sales/net assets) (Stock and WIP 365)/direct costs (Non-equity finance/equity finance and non-equity finance) 100 Net profit before tax and interest/interest payable Current assets: current liabilities (Current assets stock and WIP): current liabilities (Creditors 365)/operating costs (Debtors 365/sales)
A 000 C 000 1,741 339 1,600 800 D 000 3,080 534 3,241 1,200

Turnover Operating profit Net assets (total assets less Current liabilities) Non-equity finance

612 37 48

295 Pearson Education Limited 2006

Barry Elliott and Jamie Elliott: Financial Accounting and Reporting (tenth edition) Instructors Manual

Equity and non-equity finance Interest payable Direct costs Stock and WIP Operating costs Current assets Current liabilities Current assets excl. stock Creditors Debtors

48 410 27 575 96 104 69 93 69

1,600 85 1,191 149 1,402 290 232 141 141 131

3,241 96 1,735 449 2,546 690 356 241 280 241

296 Pearson Education Limited 2006

Barry Elliott and Jamie Elliott: Financial Accounting and Reporting (tenth edition) Instructors Manual

Chapter 28: Question 8 Chelsea plc


(a)
Profitability: ROCE Wimbledon outperformed: the industry by 8% and Kensington by 6%. Wimbledon follows a high volume/low profit pricing policy Low profit evidenced by extremely low profit margin. High volume evidenced by the asset turnover figures of 12 and 4 for fixed assets and total assets. Profit margins:

Kensington and the industry, in contrast, have achieved 2.3 and 1.5; and 5.1 and 2.5 respectively. Kensington perhaps moving up market with lower volume/ higher margin.

Cost control: Wimbledons 7% (12 5) shows lower overhead costs as a percentage of sales compared with Kensington and industry averages of 14% and 13%.

Liquidity: Wimbledon has a lower debtor collection period and stockholding period suggests better working capital management than in Kensington. Kensingtons acid test ratio of 0.5 appears low compared with 0.9 in Wimbledon and the industry average of 1.3. This appears dangerously low when taking into account the long debtor collection period.

Overall, Wimbledon appears the better investment: making better use of assets better cost control well managed working capital potential for borrowing to gear up return on equity is healthy.

(b)

Matters to be investigated before a final decision can be made:


Check if activities of companies are actually similar. Obtain the absolute figures () for turnover, profits, assets etc. Determine unexpired economic lives of fixed assets in each company. Check quality of management and confirm whether likely to remain. Confirm managements strategy increased markets or diversification. Obtain details of date of redemption of debt.

297 Pearson Education Limited 2006

Barry Elliott and Jamie Elliott: Financial Accounting and Reporting (tenth edition) Instructors Manual

Chapter 28: Question 9


(a)
North is achieving a higher profit margin than South or East, with East achieving the lowest margin at 3%.
North (i) Profit/Sales 100 (ii) Asset turnover ROCE (i) (ii) 5% 5 times 25% South 4% 3 times 12% East 3% 4 times 12%

Whilst North has the highest profit to sales, the effect of the differences in the rate of asset turnover on the comparative performance of South and East means that their ROCE is the same at 12%. The ROCE indicates that North is performing better than South and East which have the same return of 12%. However, there are different levels of gearing as shown by the financial multiplier and when this is taken into account the position is as follows:
North (i) Profit/Sales 100 (ii) Asset turnover ROCE (i) (ii) Financial leverage ROE 5% 5 times 25% 2 50% South 4% 3 times 12% 4 48% East 3% 4 times 12% 5 60%

For a lender North has the lowest gearing and less risk (assuming that there are no other contra-indications such a s solvency or liquidity problems), for a minority investor East shows the highest ROE. However, as a measure of management performance, although East has the highest ROE it is underperforming at an operational level. If it were to achieve Norths performance (perhaps under new management) then its ROE would increase to 175% (5% margin 5 times asset turnover 5 times financial multiplier).

(b)
Pros:

Decision usefulness of consolidated accounts

Consolidated accounts give an overview of the groups results and financial position. Shareholders in parent company can see how their funds have been invested:

298 Pearson Education Limited 2006

Barry Elliott and Jamie Elliott: Financial Accounting and Reporting (tenth edition) Instructors Manual

How much of the net assets belongs to the minority shareholders. The amount of profit attributable to their shareholding. The amount of profit expressed as EPS with implication for share price movements.

Impossible for a shareholder in parent of a complex group to obtain such information without group accounts which are prepared: using uniform accounting policies across all group companies eliminating inter-group transactions.

Cons: Group accounts might combine very disparate companies with different levels of profitability, liquidity and risk profiles. Detailed information on an individual company may be disguised e.g. excessive gearing. Some group companies may be making losses this will only become apparent if the parent decides to sell the loss-making subsidiary and it is reported under discontinued operations. Also not possible to identify any extremely profitable subsidiary although this might become apparent from the segmental report. The volume of intra-group trading and intra-group indebtedness by each company will not be apparent. Where there is a minority interest, unrealised profit on intra-group sales will be eliminated 100% although it could be considered that the proportion relating to the minority interest has been realised.

299 Pearson Education Limited 2006

Barry Elliott and Jamie Elliott: Financial Accounting and Reporting (tenth edition) Instructors Manual

Chapter 28: Question 10


PROFILE 31/07/2004 12 months Turnover Profit (Loss) before Taxation Net Tangible Assets (Liab.) Shareholders Funds Profit Margin (%) Return on Shareholders Funds (%) Return on Capital Employed (%) Liquidity Ratio Gearing Ratio (%) 0.22 143.19 0.24 121.51 0.24 120.96 0.29 110.06 6.83 8.24 8.11 7.70 5.88 16.03 7.68 17.62 8.91 17.27 9.16 16.18 288,954 318,628 310,133 273,839 677,710 680,989 660,447 575,223 787,126 46,316 31/07/2003 12 months 730,913 56,139 31/07/2002 12 months 601,295 53,568 31/07/2001 12 months 483,968 44,317

300 Pearson Education Limited 2006

Barry Elliott and Jamie Elliott: Financial Accounting and Reporting (tenth edition) Instructors Manual

CHAPTER 29

Chapter 29: Question 1 Wandafood Products plc


Profit to sales was low in 207. Decline in return on assets in 207. Consider the profit to turnover and asset turnover movements. Interest and dividend cover is falling from 206. Further decline could create problems. There is an upward move in the level of borrowings. Debt to equity as indicated by the relationship between the net borrowings and the sum of net borrowings and shareholders funds is 1:10 in 206 and 1:2.5 in 209 indicating increased borrowing. Liquidity as indicated by the trend in the liquid and current ratios is levelling out.

Asset ratios Sales to working capital ratio shows a steadily increasing trend. May indicate more efficient use of working capital. Assets per share are increasing indicating that the company is ploughing back profits.

301 Pearson Education Limited 2006

Barry Elliott and Jamie Elliott: Financial Accounting and Reporting (tenth edition) Instructors Manual

Chapter 29: Question 2 Bouncy plc


(a) Ratios for a potential shareholder
206 (i) Return on equity Profit after tax and Preference dividends /Ordinary share capital + Reserves (ii) Earnings per share (iii) Dividend cover Profit after tax and Preference dividends/No.of Ordinary shares Equity profits/Proposed dividend 1,300/6000 = 21.67p 1,300/250 = 5.2 times (iv) Gearing Debt capital/Debt + Equity 1,500/8,200 = 18.3% 900/6,000 = 15p 900/250 = 3.6 times 1,500/7,150 = 21.0% 1,300/6,700 = 19.4% 205 900/5,650 = 15.9%

(b) Solvency ratios for a potential lender


(i) Debt equity (ii) Solvency (iii) Interest cover (iv) Liquidity Debt:Equity Current assets:Current liabilities Profit before interest:Interest 1,500:6,700 = 1:4.5 3,810:1,960 = 1.9:1 2,200/170 = 13 times 1,500:5,650 = 1:3.8 3,610:2,060 = 1.8:1 1,570/150 = 10 times 1,540:2,060 = 0.75:1

Current assets Stock:Current li- 1,710:1,960 abilities = 0.87:1

(c) Comments from potential shareholders viewpoint


The return on equity has improved by approximately 25%. The dividend is well covered and has improved in 206 from 3.6 in 205 to 5.2 in current year. The EPS figure is in line with the return on equity and is acceptable. The gearing is low at 18.3% so that the business enjoys lower earnings risk. Comments from viewpoint of lender The current ratio at 1.9 and acid test ratio at 0.87 are both improving and interest is well covered at 13 times. Gearing is low and coupled with the improving return on equity and sound interest cover means that the company is able to increase its long-term borrowing. The increase in the share price over the last three years is understandable given the picture presented by the ratios.

302 Pearson Education Limited 2006

Barry Elliott and Jamie Elliott: Financial Accounting and Reporting (tenth edition) Instructors Manual

(d) Advising on scheme to choose


It is interesting to assess the schemes from their impact on earnings per share and return on equity. Assuming a rights issue
000 Profit before interest and tax Interest expense currently Less: Debenture interest (10% of 1.5m) Bank charge interest Taxation Loss of interest allowance 40% of 150,000 Revised profit after tax Earnings per share: Shares in issue 3,000,000/0.5 = New shares 6,000,000/1.5 = 6,000,000 4,000,000 10,000,000 EPS = 1,390,000/10,000,000 Return on Equity = 1,390/(6,700+6,000) 100 13% Debentures 000 Profit before interest and tax Interest expense (6,000 13%) Taxation Less tax savings on loan interest (780 170) 40% Revised profit 730 (244) 486 934 000 2,200 (780) 1,420 = 13.9p = 11% (730) (60) (790) 1390 (170) 150 (20) 2180 000 2200

EPS = 934/6,000 = 15.6p Return on Equity = (934/6,700) 100 = 14% The decision based on EPS and return on equity supports the loan funding scheme. Other factors to be taken into account: Consider the increase in gearing from 18.3% to 47.2% (6,000/12,700)
303 Pearson Education Limited 2006

Barry Elliott and Jamie Elliott: Financial Accounting and Reporting (tenth edition) Instructors Manual

Chapter 29: Question 3 Liz Collier


(a) Option 1
Profit [21,000 140/100] Year 1 29,400 1,000 28,400 Less interest at 10% per annum Year 2 29,400 1,000 28,400

Comments: (i) The loan of 10,000 is paid out of incremental cash flow generated by profit in 14.5 months. (ii) It is likely that some benefit will continue after the end of year 2 and marginally improve her lifestyle. (iii) It is assumed that the 40% increase is a reasonable and feasible forecast. Option 2
Partnership profit [profit is 33,000 120/100] Less cost of amalgamation Less salaries Liz 2% of 126,000 Joan 2% of 72,000 Profit share: Liz 3/5 Joan 2/5 Comment: (i) Liz will receive Salary Profit share 2,520 17,262 19,782 2,520 21,384 23,904 (17,262) (11,508) (21,384) (14,256) (2,520) (1,440) 28,770 (2,520) (1,440) 35,640 (6,870) Year 1 39,600 Year 2 39,600

Liz is worse off in year 1 by 1,218 [21,000 19,782] and better off by 2,904 in year 2. Her share of the initial investment is 4,122 i.e. 3/5 of 6,870. This investment will be repaid in 2.5 years and the benefit will accrue in perpetuity. From year 2 onwards it generates a ROCE of 70% i.e. 2,904/4,122. It seems a good proposal assuming the figures are reliable and that the partners are able to work in harmony. There is potential for expansion with synergy effect.

304 Pearson Education Limited 2006

Barry Elliott and Jamie Elliott: Financial Accounting and Reporting (tenth edition) Instructors Manual

Joan will derive a benefit in year 1 and a higher return in subsequent years i.e. 3,696/2,748 100 = 134% in year 2. This might indicate that the profit-sharing ratio is unfair to Liz and should be reviewed if this option is selected.
Option 3 Profit [21,000 8/10] Franchise profit Less interest @ 10% per annum Total profit 23,800 Year 1 16,800 15,000 (8,000) 26,050 Year 2 16,800 17,250 (8,000)

Comments: (i) Incremental profit compared with present position is: (ii) Franchise projected profit: Year 3: 19,838; Year 4: 22,813; Year 5: 22,813 It will take 6 to 7 years to repay 80,000 from incremental cash flows. After year 8 it could be a very profitable proposition. (b) Option 1 gives a 40% increase over two years. It is unlikely that this increase can be maintained in year 3 and subsequent years without additional expense on advertising etc. The initial outlay is moderate and is repaid quickly from additional cash flow. Liz will maintain her independence and improve somewhat her standard of living/lifestyle. Option 2 shows a reduction in profit in year 1 compared with the present and a 2,900 increase thereafter in year 2 and subsequently. The initial outlay is moderate and there may be longerterm prospects without additional expense after year 3. There is however a loss of independence as a partner. There may be hidden costs not provided for and high opportunity costs. Option 3 requires substantial investment of 80,000 which may be repaid until about year 7 out of incremental cash flows. (c) Reservations Option 1 The ability to increase turnover by 40% and the maintenance of the level of sales after year 2. 2,800 5,050

Option 2 The ability to work amicably with Joan in the partnership. Risk of poor decisions by the other partner which then bind the firm. Possibility of administration costs not included in the estimates given. Basis of profit-sharing ratio seems to be biased in favour of Joan.

305 Pearson Education Limited 2006

Barry Elliott and Jamie Elliott: Financial Accounting and Reporting (tenth edition) Instructors Manual

Option 3 Need to reduce existing sales. Involvement with franchise constitutes a refocusing of the business with attendant risks. Reliability of the estimates particularly after the first 2 years.

306 Pearson Education Limited 2006

Barry Elliott and Jamie Elliott: Financial Accounting and Reporting (tenth edition) Instructors Manual

Chapter 29: Question 4 Chekani plc


(a) Principles of share valuation:
No one method of valuing shares. Accounting values only a guide as to starting point and/or maximum value for negotiating between parties. Share value is the amount that the buyer is willing to pay and the vendor willing to accept.

Methods or bases of valuation: Dividend yield basis. Price/earnings ratio. Net asset value based on a going concern or break-up value. Valuation of an unlisted company of this type needs to take account of the difficulty associated with selling the shares and the increased risk of the investment, by adjusting quoted company yields or PE ratio. Minority holdings usually valued on dividend yield basis, which is not appropriate for majority holdings of this type. Majority holdings valued on PE basis, using adjusted quoted company PEs; the net asset value, as a going concern, gives a minimum valuation.

(b) Alternative valuations (see appendix for calculations)


Dividend yield basis. For calculations, see appendix. Discussion of investors required rate of return, quoted company yields and possible uplift % for an unlisted company. Appendix shows 1.28 (with 30% adjustment for unquoted status). Method not appropriate here. Net asset basis For calculations, see appendix. Discussion of treatment of: contingent liability purchased goodwill unamortised book value v current market value of assets relating goodwill and current market values.

307 Pearson Education Limited 2006

Barry Elliott and Jamie Elliott: Financial Accounting and Reporting (tenth edition) Instructors Manual

Preference shares These could be deducted at their nominal value as the liability due to outside interests. Alternatively, the better student could argue that the approach to be adopted should be to value the company as a whole, e.g. Gross asset value, the amount Chekani would pay for the whole enterprise, including the preference shares, and then to deduct the value of the preference shares, i.e. total value less the amount that will not be acquired.

Debentures Either the premium is an extra cost to Chekani and is ignored in the valuation, or it is included on the basis that it is part of the value of the companys net assets.

Earnings basis Calculations per appendix Comment on use of P/E ratios for quoted companies and discount rate applied. Treatment of exceptional item, maintainable earnings.

(c) Appropriate strategy


Normally do not sell below net asset value, based on revalued amounts. This is the highest price here, 3.02, and may not be acceptable to the purchaser due to factors such as: falling profits over last three years, return on investment may be more important than asset value value includes estimate of current values which may not be realisable future prospects and forecasts are not given and cannot be taken into account, but they could provide evidence to support a higher (upturn in prospects expected) or lower (no improvement or downturn forecast) price.

The final price is likely to be between the two net asset basis prices, the revalued asset basis figure 3.02 and the value based on net assets per balance sheet, 2.175. Opening bid should be around the higher figure, say 3.00, leaving room to negotiate downwards, possibly with a view to agreeing a final price around 2.60.

Appendix
Dividend yield
Net dividend on ordinary shares Dividend per share 3 million/40 million 3m 7.5 pence

Investors required rate of return, based on average gross yield of the two quoted companies given, (4.9 + 4.1)/2 = 4.5%
308 Pearson Education Limited 2006

Barry Elliott and Jamie Elliott: Financial Accounting and Reporting (tenth edition) Instructors Manual

Value per share: No allowance for different risk (1 7.5/4.5) = 1.67 per share Adjusted yields, allowing for increased risk of unquoted share, using 30% (Note: other adjustment % are permissible), (4.5% 1.3) = 5.85%, so 1 7.5/5.85 = 1.28 per share.
Asset valuation basis Assets at balance sheet values Goodwill Property Plant Investments Net current assets Contingent liabilities 10% Debentures Preference shares 15,000 30,000 60,000 15,000 12,000 (3,000) (30,000) (12,000) 87,000 Value per share Assets revalued/ goodwill written off etc. 0 56,250 60,000 22,500 12,000 (3,000) (33,000) (10,800) 103,950

87,000/40,000 = 2.175 103,950/40,000 = 2.60

Other variations could be: Goodwill not written off 103,950 + 15,000 = 118,950 118,950/40,000 = 2.97 Debentures and preference shares at nominal value Goodwill not written off and debenture/preference at nominal value (maximum valuation of assets) 103,950 + 1,800 = 105,750 105,750/40,000 = 2.64 103,950 + 15,000 + 1,800 = 120,750 120,750/40,000 = 3.02

309 Pearson Education Limited 2006

Barry Elliott and Jamie Elliott: Financial Accounting and Reporting (tenth edition) Instructors Manual

Valuation on an earnings basis Capitalised using PE ratio of similar listed companies: Average of 11.3 and 8.2 = 9.75 Earnings: Profit before interest and tax Interest Tax Preference dividends EPS = 11,610/40,000 = 29.025 pence per share 29.025 9.75/100 = 2.83 Discounted 29.025 6.825/100 = 1.98 21,000 (3,000) 18,000 5,550 12,450 840 11,610 Discount at 30% = 6.825

310 Pearson Education Limited 2006

Barry Elliott and Jamie Elliott: Financial Accounting and Reporting (tenth edition) Instructors Manual

Chapter 29: Question 5 Johnson Products Ltd


(a) (i) Sale of shares to Sonar Products Ltd
The 75% holding constitutes a controlling interest and can be valued on an earnings basis to indicate the amount that the buyer could offer and reasonably assume would be acceptable to the seller. A valuation on an earnings basis gives a value of approximately 300,000 or 37p per share. The value is computed using the following formula: Value = Earnings/% earnings yield required For this part of the question we need to estimate the amount of the earnings that are to be capitalised and the percentage earnings yield required from the information given in the question. The earnings could be based on the final year figure or perhaps a weighted average. For the purpose of illustration the weighted average is being used in this solution, calculated as follows:
Earnings 79,400 (27,600) 56,500 88,300 97,200 1 2 3 6 Average earnings = 524,700/6 = 87,450 56,500 176,600 291,600 524,700 Weight Product

Note that in part (c) of the question there is a further discussion required of the principal matters that need to be taken into account when assessing future maintainable earnings. The percentage earnings yield required is based on the information provided in the question about the three other companies:
Gross dividend % yield Eastron Westron Northron 15 10.5 13.4 Retention % 25 16 20 Earnings % yield 20 12.5 16.75

The average percentage earnings yield = (20 + 12.5 + 16.75)/3 = 16.4%.

Based on the estimated average earnings which are regarded as maintainable and the estimated percentage earnings yield required, the valuation of the 75% shareholding is as follows:

311 Pearson Education Limited 2006

Barry Elliott and Jamie Elliott: Financial Accounting and Reporting (tenth edition) Instructors Manual

Value of company Value of 75%

= 87,450 100/16.4 =

533,232 399,924 99,981 299,943

= 533,232 75/100 =

Less say 25% for lack of marketability

This value is an estimate of the amount that would be acceptable to R. Johnson.

(ii) Sale of shares to the staff


The possible sale to the staff would result in a widely held share capital with no single person holding in excess of 4% of the share capital. Consequently it is felt that the shares should be valued on a dividend basis using the formula that the value of a share would be the dividend divided by the percentage dividend yield required less 25% for lack of marketability. The dividend is assumed to be 5p per share and the percentage dividend yield required is estimated at 12.97 being the average of the yields for the three comparator companies.
The value of a share = 5/12.97 100 = Less 25% Value per share Value of 810,000 shares 38.55p 9.64 28.91 234,171.00

(iii) Sale to Divest plc


The realisable value of the business is: Land Premises Equipment Stock Debtors Cash Creditors Non-current creditors Realisable value Less 16.7% (based on the need to obtain 20% return) Value of business Value of 75% The three values are therefore: (i) (ii) (iii) Sale to Sonar Products Ltd Sale to minority interests Sale to Divest plc 299,943 234,171 689,375 480,000 630,000 150,000 98,000 168,000 70,000 (335,000) (158,000) 1,103,000 183,833 919,167 689,375

312 Pearson Education Limited 2006

Barry Elliott and Jamie Elliott: Financial Accounting and Reporting (tenth edition) Instructors Manual

(b)

Maximum that would be offered by Sonar Products Ltd on basis of information provided in the question

The valuation would be calculated on a return on capital basis to indicate the maximum amount the buyer would be prepared to offer. The maximum that Sonar Products would be prepared to pay may be estimated by reference to the rate of return that they presently achieve. Given that they currently achieve a rate of return on capital employed of 12.5% the amount they would regard as maximum is 524,700 calculated as follows:
Average earnings 87,450 Capitalised at % return on capital (100/12.5) (75/100) = 524,700 % Holding Maximum value

(c)

Principal matters to take into account when estimating future maintainable earnings

There are a number of matters that could be mentioned and in this answer a selection of relevant matters is given. There are others that could be put forward as satisfactory replies to this question.

(i)

Past performance
Past performance, i.e. past earnings, is the main indicator of future potential. One cannot merely carry out an extrapolation of the past three to five years But it is an indication of how well the company has operated in the past in comparison with other companies within the same industry. This means that one would need to obtain information about the earnings of the three comparator companies over say the past three to five years and assess how well Johnson Products Ltd has fared in comparison with these. One could pay attention to the compound annual growth rates in sales and operating profits and profits for the year, and look at the implication of financial and operating gearing.

(ii)

Forecast for the industry


It is important to form a view on the possible growth or decline within the industry sector in the future. Although the past earnings are the base from which accountants start, they also needs to have regard to the expected movements within the industry in the future. We are attempting to estimate future maintainable earnings and clearly the rate of growth in the industry is important.

313 Pearson Education Limited 2006

Barry Elliott and Jamie Elliott: Financial Accounting and Reporting (tenth edition) Instructors Manual

(iii)

Changes in the activities undertaken

The activities that generated the past earnings will be known. It is important to identify the extent to which these will be varied. There are various indicators that will be apparent from an examination of the accounts themselves, such as research and development expenditure new fixed assets capital investment contracts outstanding at the balance sheet date, and even surplus funds that are not currently invested within the business because they indicate the capacity to move into new activities or to expand the level of existing operations.

(iv)

Rationalisation

Consideration needs to be given to the likelihood of the acquirer selling off parts of the acquired company in order to improve performance or to release cash for the payment of interest or for other purposes.

(v)

Management and staff


These are an important component for success in any business. It is possible to gain an impression from the accounts and filed documents of average wage levels and the stability of the board. However, to obtain more detailed information, it would be necessary to have the cooperation of the company because one would be seeking more detailed information on service contracts performance-related pay the rate of labour turnover. It is clearly more fruitful if it is possible to obtain the co-operation of management to obtain these data.

(vi)

Accounting policies

If it is assumed that the new owners will be able to control the accounting policies then clearly it is of interest to identify how the past policies will be varied. For example, there are the areas such as depreciation and long-term contracts where the company might follow a more or less conservative accounting policy.

314 Pearson Education Limited 2006

Barry Elliott and Jamie Elliott: Financial Accounting and Reporting (tenth edition) Instructors Manual

(vii)

nterim accounts

If it is possible to obtain access then the interim accounts, management accounts, budgets and forecasts will give an indication of the companys strategy and its success over the immediate past few months. This could give a more current feel for the companys progress.

(viii) Ratio analysis of balance sheet


The matters referred to in paras (ii)(vii) are specifically towards the future. Whilst the emphasis is on the future we also need to refer to the last balance sheet to pick up items such as high gearing, poor liquidity references to post balance sheet events, or contingent liabilities that might impact on the future prospects of the company. In conclusion therefore the question is looking for a recognition that the valuer needs to be forward looking, identifying as clearly as possible the future maintainable earnings.

315 Pearson Education Limited 2006

Barry Elliott and Jamie Elliott: Financial Accounting and Reporting (tenth edition) Instructors Manual

Chapter 29: Question 6 Business Risk Management


(a) Identification and prioritisation of risks

All types of risk are relevant to an existing or potential shareholder including both downside risks (possible losses) and volatility risks (possible gains or losses). Shareholders are not protected if they only receive details of downside risks and sell their shares inappropriately. Developments to date have been aimed at addressing a particular problem e.g. SSAP 25 Segmental Reporting. This has meant that companies have had prescriptive requirements which might not have reflected the actual risks which are relevant to their company. Risk that may be relevant include: product or service failure new regulations product development with heavy R&D costs before cash flows in.

Internal risks include: process risks e.g. arising from employees such as risk of losing key staff, suppliers and manufacturing process whereby products are not delivered on time or to correct specification. financial risks e.g. price, liquidity and credit risks.

External risks include: social, political and economic forces, e.g. risk of new employee protection regulations financial risks, e.g. exchange rate movements.

Risk prioritisation The normal materiality criterion applies and attention should be drawn to risks in accordance with their significance.

(b)

Managing risk

There are different views on the nature of the disclosure. One view is that it is sufficient to confirm that the company has complied with the Combined Code. There is also the view that there should be detailed disclosure of particular steps taken e.g. insurance, hedging, outsourcing.

(c)

Measuring risk

There is a wide range of measures that could be applied to measuring risk and it is important not to concentrate only on deterministic data e.g. potential losses on exchange, but also to consider how to report on strategic risks.

316 Pearson Education Limited 2006

Barry Elliott and Jamie Elliott: Financial Accounting and Reporting (tenth edition) Instructors Manual

Accounting measures already exist internally, e.g. reporting provisions and contingencies and producing ratios such as gearing and liquidity, trend analysis and benchmarking. Accounting measures also exist externally, e.g. bond rating by credit agencies, benchmarking. Non-accounting measures are also important, e.g. price competitiveness, delivery times, level of warranty claims.

317 Pearson Education Limited 2006

Barry Elliott and Jamie Elliott: Financial Accounting and Reporting (tenth edition) Instructors Manual

CHAPTER 30

Chapter 30: Question 1


The IFRS Taxonomy has the element Inventories identified as bearing the name of Inventories, being of the type monetaryItemType and that it is expected to have a debit balance
<element id="ifrs-gp_Inventories" name="Inventories" type="xbrli:monetaryItemType" substitutionGroup="xbrli:item" xbrli:periodType="instant" xbrli:balance="debit" nillable="true" />

As the explanations in this text do not go in detail as to the different structures of all the files used in XBRL, it is not expected that students would be able to reproduce this type of coding. Students may have visited the example at http://www.xbrl.org/Example1/ and come up with the following:
<ifrs-gp:Inventories ContextRef=Current_AsOf UnitRef=U-Euros Decimals=0>100000</ofrs-gp:Inventories>

Although this is an example of XBRL works, the coding really is from an Instance Document and not from the underlying schemas etc. If the students followed the naming conventions used (see http://xbrl.org.au/training/ XBRLNamingConventions.pdf) then it is probably more likely that students come up with:
CurrentAssets Inventory Or: CurrentAssets CashAndCashEquivalents Inventory

The placement of Inventory depends thus on the standards position as to Inventory belonging to the class of Cash and Cash Equivalents or to Other Current Assets. Agricultural assets may well be split between Current and Non Current (Timber Trees or plantations) Another good overview example of the application of the IFRS can be found at http://xbrl.iasb.org/int/fr/ifrs/gp/2005-01-15/Samples.htm

318 Pearson Education Limited 2006

Barry Elliott and Jamie Elliott: Financial Accounting and Reporting (tenth edition) Instructors Manual

Chapter 30: Question 2


List the steps in the approval process for a taxonomy to be considered to have final status. (Hint: look at www.xbrl.org ) Students would have looked at http://www.xbrl.org/FormingAJurisdiction/ and perused the zip file and found that there are 2 different classifications for Jurisdictions as listed in the table below:
Established Jurisdiction A non-profit organisation, involved in business reporting, that collects dues from ten or more members and pays dues to XBRL International. Provisional Jurisdiction A non-profit organisation involved in business reporting that has a group of individuals (often called a working party) creating an XBRL Taxonomy or other XBRL work products, and that pays dues to XBRL International.

Jurisdictions are normally established within a country where a particular accounting standard is applicable. Should a country adapt the IFRS then, technically, it may not need to form a jurisdiction. Should there be a need to adapt (XBRL refers to this as extend) then there will be a need if the country needs to use XBRL.

319 Pearson Education Limited 2006

Barry Elliott and Jamie Elliott: Financial Accounting and Reporting (tenth edition) Instructors Manual

CHAPTER 31

Chapter 31: Question 1 BC


(a) Provision at 31.12.0 = no. of shares option price probability of achieving the option 1/3 = 1,000,000 0.05 0.6 1/3 = 10,000 Provision at 31.12.1 = no. of shares option price probability of achieving the option 2/3 = 1,000,000 0.10 0.7 2/3 = 46,667 Provision at 31.12.2 = no. of shares option price probability of achieving the option = 1,000,000 0.15 0.85 = 127,500 Provision at 31.12.3 = no. of shares option price probability of achieving the option = 1,000,000 0.30 1.0 = 300,000 Final cost at 30.6.4 = no. of shares option price probability of achieving the option = 1,000,000 0.35 1.0 = 350,000
Year ended 31 Dec 200 31 Dec 201 31 Dec 202 31 Dec 203 31 Dec 204 Total charge Provision 10,000 46,667 127,500 300,000 P&L a/c charge 10,000 36,667 80,833 172,500 50,000 350,000

There will be no provision in the financial statements for the year ended 31 December 204, as the option will have been awarded to the director. The total cost of the option at 30 June 204 will be charged as directors remuneration and credited to the share premium account.

320 Pearson Education Limited 2006

Barry Elliott and Jamie Elliott: Financial Accounting and Reporting (tenth edition) Instructors Manual

(b) The total cost of the share option using the different methods will be:
Method IAS 37 US FAS123 & IFRS 2 Total cost 350,000 30,000

UK recommendation

300,000

(c) (i) The IAS 37 method gives a cost for the option which is consistent with the International Accounting Standard and the IASCs Framework for the Preparation and Presentation of Financial statements. Taking the value of the option and the probability of the option being granted tends to give an uneven charge for the option, with a higher charge in later years. This example tends to exaggerate this uneven charge, but the charge will tend to be higher in later years because the probability of the option being granted is higher near the end of the option entitlement period (and less in earlier years). Also, there is an element of discounting in determining the option price, so the option price will be lower in earlier years than later ones. In addition, the option price will be lower in earlier years as it is less certain that the share price will be above the option price in earlier years than later ones. (ii) In this example, FAS 123/IFRS 2 gives a charge to the profit and loss account of only 8.6% of the cost of the IAS 37 method (and only 10% of the UK recommendation). This charge occurs in the year ended 31 December 200, and there is no subsequent charge in the profit and loss account. (iii) The UK recommendation gives a charge of 86% of the IAS 37 method, which is a more realistic figure than using the second method. However, the charge only occurs at 31 December 203, which is 3 years after the option was granted to the director. It would seem more appropriate if the charge was made during the period the director earned the entitlement to the share option.

321 Pearson Education Limited 2006

Barry Elliott and Jamie Elliott: Financial Accounting and Reporting (tenth edition) Instructors Manual

Chapter 31: Question 2 Rolls Royce


(a) The composition of the Remuneration Committee is appropriate as it comprises entirely non-executive directors. It is good that they meet regularly and receive appropriate professional advice. However, the statement that they meet regularly is vague and could be more specific, stating the number of meetings held in the year ended 31 December 1999. There appears to be a slight conflict in that the Chairman and Chief Executive attend the meetings. They are virtually members of the Remuneration Committee. However, the Remuneration Committee need relevant information on the performance of the executive directors, and one good way of obtaining this is from the Chairman and Chief Executive. Also, the Chairman and Chief Executive may have relevant information on comparable salaries for directors in other similar companies. However, there may be problems with the Chairman and Chief Executive attending the meeting, as the Remuneration Committee may want to discuss information provided by the Chairman and Chief Executive but with both the Chairman and Chief Executive being absent from the meeting. For instance, it would not be possible for the non-executive directors to discuss the point do we believe what the Chairman and Chief Executive are saying when those people are present at the meeting. Thus, the non-executive directors should have the opportunity to discuss matters alone. The note to the financial statements suggests that either the Chairman or Chief Executive would be present at the meeting. The likely close relationship between the Chairman and Chief Executive could result in the Chairman disclosing to the Chief Executive confidential matters relating discussions by the Remuneration Committee on the Chief Executives performance and remuneration. One would be more confident about the independence of the Remuneration Committee if the financial statements disclosed that the non-executive directors of the Remuneration Committee have the right to discuss matters without the Chairman and Chief Executive being present. (b) Like Diageo plc, the statement about Base Salary is qualitative rather than quantitative. It appears that the median-level base salary would be given even if the performance of the directors is poor. Also, mention of performance-related schemes does not seem relevant and appropriate in a section discussing Base Salary. As explained in the main body of the Chapter, it is difficult to determine the effectiveness of directors performance. The financial performance of a company may be poor because the directors performance is poor, or the performance may be poor because of a recession in the market for the companys products. Also, directors may work very hard when the company is performing poorly to overcome the problems it is experiencing. Directors should be paid on how hard and effectively they work, and financial performance may be a poor measure of their performance. (c) This paragraph explains that the annual performance award scheme gives a performance award of up to 60% of basic salary. It talks about a reducing scale of maximum percentages for senior employees - are these non-directors or do they include directors? It is unclear about whether the performance award is graduated, so that the achievement of the full performance requirement obtains 60% of salary, with reducing figures for lower achievement of the performance. Also, at what point is no performance award made?

322 Pearson Education Limited 2006

Barry Elliott and Jamie Elliott: Financial Accounting and Reporting (tenth edition) Instructors Manual

On the award itself, it says that a third of the award is paid in Rolls-Royce shares. How and when is the charge for these shares made in the financial statements? It would appear that the charge should be made when the shares are purchased. However, the charge could be avoided at that stage by including the shares at cost in the balance sheet. If a third of the award is made in shares, then two-thirds must be made in cash. When is the charge for the cash part of the award made? This charge is probably when the payment is made, whereas it would be better to make the charge during the period when the director works to achieve the performance target. The final statement in this paragraph says the performance award provides a culture of share ownership amongst the Groups senior management. From the companys financial statements, it is very difficult to see whether this is happening. The directors held more (or the same number of) shares at 31 December 1999 than they did a year earlier, but there was no information on the number of shares issued to directors in that period. (d) Once again, the directors can obtain an award of 60% of salary under the LTIP. This seems like double counting as they can also obtain 60% of their salary from the annual performance award scheme (APAS). This gives a total possible bonus of 120% of the base salary. The wording of the paragraph is very similar to that for the APAS, so the comments are similar. There is maximum award of 60% of salary, but is this proportionally reduced when the full target is not met? There is some explanation of how the award is calculated, and no award is made if the companys performance is below average (i.e. 10th out of 19 or below). The timing of the charge for the LTIP to the profit and loss account is uncertain, in a similar way to the APAS. It is probable that the cost is charged when the shares are purchased, but the cash portion is only charged when the payment is made to the director, rather than during the period when the director is working to achieve the target. The last paragraph is clear in explaining that no award was made in the year ended 31 December 1999, and none was realised (i.e. paid). (e) The Combined Code recommends that rolling contracts should be no more than a year, whereas the company is providing a rolling contract of two years to most of its executive directors. The rolling contract means that if a director is dismissed, he/she is paid one (or two) years salary on termination. One would have to look at the financial statements of other companies to see if they are using a one-year rolling contract for directors, or a longer period. It appears that most companies now comply with the one-year term of the Combined Code, so Rolls-Royces terms are generous to its directors. On new directors who are initially given a two-year rolling contract, there is no explanation of the initial period after which the contract is reduced to one year. (f) The first paragraph under Compensation and mitigation seems reasonable, as it appears to prevent incompetent directors from receiving a large termination payment. Also, if a director was close to the retirement age it would be unreasonable to give the director a termination payment which extends beyond the retirement age. For example, if the director was exactly 64 and the retirement age was 65, then the termination payment should be limited to 1 year, so it only covers the period until he is 65. So, the first paragraph seems reasonable.

323 Pearson Education Limited 2006

Barry Elliott and Jamie Elliott: Financial Accounting and Reporting (tenth edition) Instructors Manual

The second paragraph gives an example of the application of the first paragraph. The compensation paid to the director seems generous and there are many unanswered questions. Although the individual was a director until the companys year end of 31 December 1999, he is not included in the list of directors and their biographies. So, there is no disclosure of the age of the director at 31 December 1999 and his position in the company. One would have to look at the previous years financial statements for this information, and, like many other shareholders, I have thrown away the 1998 financial statements! Thus, the financial statements should give biographical details of all directors who served the company during the year. There is a need for such a requirement under the Companies Acts or Stock Exchange codes. The other unanswered question is why did the director retire early? The Chairmans statement only thanks the retiring director for his services, and the only other information on this payment is given in this question. How long had the director served the company and in what positions? In practice, it is unlikely that companies will disclose reasons why directors leave the company or retire early. Sometimes, they express appreciation for the directors contribution to the company. However, there may have been heated disagreements between the directors with the losing directors leaving the company. These losing directors may be given a generous termination payment on the understanding they will keep quiet about the dispute. There is likely to be little or no mention of these problems in the companys financial statements. Also, full disclosure of the problems by the company is likely to lead to expensive legal action by the losing directors against the company. It is probably much better to obtain information about the directors resignation from the financial press than from the companys annual financial statements. From the figures given in the question, it is apparent that the retiring director has been given two years basic salary in compensation payment. This is very generous. Also, there is almost no disclosure of information about the director in the financial statements, so it is impossible to determine whether there should be any reduction in the termination payment, using the rules in the first paragraph of the statement. It appears that many UK companies are very generous in their termination payments to directors. Very few UK employees who are not directors are given two years termination payment when they retire early. Some directors can earn large sums from termination payments. For instance, a director working for company A may obtain a job at company B in six months time. Then, he makes a nuisance of himself at company A which results in him being dismissed from company A with a generous compensation payment. He then moves on to company B, obtains a job at company C, makes a nuisance of himself at company B and is dismissed and so on!

324 Pearson Education Limited 2006

Barry Elliott and Jamie Elliott: Financial Accounting and Reporting (tenth edition) Instructors Manual

Chapter 31: Question 3 Risk-Averse Auditors


Explanations of why auditors perform more work than is necessary include: (d) being sued involves a lot of scarce senior management time (e) it is very worrying to be sued, so auditors would want to keep the number of claims to a minimum (f) being sued damages the reputation of the audit firm, which is likely to reduce the number of new audits acquired and losing some existing audits (g) if the auditor is found to be negligent, he/she will have to pay out damages. Most of this would be covered by Professional Indemnity Insurance (PII insurance). If there are many claims, the PII insurance company is likely to increase the insurance premiums

325 Pearson Education Limited 2006

Barry Elliott and Jamie Elliott: Financial Accounting and Reporting (tenth edition) Instructors Manual

Chapter 31: Question 4 Audit Firms and Consultancy


(a) An audit firm can provide significant help to the audit client through consultancy work. The audit firm will know the audit client from the audit, so this will reduce the learning time when carrying out consultancy work, thus reducing its cost. Also, audit firms have knowledge in specialised areas, which would benefit the audit client. Audit firms have been developing financial and related business services as part of their consultancy business, so there are many areas where the audit firm can help the audit client. The audit firms have specialised skills in information and computer systems, and E-commerce and Internet applications. Also, audit firms have always had specialists who deal with company and personal taxation, and accountants skills in this area are probably better than those of any other business. So, it can be seen that auditors can provide high quality services to audit clients, often at a lower cost than could be provided by other consultants. (b) The provision of consultancy services creates independence problems for the audit firm: (i) The auditor may be reporting on his/her own work, such as when reporting on financial statements prepared by the auditor, or accounting systems which have been recommended by the audit firm. If the auditor both prepares and audits the financial statements, the quality of the audit will be less than if these two functions were undertaken by different people. This is because one is poor at checking the accuracy of ones own work. An independent person is much better at detecting errors in another persons work. Also, if the auditor finds errors in work carried out by the firm, he/she will be reluctant to highlight these errors, as they could be a sign to the audit client of the low quality of the audit firms work. So, the audit firm may give an unqualified audit report when the audit report should have been qualified (modified) because of material errors in the financial statements. (ii) the second problem is that with the higher fee from the combined audit and consultancy work, the auditor will be reluctant to qualify the audit report, as this could result in a loss of both the statutory audit and the consultancy work. The profitability of the nonaudit work for the FTSE 100 listed companies is six times the profits from audit work. (iii) The ethical rules of most of the accounting bodies, the IFAC and the US SEC say the auditor should avoid making management decisions when performing consultancy work. However, there are problems in defining the situations when the auditor would be making management decisions, and it is difficult for third parties and regulators to detect whether audit firms are carrying out management decisions for the client company. The easiest solution to this problem is to prohibit auditors from carrying out consultancy work for audit clients. (c) Audit firms want to continue to perform consultancy work because of the high profits from this work. If an audit firm has acquired a new audit, it is both a regular annual income stream from the audit, and, being auditor, the firm has a greater chance of being selected for consultancy work than competitor consultants. This is because the audit firms consultants will be able to avoid some of the learning costs and the audit client will know the audit firm (and probably have a good relationship with the audit firm) so they will feel more confident in awarding the consultancy work to the audit firm than to a consultant who they have no experience of.

326 Pearson Education Limited 2006

Barry Elliott and Jamie Elliott: Financial Accounting and Reporting (tenth edition) Instructors Manual

Chapter 31: Question 5 Auditor Accountability


(a) Where the auditor reports to shareholders, the work the auditor carries out is determined by the information required by the shareholders. In a statutory audit (i.e. one governed by the countrys legislation) the work the auditor needs to carry out is determined by the information the countrys law requires the auditor to report on. In addition, it is common for the statutes to give the auditor the right of access to the companys accounting records and to obtain explanations from the companys staff, including the directors. In this situation, it is the shareholders or the countrys statutes which determine the work the auditor is required to carry out. In this situation, the directors cannot limit the work the auditor is required to carry out. This is quite different from situation (b). (b) Where the auditor is providing consultancy services for the client company, the directors specify the work the audit firm is required to carry out. Thus, the audit firms responsibility is to the directors of the company. For consultancy work, the directors can prevent the audit firm from looking at parts of the companys business. This limitation of the auditors work is not allowed for the statutory audit. So, for consultancy work, the auditors work is controlled by the directors. For the audit, the auditors work is determined by statute (or the shareholders) and it cannot be limited by the directors. If the directors tried to limit the auditors work in carrying out an audit, the auditor would probably give a qualified (modified) report on the financial statements, stating the way the directors have restricted the auditors work and its possible effects on the financial statements.

327 Pearson Education Limited 2006

Barry Elliott and Jamie Elliott: Financial Accounting and Reporting (tenth edition) Instructors Manual

Chapter 31: Question 6 Auditors as Shareholders


There is a concern that auditors should be impartial and also be seen to be impartial in carrying out their duties as auditors. This means that they should have no personal pressure to influence the reported income of the business. Whilst it is of course possible for an independent auditor to ignore the personal implication of the shareholding and to act professionally and objectively, the public might well take a critical view. Consequently the professional bodies prohibit an auditor from holding shares in a client company.

328 Pearson Education Limited 2006

Barry Elliott and Jamie Elliott: Financial Accounting and Reporting (tenth edition) Instructors Manual

CHAPTER 32

Chapter 32: Question 1 Plus Factors Group plc


(a)
Value added statement for year ended 30 September 20X9 20X9 000 Turnover Bought-in materials and services Value added by group Share of profits of Associate Applied the following ways: To employees To providers of capital To government For asset maintenance and expansion 2,193.5 735.7 464.7 415.2 3,809.1 57.6 19.3 12.2 10.9 100.0 2,153.6 566.5 527.9 226.4 3,474.4 62.0 16.3 15.2 6.5 100.0 8,613.6 4,815.4 3,798.2 10.9 3,809.1 100.0 % 20X8 000 7,560.1 4,096.4 3,463.7 10.7 3,474.4 100.0 %

(b)
20X9 Sales per employee Value added per employee Average remuneration per employee 43,974 20,379 11,201 20X8 37,612 17,232 10,714

(c)

Problems arise because there is no standard defining the terms e.g. turnover gross or net of VAT, treatment of minority interests, should VAT appear in the government section?

329 Pearson Education Limited 2006

Barry Elliott and Jamie Elliott: Financial Accounting and Reporting (tenth edition) Instructors Manual

Working 20X9 20X8 000 Turnover Sales including VAT VAT at 15% Bought-in materials and services Cost of materials Creditors at end of year Add: Payments in year Less: Creditors at start of year Materials purchased in year Add opening stock Less closing stock Add: Bought-in services Auditors remuneration Hire charges Other overheads Employees Benefits Pensions Salaries and wages Providers of capital Debenture interest Debenture interest Discount on debentures Dividends Preference Preference Ordinary Ordinary Minority interest [7% of 200,000] [7% of 500,000] [8m at 4.28p] [10m at 4.69p] 469.0 167.2 735.7 144.1 566.5 35.0 342.4 14.0 [11% of 600,000] [11% of 550,000] 60.5 4.0 66.0 12.2 66.5 1,012.4 4,815.4 109.9 319.8 1,763.8 2,193.5 11.9 367.3 738.3 4,096.4 68.4 222.2 1,863.0 2,153.6 1,244.2 3,622.9 4,867.1 1,109.1 3,758.0 804.1 (837.8) 1,109.1 2,971.4 4,080.5 987.2 3,093.3 689.7 (804.1) 9,905.6 1,292.0 8,613.6 8,694.1 1,134.0 7,560.1 000

330 Pearson Education Limited 2006

Barry Elliott and Jamie Elliott: Financial Accounting and Reporting (tenth edition) Instructors Manual

Maintenance and expansion of assets Profit before tax Less: Taxation Minority interest Dividends Retained profits Depreciation 1,437.4 464.7 167.2 504.0 301.5 113.7 415.2 1,156.4 527.9 144.1 356.4 128.0 98.4 226.4

331 Pearson Education Limited 2006

Barry Elliott and Jamie Elliott: Financial Accounting and Reporting (tenth edition) Instructors Manual

Chapter 32: Question 2 David Mark


(a) Your proposal to close the branch is ill advised. Apart from the social implications of closure referred to below, the loss accruing to your organisation based on 20X4 figures would be approximately 2,800 made up as follows:
Contributions/gross profit lost Expenses/costs save: Salaries and wages (all) Rates (all) Advertising (specified) Delivery van expenses (all) General expenses assuming all relate to branch Telephone (specified) Wrapping materials Loss if closed 78,540 2,865 1,320 5,280 1,188 1,056 2,640 92,889 2,811 95,700

(b)

Increased turnover if Peters suggestion is followed

To cover 125,500 expenses (including presumably the extra staff required) will need additional gross profit of (125,500 111,237) 14,263 thus requiring
Sales (14,263 4) of But current expenses of 111,237 are not covered by currently generated gross profit because a loss of 15,537 occurs. If this is to be absorbed then additional turnover is necessary (15,537 4) Total additional turnover 62,148 119,200 57,052

This assumes that the branch will be expected to absorb existing fixed charges i.e. salary of D. Mark 10,560, advertising 1,320 and telephone of (1,584 1,056) 528 and, if demanded, the delivery charge attributable to Arton of 5,280. [Total costs estimated of 125,500 have presumably allowed for additional wages and the van charge; or additional wages, having deducted the van charge. One way or the other the wages figure will have compensated for the van be it a plus or minus. If van is included then wage figure will be incorrectly budgeted in the data of the question, because it should have been excluded.]

332 Pearson Education Limited 2006

Barry Elliott and Jamie Elliott: Financial Accounting and Reporting (tenth edition) Instructors Manual

Extra part-time workers necessary per formula =

119,200 30,000

= 4

If figures are to be based on costs specific to the branch of 92,889 then the additional turnover will still be 119,200 because Peters expenses of 125,500 remain unadjustable for fixed expenses, be they included or excluded, in this solution.
i.e. (125,500 92,889) 4 = But the 92,889 already includes a contribution of 2,811 via sales (4) of So extra turnover is: Or required turnover of 25% gross profit content to generate absorption of Peters estimated costs of 125,500 = 4 Current level per accounts Additional turnover = 502,000 = 382,800 = 119,200 11,244 119,200 130,444

(c)

Comments on social implications of closure

1. Loss of a local shopping amenity in village. 2. Inconvenience to local residents travelling to nearest supermarket. 3. Loss of employment for 8 people and loss of the benefit of their disposable income if they are local residents. 4. Impact on family life with parents having to work. Comments on social implications of Peters recommendation. This would avoid the problems referred to above.

333 Pearson Education Limited 2006

Barry Elliott and Jamie Elliott: Financial Accounting and Reporting (tenth edition) Instructors Manual

Chapter 32: Question 3 Hythe plc


(a) Value added statement year ended 31 December 20X6
20X6 000 Turnover Bought-in materials and services Value added Applied in the following way: To pay employees Wages and salaries To pay providers of capital Interest on loans Preference shareholders dividend Equity shareholders dividend To pay government Corporation tax To provide for maintenance and expansion of assets Depreciation Retained profits 155 2 157 1,849 Value added per employee Sales per employee Average earnings per employee 46,225 128,100 20,250 100.0 8.4 0.1 144 70 214 1,834 43,667 109,619 18,952 100.0 7.9 3.8 402 21.7 393 21.4 168 24 288 480 9.1 1.3 15.6 151 24 256 431 8.2 1.3 14.0 (W2) 810 43.8 796 43.4 (W1) 5,124 3,275 1,849 100 % 20X5 000 4,604 2,770 1,834 ___ 100 %

334 Pearson Education Limited 2006

Barry Elliott and Jamie Elliott: Financial Accounting and Reporting (tenth edition) Instructors Manual

20X620X5 Workings (1) Bought-in materials and services Materials consumed Fuel consumed Hire of plant and machinery Auditors remuneration (2) Wages and salaries Wages Salaries 607 203 810 598 198 796 2,934 290 41 10 3,275 2,482 242 38 8 2,770 000 000

(b) A value added statement is a measure of the wealth created by a business. It is the amount
of value added by manufacturing, distribution and other businesses to the cost of raw materials, products and services purchased. It shows the total wealth created and how it was distributed, taking into account the amounts retained and reinvested in the group for the replacement of assets and development of operations. Financial statements have been regarded as primarily intended for equity investors whose interest has been focused on profitability, capacity to adapt and solvency. The value added statement has perhaps been seen as of more interest to staff who have had little recognition by standard setters. Even in 2004 when there is a growing interest in social, environmental and ethical issues there is no financial reporting standard relating to human asset accounting in the balance sheet or value added statements. There is a further argument that the data already appears within the existing primary reporting statements and that there is consequently little point in producing yet another statement.

335 Pearson Education Limited 2006

Barry Elliott and Jamie Elliott: Financial Accounting and Reporting (tenth edition) Instructors Manual

Chapter 32: Question 4 Gettry Doffit


1 Quantities of chemicals received by the company for disposal on site represent a liability for costs of disposal at the year-end. The work would be undertaken by Gettry Doffit plc on a contractual basis and, clearly, income from contracts (short-term as defined in the original SSAP 9) should not be credited to profit and loss account until the work has been completed i.e. on the completion of the contract. Therefore, it would appear that such quantities should be carried in the balance sheet as a liability at the higher of: (i) invoice cost to the customer or (ii) estimated cost of disposal. Applying these principles to (A) axylotl peroxide and (B) pterodactyl chlorate: Re (A) This contract will give rise to certain revenue of 87,179 i.e. 170 million won @ 1,950 to the . This is because the invoice value in won has been sold forward at the stated rate of the forward contract. It is therefore appropriate, and permissible per SSAP 20, to use the forward rate as the transaction value in the books at all dates, and given such treatment, no exchange differences will arise. There should be a debtor and creditor for this amount in the balance sheet i.e. the debtor for the certain amount receivable should not be dealt with as income until the contract is completed. Any profit arising would be dealt with in the year to 31 March 20X6.
Dr Debtors 87,179 87,179

Cr Creditors accruals and deferred income

It is possible that the company could choose as a matter of accounting policy to use rates on the date of the transaction and then retranslate on settlement/balance sheet date giving rise to exchange differences. The alternative numbers arising are dealt with below. The costs incurred up to the year-end will be dealt with as follows
Dr Creditors Cr Bank 60,000 60,000

The creditor balance would be debited with the estimated further costs to completion of 15,000 in 20X5/X6 leaving the company with profit of 12,179 in 20X5/X6. Also in 20X5/X6 1.5.X5 the company would receive 170 million won and realise, per the terms of the forward contract, 87,179, thus eliminating the debtor. Had actual rates been used:
Dr Debtors 170 million won @ 1,900 = Cr Creditors 89,473 89,473

The balance on creditors in 20X5/X6 will then be a profit of 14,473. However, the debtor would have to be retranslated at the 20X5 year-end 170 million won @ 2,000 to the = 85,000 giving rise to a loss in that year of 4,473. On settlement the debtor will realise 87,179 giving rise to a gain of 2,179.

336 Pearson Education Limited 2006

Barry Elliott and Jamie Elliott: Financial Accounting and Reporting (tenth edition) Instructors Manual

In total, 14,473 plus 2,179 less 4,473 = 12,179 (as when the forward rate was used to start with) would be credited to profit and loss account, though in this case partly in 20X4/X5 and partly in 20X5/X6. Re (B) As this is disposed of per the terms of this contract, neither a debtor nor a liability arises. The point where revenue should be recognised is the date of processing, and it is clear per the terms of the contract that no loss can arise. The costs of the break-down should therefore be carried forward as work-in-progress, perhaps reduced for the worth of the by-products. 2 The won forward contract has been exhaustively dealt with above. As the contract to buy dollars is to be used to finance trade purchases overseas, the transaction poses no problems provided the dollars will be used to purchase stocks whose realisable amount is greater than (70,000 @ 1.60) = 43,750. Indeed, it would make sense not to reflect such a contract in the accounts, it being more appropriate to disclose the detail under commitments. There are, however, other pertinent points to be made. If the dollars are not to be applied towards a trade purchase, the company would have surplus dollars which may only be converted back to sterling at a loss. Such a loss should be recognised in accordance with the prudence concept, although there may be mitigating factors such as an alternative use for dollars. 3 Given the raising of the irrevocable letter of credit, all that the Nigerian supplier has to do is to ship the goods specified in the letter, present the bill of lading as proof of shipment, and await payment. Thus, the company must pay for goods supplied in accordance with the contract terms, and cannot cancel. Therefore, a liability of (130 90)/130 65,000 = 20,000 should be recognised immediately, unless a variation can be negotiated with the supplier or an alternative use found for the chemical. 4 The spillage is a post balance sheet event. No liability should be recognised in the accounts unless the going concern concept is threatened. However, the potential liability is so material as to require disclosure under SSAP 17. In a normal joint venture the companies trade as partners, with joint and several liability. The precise apportionment of the liability may require a contribution from Dumpet Andrunn plc. If they cannot pay, it is likely that Gettry Doffit plc will have to. The likelihood of a liability crystallising, the likely amount, and any recovery from Dumpet Andrunn plc, must be assessed, and full details given in the notes to the accounts and referred to in the directors report. As it is likely that the company will resist the claim, the maximum payable should probably be disclosed as a contingent liability. The possibility of an insurance recovery should also be examined.

337 Pearson Education Limited 2006

You might also like